Вы находитесь на странице: 1из 304

r ,I

!
II
4.7 Beams/Shear at"!,d Moment Distributions

prevent or relieve large axial stretching stresses when it is loaded, and/or


the potentially large axial stretching or corrtpressing stresses when it is
cooled or heated. Thus for a simply supported beam without applied axial
(x direction) loads, the horizontal componedt of the piri reaction will be
zero. Let us proceed to find the disttibutiorl' of shear force and bending
moment in the beam of Figure 4.36. .

"

E x A M p L E 4j14~
~-
~'I~;,j
---4ft--+---6ft~'..
x I." ~ ~
Find the distribution (as functions of xl of the norIPal (N) and shear
bending moment (M) for the beam shown in the figure.

SOLUTION
i
WI forces and

For finding the reactions, we replace the uniformly distributed load by the equi-
.-
,
,

pollent single force shown in the free-body diagi~m. Note that Px = 0 because

Px = 0 by ~Fx = P!
'--1---
I 900lb
1
I
!
it is the only external force in the x direction. Sutnming moments about the pin
gives the roller reaction:

1:Mp = 0 = R( 10 ftl - (900 Ib)(7ft):i


,

1=4 ft--I--6ft--t R
R = 630lb
I'

'i

Then

LFy = 0 = Py + R ~ 900 = Py = 270lb


,
As a check, ii
II
1:MR = 0 = (900 Ib)(3 hI - (270 Ib)(10
,
ft) = 0

Now we proceed to the determination of V(x) and M(x) j note that N(x) == 0 in this
beam.

If we make a sectional cut between x = 0 and x = 4 ft, we can obtain


expressions for V and M valid throughout that idterval (see Figure I).
We always place V and M (and N, in beam~' where it is present) on the cut
sections in positive directions according to our sign conventions. Hence in Fig-
ure I, V is drawn upward and M counterclock.yise on the exposed right-hand
Figure 1 face. Equilibrium then requires :1

+t 1:Fy = 0 = 270 + v=v= -270Ib


,I
(I)
~ 1:MA = 0 = M- 270x => M = 279x lb-h (2)

(Continued)
1
~ ,!
the reader should note that at x = 0, these expr6ssions give the correct V and
M-namely, a negative shear force of 270 Ib (up orl a left-hand face is negative VI
and a zero moment.:
From x = 4 ft to x = 10 ft, Equations (11and (21will not be valid because
the distributed load "starts" at x = 4 ft and was i~ot included in the free-body
diagram of Figure 1. Thus we must make a new cut,~henever something changes,
1
such as a distributed load starting or ending, or concentrated load or couple
I
appearing. The new free-body diagram [Figure 2 (alHwillbe valid for the rest of the
beam (4 ft < x < 10 ft). '
ISO(x - 4) Ib
ISOlb/ft
"
, '

:--~;~4f\M
A/4J
2701b
t
270lb

Figure 2(a) Fi~ure 2(b)

For purposes of finding V and M from the e4uilibrium equations, we may


replace the (part of the) distributed load by its equiJollent /'force-alone, // shown in
Figure 2(bl. Then we obtain, from the equilibriurA equations for the material to
the left of the cut section, '

+t r.Fy = 0 = 270 - 150(x - 4) + V

V = 150(x - 41- 2701b

and

~ :EMA = 0 = -270 Ib(x ft) + [150(x 4) Ib{ (x ~ 4}tJ +M

M = 270x - 75(x - 4)21b-fi

Note as a check that at x = lOft,


Vlx=lOft = 150(10 - 4) - 270 = 6301bl
which is the shear (value and sign) produced by tHleroller reaction R on the right-
'j hand fac{ at x = 10 ft. Also, ,!

Mlx=IOft = 270(101 ~ 75(10 -412 =0


I j.

This result also agrees with what is actually gOing,!onatthe right end of the beam,
for there is no moment there. These "self-checkst should always be made. They
are not foolproof, but the chance of making an enor in each of V(xl and M(x) that
still give the correct values at the right end of th~ beam is highly unlikely.
'J

,i

The reader is asked to note for funhe reference that, in both


segments of the beam in Example 4.14, we have dM/dx = - V. Also, if
we define q(x) to be the distributed load irltensity, positive if upward,
then dV/dx = -q:

304
:i
r ,I
II
4.7 Beams/Shear andlMomenl Distributions 305
Ii.1
i'
0<x<4ft 4 ft <,Ix < lOft
,
dV dl-270) dV d[1501x - 4) - 270}
dx dx dx , dx
it
=0 1501b/ft
=
i
This is - q(x) because This is - q(x) because
q(x)= 0 here. q(x) = - I 50 lb~ft here.
Ii
dM d(270x) dM d[270x - 75(x - 4)2)
dx=~ dx dx

= 270lb = 270 ~ 150(x - 4)lb


This is - V(x) because This is - V(x) be~ause V(x) =
V(x) = - 270 lb here. - 270 + 150(x :L 4) Ib here.
I

:i
These relationships turn out to be true in general; we shall derive
and study them in the next section of thisch~pter.

~ 'I
E x A M p L E

; .' i 2500 Ib-ft Obtain expressions for the shear force V(x) and bending moment M(x) for the
....
::vwcsxew0. --~~. cantilever beam loaded as shown in the figure. !

o 6 ft .
7"" " .

~
Ii
,

SOLUTION
A cantilever beam is one which is fixed at one end into a structure (such as a wall)
l
and juts out with its other end unsupported, or "free ." The free-body diagram will
help us compute the reactions at the wall. [Note th~t N(x) is again identically zero
here.] ",

M-C1f---
!-3ft-

---lP
120(6)= 7201b

2500I.f,

r.F y = 0 = - 720 + Fw = Fw = 7201b :1


(11
r.Mo = 0 = 2500 lb-ft - (720 Ibl(3 ftl - Mw
(2)
if

Mw = 3401b-ft

(Continued)
~.

ii
In this problem we are lucky because a singleilcut section (see the figure at the
left) will yield V(x) and Mix) expressions valid for the entire beam.
To find V and M, we replace the distribut~li load by the force I 20x lb located
at x/2 as shown in the figure below: .
x . 1

Equilibrium of the cut section requires

+ t LFy = 0 = V + 720 - 120x


(3)
V = 120x - 720 lb

LMA = 0 = M + (I 20x lb) G )1-


ft
I
340 lb-ft - (720 Ib)lx ft)
(4)

M = - 60x 2
+ 720x + 340 lb-ft
Note that:

a. At x = 0, V = - 720 lb, which checks. :1


b. At x = 0, M = + 340 lb-ft, which checks.
c. At x = 6 ft, V = 120(6) - 720 = 0, which checks.
d. At x = 6 ft, M = -60(6)2 + 720(6) + ~40 = 2500 lb-ft, which checks.

It is our experience that students ofteb have trouble, at least at first,


in distinguishing between the meanings ~f:
a. The sign beside an equilibrium equ~tion, such as the in Equa- +t
tion (1) of the above example; and !!
b. The sign given to the scalar V(or to th~ scalar M) representing a shear
force (or a bending moment).. ,:;
II
The difference between these signs is explained as follows:
The sign +t
is a temporary conveniehce, where we are agreeing ori
which unit vector is factored when the fotces (or moments) are summed
and equated to zero. In the preceding elample, Equation (1) could be
written more formally as the y componen!~ of LF = 0:
720(-,) + Fw(J) = 0 I

We see that if we suppress the ",," Equati'on


I'
(II of the example remains.
.~
t
Thus the sign + in front of that equatio~ is merely saying it is "plusj"
that is omitted, and has nothing to do wit~ a shear force sign convention.
If we, however, want to know the shear fd~ce on the left end of that same
beam, it comes from an upward force (of Fi = 720 lb) on a left-hand face,

306
..j:
4.7 Beams/Shear and Moment Distributions 307

which is negative shear force by the sign convention for V. that is V =


- 720 lb there at x = o. .."
In our final example of this section, 'Ye shall consider a simply
supported beam loaded in four commonly of cur ring ways: by concen.
trated axial and lateral forces, by a concentrated~couple; and by a dis.-'. <

tributed loading. '

E x A M p L E 4. :~6••••••.....
Find the axial force, shear force, and bending morAent distributions throughout
the beam shown in the figure. Ii

x
A

-1"""'" II.sm Il.sm I D


f---3m-----------j--3 m-
~I~
SOLUTION
Forthe reactions, we shall use the free-body diagram in which the distributed load
(see Section 2.9) has been replaced, solely for purppses of using the equilibrium
equations, by the area under the loading curve acting along the indicated line.

__ 3m
2 kN
_______
+(3)8
2m
=1 12
kN;
/-'1

I.Sm I 1.5 ill I-- /./ I r


I // /'""1- 3(3) = I ill

Ax 4kN
e 3kN. ill '1'
Because the force-alone resultant of 12 kN r1acting 1 meter from the right
end of the beam produces the same contribution ti) the equilibrium equations as
does the actual distributed loading, it can be used to advantage in computing the
three reactions Ax, Ay, and B: 'I

"'£Fx = 0 = 4 kN - Ax =A x = 4 kN !i
(to the left as assumed I

"'£Fy = 0 = A y
- 2 kN - 12 kN +B I

"'£MA = 0 = -2kN(3m) + 3kN. m- 12kN(8ml + B(9m)


(Continued)
(6 - 3 + 96) kN . m'
B == ------- =' 11 kN
9m ,I

Therefore, from the "yequation,"

Ay = (2 + 12 - 11) kN

Ay = 3 kN

and the reactions have been found. Now we proc~ed to find N(x), V(x), and M(x) in
the variou~egments of the beam. There williibefour such segments: (0, 3)i
(3,4.5), (4.5 6), and (6, 9) because at x = 0', 3, 4.5, and 6 m, we have
v concentrate forces, couples, and/or the begl'nni.,.,ng of the distributed load.
M
We fir cut the beam at a value of x betweeh 0 and 3 meters, 0 < x < 3 m.
N We consid rthe section to the left of the cut sh~wn in the figure.
4kN
Note that the sign convention is always ini!agreement with Figure 4.34 for
3kN
the "right.hand face" above. To determine N, V, and M, we simply ensure that
I~x the section is in equilibrium:

L. r.F" = 0 = - 4 kN + N => N = 4 kN
+t r.Fy = 0 = 3kN + V=> V'= -3;kN

~ r.MA = 0 = M - (3 kN)(x m)

M = 3xkN' m i
i
Therefore, over the first 3 meters of the bedin, the axial and shear forces are
constant while the bending moment grows lin~arly with x. Note too that the
derivative of M with respect to x equals the value Of (- V). As we have mentioned,
this will later be seen to be true in general.:
We next cut the beam between the 2.kiN' and 4.kN forces. From the
equilibrium equations, we obtain

2kN r.Fx = 0 = N - 4 kN => N = 4 kN


:;

r.Fy = 0 = (3 - 2) kN + V
4kN
N
A V = -1 kN

r.MQ = 0 = M + (2 kN)(x - 3) m- (3 kN)(xm)


3kN
M = (6 + x) kN. m ~. "!
'I
We note that the values of N are the dme on either side of the point
x = 3 m. This is also true for M; approaching x = 3 m from the left, M ...••
3(3) = 9 kN . m, and from the right, M ....•61+ 3 = 9 kN . m. Thus Nand
M are continuous through the point, but not sb for V.* A free body surround.
i
(Continued) ..r,'

* This is to be expected because we have no con*entrated longitudinal force or couple


applied at the point, but we do have a concentra~ed transverse external force there.
Concentrated forces or moments will always and only give rise to discontinuities in the
respective related quantity N, V, or M. "

308
r:G--------~--------------
(
.
\
I

ing the point shows the situation, and why there i'k no single value of Vat the
point x = 3.' . j

Passing next over the concentrated axial force of 4 kN at x = 4.5 m, we see


that to the right of this force, there is no axial force!in the beam:

....:!:.... r.Fx = 0 = N - 4 kN + 4 kN
N=O
(showing only shear forces)
V and M this time retain their earlier expressions, as the reader may note:

V=-lkN
M = (6 + xl kN . m

3m

3kN

When we reach point D, two things happen:

1. There is a concentrated couple there that causes a jump in the expression for
M(xl.
2. A distributed load begins that will affect both V and M.

s(x ~i6)kN/m
2kN
/ V
M
N
4kN

~3_k_N X~~

I
Shown in the figure is a free-body diagram of a s~ction of the beam to the left
of a cut at a point with x > 6 m. Equilibrium requires

....:!:.... r.Fx = 0 = - 4+ 4 +N => N = 0

. (Only the portion of the beam being stretched carri~s any axial force.)

r.F = 0 = 3 -
y
2 + 1
V - - (x -
2',
[ ,:(x - 6)J
6) 8.' --
3
,i
i
area of triangular loading
curve (see Ex~mple 2.36)

V = - 1 + -4 (x - 6)2 kN
'. 3
(Continued)

309
<;"!"<'-------,..I---------------------------

;1
(Note that as x grows from 6 m, the shear must become more and more positive.)
i

'+) LMQ = 0 = -3x + 2(x - 3) + 3 't M


+ '34 Ix - 6) 2 (x -3 6)
downward force ; location of
from distributed , resultant of
load from distributed load
preceding equation Is~e Example 2.361

41
M = 3 + x - - Ix - 613 kN . m
9

As checks on the solution, note that at x = 9 m:

N=O

V = ~1 + -4 (3 2 ) 11 kN
3

4
M = 3 + (9) - ~(33) = 0
9
i

This check is at the right end of the beam shown in the figure. We have:

N should be zero: ./
V should be 11 kN. t/
M should be zero. t/

P R 08 L EMS / Section 4.7

4.102 At what point Is) on the beam shown in Figure a. Zero moment
P4.102 do the following occur 1 b. Maximum moment li~ivethe value)

a. Zero shear 4.104 Repeat the precedii),gtwo problems for the beam of
b. Ma){jmum sh,ear (giye the value) Figure P4.104.Assume the reaction of the ground onto the
beam is uniformly distriblhed.

) B
x

Figure P4.102 Figure P4.104

4.103 In the preceding exercise, at what point(s) on the


beam do the follbwing occur?

310
4.7 Beams/Shear and: Moment Distributions
!i
311

4.105 For the beam in Figure P4.10S, what is the bending 4.110 A signboard, Be, is, to withstand a windload of
moment of largest magnitude and where does it occur 1 w = 80Ib/ft. It is supported by the structure shown in.
Figure P4.11 O.
4.106 How do the shear force and bending moment vary
in the central 3-ft segment of the beam in Figure P4.106? a. Determine the magnitude of the windload on the
signboard, Be. "
4.107 Find the maximum shear and bending moment in b. Identify each of the tw6-force member(sl, if any.
the beam shown in Figure P4.107. c. Determine the force in each two-force member.
d. Determine the horizontal and vertical components of
4.108 Given the b~am shown in Figure P4.108:
the reaction forces at A!iand F.
a. Find the axial force (N), shear force (V), and bending e. Show the axial and shear forces and the bending
moment (Ml in the portion of the beam along x, as moment at point H of:member DEF, and show the
functions of x. results on a free-body diagram that includes the cut
b. Sketch the graph of the functions in part (al and label section there.
the values at x = 0, 2 m, and 4 m.
4.111 InFigureP4.111:
4.109 In Figure P4.109: ii
a. Find the reactions at the wall for the cantilever beam.
a. Find the reactions at the wall for the cantilever beam b. Find the shear force, bending moment, and axial force
shown. as functions of x.
b. Find the shear force, bending moment, and axial force
in the beam a~ functions of x.
c
1000N 250N 30°
D

x -
t t
5 ft
10ft

11m
J
~
Figure P4.105
l-2m. -2m---l 1000N

Figure P4.108

100Ib 100 Ib

," .~ Figure P4.11 0

2501b/ ft
Figure P4.106

, 3001b
x I 150I

1---6ft-
,~ft .
f~ Figure P4.109

~"

1.:.:_ m--l
15 } 1200 Nlm
9000N/m { } 4500 N/m

m: . L= 3m----.1
Figure P4.111
Figure P4.1 07
312 Chapter 4 Structural Applications

In terms of P and L, find the extreme values of the shear 4.118 For the cantilever, beam loaded as indicated in Fig-
force V and bending moment M in the beams in Problems ure P4.118, find the shea~ force Vand bending moment M
4.112-4.117. Also give the locations along the beam as functions of x. Then find the shear force and bending
where these extreme values occur. (It will help to graph moment at the point where the bending moment in the
V(xJ and M(x).) beam is the largest in abs?lute value.

4.112
p
90N/m

Figure P4.112 Figure P4.118


'I 2m
.....1-.. L--~.I,~
----7 x

30N

Write expressions for the shear force V(xl and bending


4.113
moment M(xl in the beams shown in Problems 4.119-
P P/3

rl ..
~,:~1
4.122.

~..
4.119
R•••• P~113

~
Figure P4.119
4.114
P

4.120
Figure P4.114

4.115

Figure P4.120

4.121

6001b/ft FigureP4.121

A B

4.116
p 1=-3~~1~6ft :r~
Figure P4.116 4.122 .j

!l\l-o>------L----
Figure P4.122

4.117
x 2113---1
Figure P4.117
L
4.8 Differential Relationships in a Beam 313

200N

0.2 m • 0.2 ;1~B 0.5 m-I

~ ) x
Figure P4.124
Figure P4.123

)x
200N/m 600N
100lb/ft

Figure P4.126
Figure P4.125
m I k-i
.1
.---)-,>X
0.2m 0.2m ~

4.123 For the beam shown in Figure P4.123, write equa-


tions for the shear force V and bending moment M as
functions of x, in the intervals a < x < 0.1 m, 0.1 < x <
0.4 m, 0.4 < x < 0.6 m, 0.6 < x < 0.8 m, 0.8 < x <
1.0 m, and 1.0 < x < 1.5 m.

4.124 For the beam shown in Figure P4.124, write equa-


tions for the shear force V and bending moment M as Figure P4.127
functions of x, in the intervals a < x < 5 ft, 5 < x <
25 ft, and 25 < x < 31 ft. 4.126 Find expressions for the shear force and bending
moment in the beam shown in Figure P4.126. What is the
4.125 In Figure P4 .125:
bending moment of largest magnitude and where does it
a. Write algebraic expressions for the shear force and occur?
bending moment in the beam.
b. Find the shear force of largest magnitude in the beam. 4.127 In Figure P4.127 find the shear force V(x) in the
c. Find the bending moment ofJargestmagni.tllde.in,the , " segment of the )Jearn
.
a ,:<
.1
x. < 5 ft and the bending
beam. moment MIx) in the segment 5 < x < 10 ft.
I

;i
Differential Relationships Between q(x), Vex),
and M(x) in a Beam/Shear and Be~ding Moment
~4.8 Diagrams ..
We noted at the end of Example 4.14 that in both of the analyzed
segments of the beam, the derived expressions for q(x), V(x), and M(x)
satisfied the equations
dV dM
-q and
dx
-v
dx
,,,", ---------------------------------------------------

314 Chapter 4 Structural Applications

The same is true in Example 4.15, where

dv d(120x - 7101
-----= 120 = -q (since q = - 120)
dx dx

and

dM d(-60x2 + nox + 340)


--------- = - 120x + 720 = - V
dx dx

(since V = 120x - np)


The reader is encouraged to observe that the same two equations hold in
Example 4.16 in each of the four.analyzed ~egments of the beam.
These observations are no coincidence. There is this definite rela-
tionship between the shear force and ben4ing moment in a beam. Let
us assume we don't know the result and derive it "from scratch." We
begin by isolating an elemental segment ofia uniformly loaded beam as 'I
shown in Figure 4.37:

'! (positive upward)

~~ M~~lf):VHM
Figure 4.37

Note that, as we have seen, the values of V and M are generally


functions of x, so that from the left-hand face (at xl to the right-hand face
(at x + ~x), V and M respectively change tO V + ~V and M + ~M.
I

We now establish the equilibrium of the element:

r.F y = 0 = - V + IV + ~ V) + q ~x (4.1)
f
Dividing Equation (4.1) by ~x and simplifying, ,
(

~V
-q
~x

Taking the limit of both sides,

~V dV
lim - =- =-q (4.2)
£\x-+O ~x dx
4.8 Differential Rel~tionships in a Beam 315

When q is q(x), the value of the distributed load intensity at x, the result is
the same.*
Thus the slope of the shear diagram (a plot of V versus xl at any point
equals the negative of the distributed load value at the point. Let us see by
two examples how we might use this result in!obtaining the distribution
of V throughout a beam.

E x A M p L E

Draw the sheardiagram IV versus xl for the beam loaded as shown in the figure.

200lb 300lb

B 4ft

SOLUTION

X;(ft)

In this problem, the reactions are 230 lb i at the pin A and 270 lb i at the rollerB.
At the left end of the beam, which is a left-hand face, an upward force corresponds
to negative shear. Thus the shear diagram starts at the point V = - 230 at x = O.
(Continued)

*Note that if the distributed load q(x) is continuous but not uniform (i.e., not
constant I the argument is slightly more complicated.: By the mean-value theorem for
integrals,' there is a number X such that x ::; X ::; x +:~x
and J~::Xq(~1d~ = ~x qlXI.
Then as the limit is taken X -+ x so that q(X) -+ q(x), and the result is the same.
1 ~x-o '
In this beam, there is no q(x) anywhere, so the slope of the V diagram-that is,
dV/dx or V' -is zero throughout. Hence we draw a horizontal line from point A
of the diagram to the point where x = 3 ft, point B.
At B, there will be a discontinuity, or jump, in V because of the 200-lb
downward load there. As soon as we pass the point B, the shear force on a right-
hand face will have to increase by 200 lb to mai~tain equilibrium.

200

This represents V = -230


(upward force on left-hand
face is negative shear) l~ This represents V = -30
(downward force on righi.hand
face is also negative shear)

Therefore the value of V becomes - 30 lb: (or - 230 + 200) just past B,
and stays at that value until we reach point C. There, we get a second jump,
this time of 300 lb in V by the same reasoning' as at B. This puts us at V =
-30 + 300 = 270 lb. Again, because q(x) = a'.in this beam, We extend the
line straight across to D, and end at V = 270 lb. '
It is important to note that we have an automatic check on our solution
because we know the answer for Vat the right end of the beam, a right-hand face.
In this problem, we have an upward reaction ofi 270 lb there. Thus (up on. the
right is positive) V has to be + 270 lb at x = 1a fti otherwise we have made a
mistake. This automatic check should always be made because if you end at the
correct value of V, the chances are very good. that the diagram was drawn
correctly.

E x A M p L E

Draw the shear diagram IV versus xl for the beam of Example 4.16.

SOLUTION

We have found the reactions to be as indicated in the figure on the next page. We
present the shear diagram and proceed in the series of steps below to explain how
it was constructed. .
1. The reaction at the left end gives the ~tarting value of V. It i~ 3 kN
upward on a left.hand face, which is negative sh~ar. Thus we begin with a dot at
V = - 3 on the V axis.
2. There is no change in V between x = 0 and x = 3 m. This is because in
that interval, q = a (no distributed load), so that dV /dx = O.Thus the slope of V

(Continued)

316
r-
,,

I :
2kN
8kN/m

4kN A
'E
B
3kN II kN

!_3m __ 3m-_3m- ,.

II
V(kN)

6
x(m)
D

vs. x is constant at zero, and we draw a horizontal line across from the starting
point to B.
3. As we cross B, there is a sudden jump in V of 2 kN because of the
downward 2 kN load there. .
4. Again, there is no q(xl between x = 3 m and 6 m, so dV/dx = 0 again.
We draw another horizontal line from C to D on die shear diagram.
5. At D, the couple has no effect on V. But starting there, the value of q(x)
begins to get more and more negative, linearly. Because dV/dx = -q, the slope
of the V diagram must get more and more positiVe, linearly. The change in V
between two points is seen to be the negative of the area beneath the q diagram
between those points:

_dV = - q
dx
=> .•iV2

VI
dV = - IX

Xl
2 q dx

From x = 6 m to x = 9 m, the area beneath" q versus x" is !(3 mil - 8 kN Iml =


I.', •
=-12 kN. Thus V climbs by 12 kN from -1 kN, ending at 11 kN.
6. We have an!automatic check on our diagr~m because the reaction at the
right end is applying an upward force of 11 kN, on a right-hand face, which
corresponds to V = 11 kN, a built-in check.

q
Now that we have the shear diagram in hand, we are ready to study
its companion, the (bending) moment diagrafu. We begin by writing the
moment equilibrium equation for the segmerit of Figure 4.37 reproduced
at the left.

:LM
Q
= 0 = -M + (M + ~M) + IV +~V) ~x + (q ~Xl(~;)
317

t.L.-_ ..
318 Chapter 4 Structural Applications

Therefore,

.1M q(.1xl
-v -.1V ---
.1x 2 ,
"":

As the limit is taken (.1x -+ OJ, the last two terms in this equation
vanish, * leaving

dM
dx = -v (4.3)

Thus at each point where M' is defined, the slope of the bending
moment diagram is equal to the negative of the value of the shear force at
that point.
Also, we have

fX2

XI
dM
-dx
dx
= -
fX
2

XI
Vdx

or

so that the change in the bending moment between two points Xl and x2
equals the negative of the area beneath the shear diagram between those
two points. Let us use these results to extend the last two examples to
include moment diagrams.

*If q(x) is continuous but not constant, then again using themean-value theorem,
there is a number X such that x :::;X :::;x + !'J.x and J~::x(~-
x)q(~l d~ = !'J.x(X - xlqlx).
Then as the limit
J 6x-+O
x and the moment of the distributed load q(xl about Q
is taken ' X .....•
again vanishes.
E x A M p L E
Draw the moment diagram for the beam of Example 4.17, which carried the
loading shown in the figure.

230Ib 2001b 3001b 2701b

J.,~L.-LJD
I B C
V(lb)

x(ft)

x(ft) .

SOLUTION
It is helpful to draw the various diagrams beneath one another because we can use
the relationships such as V' = - q and M' = - V more easily. Also, we can for
our convenience extend lines downward at places ~f importance, such as points
where loads appear.
The moment diagram starts at the origin because there is no momen.t
applied to the beam by the pin and no external morrient is being exerted at the end
x = O. The slope of M versus x, for 0 < x. < 3 ft, is - V, or + 230 lb. Over 3 ft,
this slope will result in a buildup of M from zero to 690 lb-ft. Note that the change
in M from its value of zerO at x =0 is the negative of the area beneath the V
diagram, or - (3 ftl(- 230 lbl, which is, again, 690 lb-ft.
The jump in Vat x = 3 ft means to us that there is a discontinuity in the
slope of the moment diagram. The M versus x curve will come out of the point B
with a different slope [namely - (- 30) or 30] thari it had when it went in. Over
the next four feet, the change in M, then, will be the negative of the area under the
shear diagram between Band C, or - (4 ftl(- 30 lb) = 120 lb-ft. Hence the new
(and the maximuml value of M is the old plus the change, or 690 + 120=
810Ib-ft.
Another change in slope occurs in the M diagram, this time at point C. To
the right of this point, the slope is - (270) = - 270 lb. The change in M between

(C;ntinued)

319_
C and D is - (270113ftl = - 81 0 lb-ft. Thus the value of M at the right end of the
beam is 810 + (- 81 01 = O. This result serves as a check on our diagram (just as
we had with the shear diagram examples) because the value of M at x = 10 ft is
indeed zero at a roller with no external couple applied on the end of the beam.

E x A M p L E 4!.20~
i '
Draw the moment diagram 1Mversus x) for the beam of Example 4.18.

SOLUTION

We redraw the loading and shear diagrams below:

2kN

4kN A !B
3kN II kN
I
I-3m -3m--3m-
V(kN) E II

D x(m)

M(kN,.m)

o x(m)

We now detail the steps we underwent in sketching the moment diagram


(M versus xl above:

1. There is no moment applied to the left end of the beam, and the pin cannot
transmit a moment to the beam there ..Thus the moment diagram starts at
zero.
(Continued)

320
2. The shear force is - 3 kN between x= 0 and x = 3 m. Thus the slope of
the M diagram equals a constant + 3 kN 1m (it is minus the value of VI for
3 > x ~ O. Thus at x = 3, M = 3(3) = 9 kN . m.
3. At x = 3, the concentrated 2-kN force does not produce a discontinuity in
the M diagram. However, as soon as we passover x =3 m, the negative of
the value of V (and hence the slope of M vs. x) changes to 1 kN. Over' three
more meters, this positive slope causes a': further rise in M of 1(31 = .
3 kN . m, which equals the negative of the area under V vs. x between
x = 3 and x = 6. Thus M, at x = 6, is 9 +3 = 12 kN . m.
4. As we pass over x = 6 m, the moment diagram must experience a discon-
tinuity because of the concentrated couple applied there. The moment on
12 kN . m ( ) 9 kN . m the right-hand face just past x = 6 must drop by 3 kN . m, as shown in
the diagram. Thus the moment just to the right of x = 6 m is 12 - 3 =
3kN. m 9kN'm.
5. Between D and F on the shear diagram of EXample 4.18 the values of the
shear V get less and less negative, ending at Zero. Thus the slope of M must
get less and less positive ending at zero because dM/dx = - Vi the M
diagram is thus sketched with a horizontal tangent at F.
6. Between F and E, the shear gets more and more positive, ending at 11 i thus
dM/dx, the slope of the moment diagram, gets more and more negative,
ending with a slope value of - 11 at the right end of the beam.

Just as with the shear diagram, there is an,automatic check available for
the moment diagram. We simply see if the computed value of M at the right end
of the beam is the actuaf (known) value there. In this case, we know that M
must be zero at x = 9 m. To see if it calculates o~t to be zero, we need the area
beneath V vs. x in the interval 6 < x < 9. In that interval,

8
q = -- (x - 6)
3

so that, integrating,
4(x - W 4(x - 612
V = -
f q dx =

where C1 is found by the condition V


3 + C1
= - 1 at x
3

= 6.Integrating a second time,

4(x - 613
. M=-fVdX= - .+ x + C2
9
4(x - 6)3 . 3
- +x+
9 '

because M = 9 at x = 6+(just to the right of x = 6). Thus the change in M that


we need is

[_ 4(x ; 6)3 + x +3 J\: -12 + 9 + 3+ 0 - 6 - 3 = -9

This result gives a change in M of - 9 from its + 9 value, and it ends at zero at
x = 9 m correctly. Alternatively, we could simply substitute into M(x) to get

(Continued)

321
I j"
~

(.
M=Oatx=9m:
4(9 ~ 6)3
M= ---+9+3=0
9

Or finally, if we are just interested in the change in M between x == 6 and 9 m, we


could integrate the function - V with definite integral limits :.

~M = - f
9 V dx = -
f9 [4(X
.
- 6)2
- I •dx
J
x=6 x=6 3

= - [
4(X -
9
6)3
- X
JI9 =_. -[12
6 - 9 -(0 -
" 611 = -9

as before.
It is also very important to know the value of 114 at a point such as F where M
is a local maximum. Maxima of moment will often correspond to locations of
highest stress in sections of the beam. At F, the shear is zero:

4(x - 6)2
V=--- 1 == 0
3

so that
(xF - 6)2 = 0.75

xF = 6.87m

Now that F has been located, we can find the bending moment there:
. -4(6.87 - 6)3
MF = ---9--- + 6.87 + 3 = 9.58N . m

which is less than the 12 N . m at x = 6 m, but which is still a local maximum.

120 Ib/ft ,. E., x p L

I 25001b-ft For the beam of Example 4.15, draw the shear and inoment diagrams.

6 ft ~
SOLUTION
We could, of course, plot the curves V = 120x'~ 720 and M = -60x2 +
720x + 340 from Example 4.15. However, we shall work this problem as though
we did not know these relationships, and then use them as a check.
120(6)
r--- ---,7201b
=
The free-body diagram will help us find the reahions at the wall. Note that
1 125001b-ft for "statics purposes" (Le., use of equilibrium equations for the whole beam) we
I
D may replace the distributed load by the "force-alone resultant" of 720 lb as
indicated at the left. The equations are:
(Continued)

322

"
LFy = 0 == - Vw - 720 => Vw = - 720 lb
v
'=i) LMA = 0 = 2500 - 720(3) - Mw => Mw = 3401b.ft
x
Thus the shear diagram starts at a value of.- 720 lb when x = O.The distributed
-7201b load q(xl is a constant (-120 lb/ft), hence the slope of V will be constant too
(V' = -q = 1201.Thus the value reached by the curve at the free end is V =
- 720 + 120(6) = 0, which checks because th~re is no shear force on the
cantilever beam at x = 6 ft. Note also that the ch~nge iIi V from x = 0 to 6 ft is
the negative of the area beneath the q(x) diagram-'-that is, - (- 12016 = + 720.
For the moment diagram, we begin with the wall. moment of + 340 Ib-ft.
M
As x increases,. the values of V become less and less negative. Thus the slopes of
M versus x become less and less positive. The value of M changes over the beam
25001b-ft by the negative of the. area beneath the V diagram, which is - [t(6)(- nOll =
2160 lb-ft. Therefore Mlx~6ft = 340 + 2160 = 2500 lb-ft, which checks the
applied moment at the free end.
3401b-ft Note that the V vs. x curve has the equation V = 120x - 720, which
checks the result in Example 4.15. The M vs. x curve is a parabola (it is second
order in x because it is the integral of a' straight line equation) with vertex at
x = 6. Thus M - 2500 = -k(x - 6)2, and the constant is evaluated using
M = 340 when x = 0:

340 - 2500 = -k(-612 => k = 60lb/ft

Thus

M = 2500 - 60(x - 6)2

=-60x2 + 720x + 3401b-ft


This result also agrees with_Example 4. 15.

E x A M p L E 4~22~
Draw the shear and moment diagrams for the beam loaded as shown in the figure.

301b 10 Ib/ft

401b-ft
1__ 4ft-=l~

(Continued)

323 ....
L..

._. ~_. ~_ .... .JiI,


'-~<'~'_--.,.----------------
...~.,'"".~--------
..
~--~, -~"'-=~,•.•
c~~
,•,],.,.."
.• "S'""'_." ".-----
,"",,"1'

---
••• ., _

SOLUTION

First we find the reactions. The uniformly distributed load is replaced by its force-
alone resultant, which has:

a. Magnitude = area beneath loading curve


= 110Ib/ftl(4 ftl = 40 lb
b. Location 2 ft to the left of D.

Thus we have the following free-body diagram and equilibrium equations:

30Ib 40lb

2 ft 2 ft 2ft 2 ft

40 Ib-ft
D

t+ "I.Fy = 0 = Ay + 30 +D -40 (I)

"+) "I.MA = 0 = (30 Ib)(2ft) + 401b-ft ...:.(40 Ibl(6 ftl + (8 ft)D


D= 17.51b

Thus from Equation Ill, Ay ~ -7.5 lb.

Here are the steps we have followed in constructing the V and M diagrams
on the next page:

I. Redraw the original loading diagram, being sure to return to the dis-
tributed load-i.e., do not use the force-alone res:ultant.

2. Draw the axes beneath one another.

.~
,

3. Start at V = + 7.5. (Recall that a downward force on a left-hand face is


positive!!
4. Since q = 0 between x = 0 and 2 ft, draw the line AB with dV I dx = 0
as shown.
(Continued)

324
,........ -
.-----

, x(ft)
I.
I'
l

x(ft)

30lb 5. The shear must decrease by 30 as we cross x = 2 ft to equilibrate the


30-lb concentrated force there. Thus put a dot at E, which has a V coordinat€ of
7.5 - 30 = -22.5Ib; see the figure at the left which shows a small element

t*l
7;,.5,lb. 1 ".5 Ib
at x = 2 ft.
6. Again draw a horizontal line, EC this time, because dV I dx = 0 in the
interval 2 < x < 4 ft.
7. Note that the couple has no effect on Vat C (zero resultant force!).
ltift 8. Because dVldx = -q, and because q(x) = -10 for 4 < x < 8 ft, the
slope of the V diagram between x = 4 and 8 ft is + 10 lb/ft. Thus it ends at V =
-22.5 + 10(4) = + 17.5 lb.
9. The check is that V really is + 17.5 lb at x = 8 ft because the reaction is
17.51b i, and an upward force on a right-hand face is positive shear.
10. Preparatory to discussing the moment diagram, let us find the location
of the point F where V is zero. This will correspond to a horizontal tangent in the
M diagram. By the similar triangles CDG and CFH, we have

22.5 10
--='""---
d I

so that

d == 2.25 ft

II. The moment M is zero at x = 0 (the pin cannot transmit a moment, and
none is externally applied directly to the beam).
12. dMldx = - V = -7.5Ib between x = 0 and 2 ft, so that M reaches a
value of - IS lb-ft at x = 2 ft.
13. Between x = 2 and 4 ft, the slope of M changes to the negative of the

(Continued)

325
new value of V, or Mi = + 22.5. The change in M from x = 2 ft to X = 4 ft is
the negative of the area beneath Vvs. x in that interval, or (22.5 Ib)(2ft) = 45 lb-ft.
Thus M at x = 4 ft is - IS + 45 = 30 Ib-ft.
14. At x = 4 ft, the moment diagram drops by the amount of the 40 lb-ft
couple; see the figure below of a small element at x = 4 ft.

401b-ft

Therefore, at x = 4 + ft, M is 30 - 40 = - 10 lb-ft.


IS. As we move from H to F (seethe V diagram), the values of Vare negative
numbers heading toward zero. The slopes of M vs. x, being equal tp - V, are thus
positive numbers heading toward zero. The change in M is

Area of V ) I
- ( between Hand F = -(2.25 ft)(22.5 lbl
2

= 25.3 lb-ft

ThusMatFis -10 + 25.3 = I5.3Ib-ft.


16. From F to D, V is now getting more and more positive, ending at
+ 17.5 lb. The slope of M thus gets more and more negative, and the change in
M is, this time,

Area of V ) _ .!.(4 _ 2.251 ft( 17.5 lb)


- ( between F and D = 2

= -I5.3Ib-ft

Thus M at x = 8 ft is 15.3 - 15.3 = 0, as it should be because no moment is


applied to the right and of the beam externally or by the roller.
We note that the maximum magnitude of morpent M is 30 lb-ft just to the
left of x = 4 ft; the maximum magnitude of shear force V is 22.5 lb throughout
the interval 2 < x < 4 ft.

In closing, we note that some prefer to define the shear force as


positive if downward on a right-hand face. This convention, although
not in agreement with definitions used iT:\the theory of elasticity, has
the advantage that the Equations (4.2) and (4.3) become V' = +q and
M' = + V This makes it easier to construct: loading, shear, and mo-
ment diagrams using the other curves, because one needn't keep up
with the minus signs. The only difference between the diagrams drawn
with V positive according to t 0~
and our convention ~ 0t
is that

326

I
4.8 Differential Relationships in a Beam 327

the shear diagram is "flipped," e.g.:

With these
signs:
M(~~DM
V _V
With these
signs:-
Mtf~PM V

1~
__ l_O_ft_~foO Ib
___

------
~ __
lO:ft
~.100 lb
_

l'
x x

j, x x
I
i.
I
Note V' =-q But now V' = +q
,
,- andM'=-V andM'=+V

Note that the moment diagram is unaffected by the change in the


sign of V

PR 0 B L EMS / Section 4.8

4.128 A distributed load in a beam varies linearly from 4.129


zero on each end to a maximum value of 8000 N/m in the 10 lb/ft

r=:~3,,13f,t
'center, as shown in Figure P4.l28. Draw the shear and
moment diagrams'for this'beame'or

. ~.OOO~/m Figure P4.129

4.130
ION/m lOON
1~2m~1--2m~f~~
FigureP4.128 3m :!.)
200N-m
In Problems 4.129-4.133 draw the shear and bending
moment diagrams for the beams loaded as shown. Figure P4.130
328 Chapter 4 Structural Applications

4.131 4.134 Draw the shear and moment diagram for the beam

J;:~-p L ~7i).C shown in Figure P4.134.

4.135 Draw the shear and bending moment diagrams for


the beam shown in Figure P4.135. Select an appropriate
Figure P4.131 free-body and write. the equations for shear and bending.
moment for 2 < x < 6 m.

4.132 * 4.136 In Figure P4.136:


a. Draw shear and mOment diagrams for the beam
shown when a = 6 ft. Indicate the values of all per-
SOON tinent ordinates.
b. Where should the roller be located (a = ?) in order
ISON. m that the greatest magnitude of bending moment in
the beam be as small as possible?
x

300lb/ft
Figure P4.132

-----7
x
4.133

~ rlO Ib 8ft

t
FigureP4.133
3h--'~-'r~ 3ft~
Figure P4.136

4.137 Draw the shear and bending moment diagrams for


the beam shown in Figure P4.137.

6kN
3kN/m

Figure P4.134

Figure P4.137

1200kN 600kN

300kN/m 4.138 Assuming the upward reaction of the "ground to be

A 600N/m 600N/m

l=2mL4m_~:J
W )x

Figure P4.135 Figure P4.138


4.8 Differential Relationships in a Beam 329

uniformly distributed, draw the shear and moment dia- 4.143


grams for the beam shown in Figure P4.138.

4.139 Draw the shear and moment diagrams for the beam
in Figure P4. 139.

Figure P4.143

* 4.144 The simply supported beam of length L in Fig-


ure P4.144 is loaded as indicated with a uniformly
distributed moment of intensity J1 per unit length. Draw
the shear and bending mom,ent diagrams. Hint: dM/dx =
Figure P4.139 - V does not apply since distributed moments wete not
included in its derivation.
Draw shear and moment diagrams for the beams shown in
4.145 ABEis a single contihuous member. Draw the shear
Problems 4.140-4.143.
and moment diagrams for the portion AB of the member
shown in Figure P4.145.
4.140
* 4.146 In Figure P4.146 the uniform rod has weight W
and rests on two props as shown, with b > a. Draw the
shear and bending moment diagrams and show that the

A
largest bending moment is at B if aJi > b.

.• L •. FigureP4.144 i:Yf38(~'8'~
Figure P4.140

F
4.141

2001b/ft
3501b

90 em
-t
l=-4 ft_~1~6
. 2 ft
ft-=\=- 5 ft-I
A C
40 em

+ )x
Figure P4.141 B
25N.m
4.142 -60em-
100N/m 90 em

ITITITY 2ooN. m Figure P4.145 .• 1.2m

£~~l~
.. , 2m --I m- -I

300N
m_.l

Figure P4.142 Figure P4.146


330 Chapter 4 Structural Applications

III/Cables

I'
I
I
~4~'9 Parabolic and Catenary Cables
::-.1
I
The flexible cable is yet another common method of supporting loads. For "
j

example, the suspension bridge has been used for many centuries and is
perhaps the best example of the engineering use of cables. We shall first
consider a cable suspended in a plane between the two points A and B, as
indicated in Figure 4.38. Let us investigate the shape of the curve in
which the cable will hang, under the action of a loading q(xl that is
everywhere vertical. In our analysis of cables, we shall take q(xl to be
positive downward.

Figure 4.38

8
T

-,.,>

Figure 4.39 Free-body diagram of cable element. t

We consider the equilibrium of a small element of such a cable, as


shown in Figure 4.39. Summing forces,
.~

LFx = 0 = (T + LlT) cosIO + LlOI - Teas e= 0

Dividing by Llx and taking the limit as Llx -+ 0, we see that


d(Tcos 0) = 0
dx
which implies that
Teas 0 = constant = Tn (4.41
4.9 Parabolic and Catenary Cables 331

Thus the horizontal component of cable tension does not change from
one end to the other. Continuing, if q(x) is constant, *

+t r.Fy = 0 = (T + L\T) sin(O + L\O) - T sin 0 - q L\x

Dividing again by L\x and letting L\x approach zero, we get this time

d(T sin 0)
=q (4.5)
dx

and substituting T = Ta/cos 0 from Equation (4.4),

d(tan OJ
Ta dx = q (4.6)

or because tan 0 = dy/dx = y',

y " =- q (4.7)
Ta
If we know the loading as a function of x, this equation may then be
integrated twice to yield the cable deflection, as shown in the following
example.

E x A M p L E
In the foregoing theory, let q(x) = constant = qo and let h (Figure 4.381 be zero;
that is, let the suspension p01nts lie on the same horizontal level. Compute the
deflection of the cable and the tension in it as functions of x. Also find the length
of the cable for a given sag H and span L. (See Figure 2.)

SOLUTION

\.
This problem is similar to that of a suspension bridge held up by many vertical
cables ~onnected to the main cable as shown in Figure 1. The more vertical cables

A B

Figure 1

(Continued)

*If q(xl is not constant, the resulting Equations (4.51-(4.71 are the same. The proof
uses the mean-value theorem as described in the footnote on page 315.
------L
y

Figure 2

there are, the closer we are to the distributed loading q(x) that was acting on the"
main cable in the preceding text.
The change to new coordinates shown in Figure 2 will allow us to take
advantage of symmetry when we evaluate our integration constants. Integrating
Equation (4.7),

q x 0, because y' = 0 at x = 0
y' = _0_ + C~ III
TH I'
And integrating again,

qox2 0, because y = 0 at x = 0
y=--+G~ (2)
2TH I'
Thus this cable loading results in a parabolic deflection shape.
To find the tension, Equations 14.4)and (4.51 give [with q = qo in Equa-
tion (4.511 .

T cos 0 = TH = constant (3)

0, because e = 0 at x = 0
T sin 0 == qox + 51J . (41

Squaring and adding,

T = .jT1 + q5x2 (5)

. The tension is seen to be minimum at the lowest point, where x = 0:

TM1N = TH (6)

and maximum at the support points A and B, where x = L/2:

TMAX = .jT1 + q5L2/4* (7)


If we wish, we can express TH in terms of the sag iH, at the lowest point of the
cable, by Equation (2):

q L2 q L2
H = --o =T H = --.o (81
8TH 8H
(Continued)

*Note that qoL is the total load supported by the cable.

332

).
p.
~
r-
1
!
I

Thus, alternatively,

and (9)

The required length of the cable for given endpoints and sag is also of
interest. It is

s = arclength
1= 2 1'~o ds = 2 1':0 .Jdx
2
+ dy2 = 2 r~20 + y'2
.Jl dx

Using Equations (I) and (8),


2
8HX)2
:I
1=2
fL'
-0

Using integral tables,


1 + ( 'IT
*
dx II 0)

,
r

(I II

Defining JJto be the ratio H/L (sag-to-span),

I = ~
2
[_1_
4JJ
sinh -1(4JJ) + .J 1 + (4JJf] II 2)

For example, ifJJ = H/L = 0.2, then sinh-I 0.8 ~ 0.733 and I = 1.1OL.
Thus the cable is about 10% longer than the span in this case.

Even if the Gablein the preceding,example is not supported with A


and B (Figure4.38) at the same level, the solution is still a parabolic curve
as long as the distributed load, viewed as a function of x, is uniform. As
just mentioned, this is the case for a suspension bridge. t But what if the
cable is an electric power transmission line, not supporting anything but
its own weight and perhaps that of adhered ice in a winter storm? In this
instance, the important difference is that q, while still a vertical loading,
is now force per unit of arclength s along the cable, and not of its
horizontal projection x. The cable now hangs in a different shape than a

* An approximate answer for I may be obtained by expanding the radical using the
binomial theorem, integrating each term, and truncating the series Iwhich converges for
18Hx/L21 < I,thatis,forly'l < II.
tProvided the suspended load far outweighs the cable itself.

333
334 Chapter 4 Structural Applications

parabola, though the two curves are close together if the maximum of y is
small in relation to the span 1. Let us, then, determine the shape of the
cable when q = q(sl.
If q = q(s), then applying 'LFx = 0 to the element of Figure 4.39
still yields

T cos 0 = constant = TH (4.8)

The y-equilibrium equation, however, changes:

+t 'LFy = 0 = (T + ~T) sin(O + ~Ol - Tsin 0 - q(s) ~s


so that

(T sin O)IXHx - T sin 0lx ~s


----~-x---- = q(s)-~x (4.91

Taking the limit, *


d(Tsin 01 _ (J ds
dx - q S dx (4.10)

= q(s)-J1 + y'2 dx
dx
= q(s)J1 + y'2 (4. II)
Substituting for T from Equation (4.8) as before,

d(tan OJ () / 2
TH dx = qs V 1 + y' (4.12)

or

(4.13)

Therefore, we find that we must solve a different differential equa-


tion this time, which is ,

y" = !LJ1 + y'2 (4.14)


TH
The solution to (4.14) is usually accomplished via a change of vari-
able. We let y' be renamed
y' = p (4.15)

so that
y" = p' (4.16)

*See the footnote on page 315.


4.9 Parabolic and Catenary Cables 335

Then Equation (4.14) becomes

(4.17)

or-

(4.18)

Integrating (with integral tables), for the case when q = qo = constant, *

In(p + vI/ + p2) = -x


qo +. K 1 (4.19)
TH

from which

____ (;0 X+K,)


.J 1 + p2 = e H - P (4.20)

Squaring,

2QOX 2K) (QOX) K


1 +J!1 = e ( TH + I - 2pe TH+ , +]iL (4.21)

Replacingp by y' and solving for this quantity,

.. '

,
) .
(QOX +K,) _(QOX +Kt)
e TH _ e. TH
y' = ----_ ...._----- (4.22)
2

the right-hand. side of which ,can be. recognized as. the hyperbolic sine.
Therefore,

y , = sm. h(qOx
T + K1
) . (4.23)
H

Integrating one last time,

(4.24)

*This would be the case if the cable were both uniform prior to suspension and also
practically inextensible.
336 Chapter 4 Structural Applications

If we now again choose the origin to be the lowest point of thecurv-e, then
y = 0 and y' = 0 at x = 0, so that

TH
(1 ) o == -cosh K1 +K2 (4.25)
qo

and !, -

(2) o = sinh K1 (4.26)

Equation (4.26) gives K1 = 0, after which itfollows from Equation (4.25)


that K2 = - THI qo. Therefore,

y =
TH (
qo cosh qox
T -
H
1)* (4.27)
. ~

Nondimensionalizing the deflection, we obtain

(4.28) I
:1: ,
I
I
':
Note that because

cosh (qox)
T = 1 (4.29)
H

we have, if qoLITH ••« 1 (where L is the span),

qoY ~
TH
(1 + q~X2) _ 1
2TiI

or

(4.30)

which was the solution to Example 4.23!

*The shape represented by this answer is often called the "catenary curve."

i.
...'.
r
I

E x A M p L E

The cable holds the balloon in equilibrium with a tension at the ground
attachment of 100 lb, If the cable weighs 0.25 lb/ft and is ISO ft long, what is the
;.,".
height H of the balloon?

SOLUTION
This is a "catenary cable" because it is loaded by its weight-that is, q =
q(s) = qo. Therefore we begin with the general solution for this class of cables,
which is Equation (4.241:

I (1) .

I
I
Using the coordinates shown in the figure, we have y = 0 at x = 0, so that

TH
0= -coshK, + K2 (2)
qo
Because qo = 0.25Ib/ft, and TH = constant == lUO cos 55° = 57.4lb, Equa-
tion (2)becomes

K2 = - 230 cosh K, (3)

Also, y' = tan (J = tan 55° at x = 0, so that

tan 55° = [sinh(qOx + K,)J\ (4)


TH x=O .

1.43 = sinh K,

Therefore, after evaluating the inverse hyperbolic sine,

K,=1.16

Thus from Equation (3),

K2 = -403

(Continued)

337
And so we get

y = 230COSh(~
. 230
+ 1.16) - 403

The length lof the cable is given by

(150 (150
1= 150= JFO ds = Js=o.JdX2 + dy2 = fXB .Jl + y'2 dx
x.=::Q

where XB is the x coordinate of the attachment lat Bl to the balloon. Because

y' = sinh (~ + l.i 6)


230

and because 1 + sinh28 = coshi 8, the integral is .'

150 = f. XB COSh('~ + 1.16) dx = 230 Sinh(~ + 1.16)./X B


>:=0 230 230. O

or

0.652 = sinh( xB + 116) - 1.44


230

or

sinh-12.09 = X
B
+ 1.16
230

Thus

1.48 = xB + 1.16
230

or

xB = 73.6 ft

Therefore, the height of the balloon is

H = yl. x=73.6
. = 230 COSh(73.6'+1~
230
16)' --' 403

H = 531 - 403 = 128 ft

338
4.9 Parabolic and Catenary Cables 339

P R OBL EMS I Section 4.9

4.147 The cable in Figure P4.147 is to support a uniform 4.149 A clothesline is deflected in an ice storm as sug-
load of 50 lb/ft between P and Q, with the lowest point B gested by Figure P4.149. A~sume that the ice load varies
being lOft below the level of Q. Find the horizontal cosinusoidally with x, with'intensity qo at x = O. Neglect
location of B and then the maximum cable tension. the weight of the clothesline and determine its deflection
curve.

-----80 ft-- . . .~ 4.150 For the cable in Figure P4.150 carrying a uniform
load q(xl = qo, prove that at the supports, the tensions are

r
I
f
and

Figure P4.147

4.148 Two cables of the suspension bridge shown in Fig-


ure P4.148 symmetrically support a total load of 80 x 4.151 Obtain the result T! = .JT~ + q5x2 (from Exam-
106 lb. The sag in the cables at mid-span is 500 ft. Find the ple 4.23) by using the free-body diagram shown in Fig-
mid-span tension and the angle e the cables make with the ure P4.151.
tower. Neglect the cable weight. 4.152 Show that summing moments about the left end of
the element of Figure 4.39 adds nothing to the results of
the analysis.
18" * 4.153 Note that if the ends of the cable are not at the same

Figure P4.148

FigureP4.150

qo = maximum intensity TH
Figure P4~149 FigureP4.151
340 Chapter 4 Structural Applications

level, as shown in Figure P4.153, then we don't know the Integrate term by term. ForHIL = 0.2 compare flL from
location of the lowest point ahead of time, and cannot take the example with this series result for: lal one term; (b)two
advantage of symmetry. For the parabolic cable, begin with terms; lc) three terms.

qox2 4.157 Given a cable with maximum allowable tension


y = _.- + C1 X + C2 To, length t, and weight per unit length qo, find the
2TH
maximum horizontal span that can exist for this cable
and solve the following problem: The pipeline weighs without exceeding To.
700 lb/ft. It is to be assumed that over the gorge it is
supported entirely by cables (i.e., neglect any forces from 4.158 In the preceding problem, find the maximum span
the rest of the pipe entering the earth at P and Q). Find the if To = qof.
required length of cable if the lowest point of the parabola * 4.159 In Figure P4.159 determine the vertical distance
has an x coordinate of 150 ft. Also find the tensions at the from B (the center of the pulley) to A.
cable suspension points A and B.
4.160 If a rope 22 ft long is to be suspended from a tree
4.154 In the preceding problem, find the length of cable
limb to make a swing, and'the attachment points are 3 ft
from the lowest point to the support at B. apart, what is the sag distance H? Assume that 1 ft is used
* 4.155 A light cable shown in Figure P4. 155 carries a load on each side for the knotting. Compare the answer with
that grows with x according to q = kxn, where k is a the straight line result of 9.89 ft shown in Figure P4.160.
constant and n is a positive integer. If the cable crosses the
x axis at x = L/2, find the deflection y(x). 1.5ft 1.5ft

4.156 Expand the integrand of Equation (10) of Exam-


ple 4.23 into the following power series:

1 .(8HX)2 = 1 ~ (8HX)2 _ ~ (8HX)4 + ...


+ L2 + 2 L2 8 L2 (-)

200ft==J _t
.. Figure P4.160

50 ft
B _t

q = constant = 700Ib/ft
Figure P4.153

Figure P4.159

FigureP4.155
4.10 Cables Under Concentrated Loads 341

* 4.161 Find the error incurred in using the parabolic


solution [Equation (4.30l] to get the center sag of a
symmetrical catenary cable if the sag -;- span is: la) /0; (bli;
(cll.

4.162 A cable of length 80 m hangs between the tops of'


two identical towers that are 66 m apart. Find the maxi-
mum sag in the cable.

4.163 A cable of length 80 m hangs between the tops of


. two towers. If the maximum sag is 5 m and the maximum
tension is 480 N, what is the weight per unit length of
the cable?

4.164 Repeat Problem 4.147 if there is no suspended load,


but rather the cable itself weighs 50 lb/ft.

4.165 In Example 4.24, if the diameter of the base (where


the cable is attached) is 8 ft, and the wind is horizontal, .
find:
a. The difference between the vertical force of the air
and the weight
b. The resultant horizontal force caused by the wind
and the height of its line of action above B.

* 4.166 In Figure P4. 166 the balloon of Example 4.24 has


come loose from the ground and has begun very slowly

Figure P4.166 '"

moving horizontally across an open field. The balloonist


lets out some air, and the balloon lowers to a height of 30 ft
and begins to drag its cable and its spike. If the net
(horizontal) wind force is 50 lb, find the length of cable on
the ground and the maximum cablii' tension.

* 4.167 The cable in Figure P4.167 has a weight per unit


length of qo. A length L hangs over the pulley as shown.
Find the maximum tension in the cable, the total length of
the cable, and the center sag.

~4.10 Cables Under Concentrated Loads


Sometimes we are faced with the necessity of analyzing a cable of neg-
ligible weight from which a number of forces (WI W2, ... , Wnl are
I

suspended at a series of load points PI; P2, ... , Pn. (See Figure 4.40.)
342 Chapter 4 Structural Applicationsd

Ln + 1 B If':' 1
H

j
x

WI

Figure 4.40

Suppose we are given the lengths of the segments (L1, L2, ... , Ln+1 in
Figure 4.40) and coordinates (D, H) of the right end with respeCt to
the left. Then if we wish to know the shape in which the cable will
hang (given by (}1, (}2, .... , (}n + 1) and the tensions in the segments
(T1, T2, ••• , Tn+ 1)' we have an extremely'difficult set of equations to
solve, consisting of 2n equilibrium equations, two for each load point,
like (see Figure 4.411
'LFx = 0 = -T1 cos (}1 + T2 cos (}i (4.31)
'LFy = 0 = T1 sin (}1 - T2 sin (}2 - WI (4.32)
WI
Figure 4.41
plus two geometric equations relating the L/s and B/s to D and H:
n+l n+ {
D = L
i= 1
LicoSOi H = -
i= 1
"
i..J L.I sin B..1 (4.33)

These 2n +.2 equations are very hard.ito s~lve because of the non-
algebraic, trigonometric functions of half the unknowns (the (}/s),even
for just two segments!
Another approach to the problem is to seek the tensions and "sags"
when we know the x coordinates of the n load attachment points PI'
P2, ... , Pn, as suggested by Figure 4.42 on the next page.
First we note that the horizontal components of all the tensions in
the n + 1 segments are the same. From Equation (4.31) we have
T1 cos (}1 = T2 COS(}2
/
Because this can be repeated for each load point, we have
T1 cos (}1 = T2 COS(}2 = ... = 1;,+ 1 cos Bn+ 1 = TH (4.34)
4.10 Cables Under Concentrated Loads 343

H= -5n+ I

Po

x
)

52

I~
y

~
---D2---D3---l~D4--D5-...
I
DI Dn+1
Figure 4.42
; .;,

which is the same result we obtained in Section 4.9 for continuously


loaded cables. We can use this "common horizontal component of
tension" result to write the vertical components of each tension in terms
of the unknown sags 81,82' ... , 8n+1. Consider segment 2 in the dia~
gram (from PI to P21 and the force that its tension exerts on PI:

P'~ P'~,_"

\ .. P2
T2, = vertical component of tension

These triangles are similar, so that

T2~' 82 -.81 . .(82 - 81)" ,


- = --- ~ T2 = TH ---
TH D2 v D2

, and in general*

T = TH(_8i~-_8i_-_l) (4.35)
'v D;

Thus if we know TH and the n sags 81, 82' ... , 8n, then the problem
is solved because the tensions may be found as follows:

(4.36)

*Note that 5j - 5j-1 < 0 for the segments with negative slopes.
344 Chapter 4 Structural Applications

It is clear that the largest tension will occur in either the first or last
segment.
Note now that in using Equation (4.34) we automatically satisfy
'L.Fx = a at all n load points. If we now set the summation of forces
in the y directionat.eachJoad ..point..equal to zero, n equations will be".~..~..
T2 generated. But all except the first and last will involve three ofthe sags.
For example, at P2' we have

Using Equation (4.35),

or

which involves 81' 82' and 83 as well as TH. if instead of summing forces
T2 we sum moments on the following series of free-body diagrams, * only two
sags at most are involved in each equation. For one full segment (see the
W, figure at the left),

or

or

For two full segments (see the free-body diagram),

or

*Note that the reactions at A and B necessarlIy equal the tensions T1 and 1;,+ I and act
along the lines of the first and last segments. .
4.10 Cables Under Concentrated Loads 345

or

This procedure will'give usn equationsras 'we sum moments on a-~' --~.,_.
series of free-body diagrams that involve successive cuts of segments
2, 3, ... , n + 1. Our problem, however, is that we have n + 1 un-
knowns (SI' S2' ..• , Sn' and THl, and until we specify one more condition
on the geometry, there is no unique solutionj that is, there are lots of
cables that could satisfy the conditions. For ex~mple, each of the cables in
the diagram has the same set values of DlJ D2, and WI' yet clearly the
sought-after sags and tensions are different for each. If for our (n + lIst
equation we specify the overall length of the cable,
lO"
L = .jDi + si + .JD~ + (S2 - SI)2 + ... + .JD~+1 + (H - snl2
then we are back to formidable analytical difficulties. A better condition
is to specify one of the sags. Then we can use the moment equations
above and solve for the other sags in terms of TH, one at a time, never
having over one unknown sag per equation. Another more realistic
possibility for the (n + 11st equation is to be given a maximum allowable
tension. This tension will occur in the first or last segment, as we have
noted. We now give examples of these ideas.

E x A M p L E
The endpoints of a cable are tied to the points A and B in the figure. The cable
carries the indicated loads at x = 2 ft, 4 ft, and 6 it'. The maximum tension in the
cable is 1000 lb. Find the tensions in all four segments of the cable, and the sags of
the load points PI' P2, and P3 •

2001b

(Continued)
or

Nb
f = 2h

To check our assumption and see whether slipping has already occurred, we
t" must find out iff<JlN.Ifso,our tipping assumption was correct: ","""

or b < 2Jlh for tipping, as we had previously obt~ined. If b = 2ph, the crate slips
and tips together, and if b > 2ph, the assumption was incorrect and it slides first,
again as previously determined.

J.tl = 0.4 c -F
E x A M p L E .
J.t2=0.3~ The masses of blocks e,
B, and A are 30 kg, 5 kg, and 15 kg, respectively. Find
the largest value of the force F for which no sliding will take place between
J.t3 = 0.25 any pair of surfaces. Assume the blocks to be wide enough so that tipping will
not occur.

SO L UTI 0 N
There are three cases to consider:
1. Impending sliding of C on B, with A and B stationary:
F
'£Fy = 0 = NI - 30(9.811 => NI = 294 N

It = flMAX = JlINI = 0.4(2941 = 118 N

...:t...'£F" =0 = F - fl => F = 118 N

2. Impending motion of Cand IJ together as one body, with impending


30(9.Bl)N
sliding of IJon a stationary block A....

EtJ=:::'
E!3-5(9.BI)N
+t '£Fy = 0 = N2

f2 =
-

f2MAX
35(9.81)

= Jl2N2
=> N2 = 343 N

= 0.3(343) = 103 N

2 ...:t... '£F" = 0 = F - f2 => F = 103 N


1 • 1-.
N2 which is less than the F value to move C alone.

. (Continued)
SOLUTION
We shall work to four significant digits in this example, We write the moment !t-
equations described in the foregoing discussion:

~ ~MA = T s H l - [T (S2 ~
H SI)J2 - 100(21 = 0

.~ 200(4)
sl(4 + 2) - S22 = --
TH

W, = 100
(II
T2., = TH(52 ~ 51)
Free-body diagram of segment 1 and
part of 2.

;'jI'

T,
,
"
i~'

'!;.
;.:\

j
Free~body diagram of segments ~,,
I, 2,'and part of 3. I"~!
[': ~;I
f
,,
1400 ;.~
s2(6 + 6) - s3(61 = -- (6) ~\ -~

TH ,
~'~

(2) ,.'.I'>
8400
1252 - 6s3 = -- ~.. -
TH
.
~f

~ ~MA = Ts H 3 - [T H ( - 3 5- S3 ) ] 12 - 200 - 1200 - 3600 = 0


,. <,

I<
5000 t
17s3 + 12(3) = -- (5) (3) 1
TH 1
"';

B
T = T (53 - 54)
4. H 5 "---
54 = H = 3 ft
4 ft 6 ft

free-body diagram of segments I,


W2 = 200 2, and 3, and part of 4.

(Continued)

346
We now have four equations Ill), (2), (3), and (4.36)] in four unknowns. First we
find the sags in terms of TH:
From (3),

1471
S3 = -- - 2.118 (4)
TH ,

From (2),
,------"'-'---,
!..
+ (_14_7_1_ 2.118)
_70_0_.0
.,I
S2 =
I TH 2 TH
! .
or

,"
I
!
S2 = --
1436
TH
- 1.059 (5)

From (II,
~
81
133.3
= ~~ + -I (1436
-- - 1.059 )
,TH 3 TH
(6)
612.0
SI = ~~
T - 0.3530
.
H

We now assume (and must later check) that the maximum tension is in
segment API' (The other possibility is P3B.) Then TI = 1000 lb, and Equa-
tion (4.36) gives

1000 = THJI +t;y (7)

or

(81

Equating the two expressions (6) and (8) for SI and squaring,

4 x 106 _ _374,500 _ 432.1 6


- '..;'f.I'I"'
--2-'-' 4 - 2 + 0.124
TH TH TH

Rearranging, 4.125T1 - 432.1 TH - 3,626,000. Solving with the quadratic


formula gives

TH = 991Alb

Back-substituting,

SI = 0.2643 ft

S2 = 0.3895 ft

S3 = -0.6342 ft
(meaning the third load point is above the x axis through A)

(Continued)
The other tensions are given by Equation (4.361:

= 991.9 Ib

= 1006lb

T4 = TH
J
. ( S4 - S3)2
v::-
1 + (S4 = - H = - 3 ft)

1097lb

We therefore see that we have guessed wrong and that the maximum tension is
instead in the right-most cable. Rewriting Equation (7) (the three moment
equations are still valid):

(S4 = -3 ft)

Equating expressions for S3 gives "j ~

06 1471
5 -2 - 1 = --" + 0.8820
~ TH TH

Squaring,

-25(~ _) 2,164,000 2595


T~ 1 = T~ + T "+0.7779
H

1.031T~ + 103.8TH - 913,400 = 0

TH = 892.2Ib
so that, using Equations (61,(5), and (41in order,

s) = 0.3329 ft

S2 = 0.5505 ft

S3 = -0.4693ft

(Continued)

348
and

TI = TH I + (-kY = 904.5 lb

SI)2
J.
T2 = TH
J (
I + ~
S2 -
= 893.5 lb

T3 = TH
J (
I
S3 - S2)2
+.~ = 905.01b

This time all the tensions are :::;1000 lb, and we have also solved for the sags.

:
To illustrate the procedure when a sag is specified instead of a
maximum tension, we shall specify that 82 = 0.5505 ft, which came out
of the preceding solution, and solve for all the tensions in the following
example.

E x A M p L E
If the sag S2 of load point P2 is 0.5505 ft, find the tensions in the four cable
segments and the sags of PI and P3 .

2001b

SOLUTION
On the overall free-body diagram,

4ft

3001b
IOOlb
(Continued) 2001b

~Aft.
1
I
t,

'+) 'LMB = a = (300 Ib)(5 h) + (200 lb)(II h)

+ 1I00 Ib)(15 h) - TH(3 h) - TJ)17 h)

or
3TH + 17Tjv = 5200

And using the free-body diagram of APIP2'

2ft 4ft

A \\
P,
0.5505 ft _1
7-
3,

1001b
200lb

'+) 'LMp2 = a = (loa IbH4 hi + TH(0.5505 hI - TJ)6 h)

or

0.5505TH - 6Tjv = -400

Solving,

TH = 891.9 lb} T j = 904.21b


T1v = 148.51b

From the same free-body, remembering that T3x = TH = 891.9Ib,


'+) 'LMA = a = (891.9Ib)(0.5505 h) + T3)6 ft) - (100 Ib)(2 ft)

-(200 Ib)(6ft)

T3v = 151.5lb

T3 = ~891.92 + 151.52 = 904.71b


148.Slb Then we use other free-body diagrams to get the unknown sags:

'+) 'LMpt = 0 = (891.91bHsd - (148.5Ib)(2 ft)

Sj = 0.3330 ft

For T2, note that its horizontal component is TH = 891.9 lb and

'LFy = a = 148.5 - 100 - T2v

T2v = 48.5 lb

Thus

T2 = ~891.92 + 48.52 = 893lb


(Continued)

350
Then, using the free-body diagram below,

"t) LMp3 == 0 = (l00)b)(10ft) + (200lb)(6ftl + (891.91b)(s31

-(148.5lb)(12 h)

, 83 = -0.4687ft (as before)

.:
200lb
,.
Also, T4v can be computed from

+t. LFy = 0 = 148.5 - 100 - 200 - 300 + T4v

.
"
T4v = 451.5 lb
I __ ~! so that

differing from 1000 Ib due to rounding to four digits.

Partial checks of the two solutions obtained in Example 4.25 are afforded
by writing the overall vertical equations of equilibrium:
0.2643 )
'LFy = -600 + 1000( .J ....
0.2643 2
+ 2 2

. (2.366),
+ 1097 .
. .)2.3662 + 52 '

-:-'600 + 131'.0 "+469.2 r=0.200 lb ./

'.
352 Chapter 4 Structural Applications

and
0.3329 )
LFy = - 600 + 904.5 ( -/-;=:==.:=.
===
y 0.3329 + 2
2 2

+ 1000( 2.531 )
.J2.5312 + 52
= -600 + 148.5 + 451.6 = 0.100 lb ./
10001b

3001b

2001b

PR 0 BL EMS / Section 4.10

4.168 The cable supports two vertical loads at PI and P2 as 4.170 In Figure P4.170:
shown in Figure P4.168. Find the elevation of the load
a. If the horizontal force P is holding the cable in
point PI with respect to the horizontal line through A, and
equilibrium, what is the value of P?
the tensions in the three segments (API' PIP2, and P2B) of
b. Find the distances dl and dz.
the cable.

4.169 In Figure P4. 169 find the sags of PI and Pz, and the
tensions in the three cable segments, if the maximum
tension is known to be 1400 lb.

B -_. -t
Sm
I. t,
t 6m
Figure P4.170

_t p
4m
3kN rk
lOON

IOkN
1----- d2---~.I
Figure P4.168

SOOlb Figure P4.169


9001b
4.10 Cables Under Concentrated Loads 353

4.171 The beam weighs 200 lb and is to be supported in 4.173 Repeat the preceding problem if the 0.6-m dimen-
a horizontal position by two cables as shown in Fig- sion is deleted and replaced by the condition that the
ure P4.171. Find: maximum cable tension is to be 600 N.
a. The angles 4>1 and 4>2 4.174 Find the angles 8 and 4> and the tensions in the two
b. The tensions in segments API and P2B
segments of the cable loadediby the 200-lb force shown in
c. The elevation H of B. Figure P4.174(a). Check your answers for 8 and 4> by
4.172 The light cable is loaded by the two downward making the construction suggested in Figure P4.1 74(b) and
forces as indicated in Figure P4.172. Find: measuring the angles with a protractor.

a. The elevation of the load application point PI with


--------10 ft-~------
respect to A.
b. The tensions in the three cable segments.

~I~
B

<1>2\ t
38 ft ._t
2001b
Figure P4.174(a,

I PI 15 ft P2
i Figure P4.171

I-Im-----1.2m----0.8m-
A -.
\\i~ 0.4m

Figure P4.172
-'
Figure P4.174(b)

250 N

COMPUTER PRO BILlE M S / Chapter 4

4.175 The vertical component by of the deflection of joint 8 and watch for the minimum, or else use calculus and then
Q caused by the force P acting on the simple plane truss in solve a resulting cubic polynomial for the cosine -of8.
Figure P4.175 is given by the equation
4.176 Solve Problem 4.161 using a computer.

PL
by = AE'
(I
sin2B cos 8
I)
+ tan28 --- L---
P

Q
where p, L, and 8 are shown in the figure, A is the area of
members BQ and CQ, and E is a constant. (Obtaining this Figure P4.175
result is beyond the scope of this book; it requires a course
in strength of materials or deformable bodies.) Use the
computer to find the angle 8 for which by is a minimum.
You may either list values of the parenthesized function of
') .
354 Chapter 4 Structural Applications ~ .
~
.

.~.

Answers to Questions I Chapter 4


Q4.1 Attic, gable end, floor truss.
Q4.2 Pin C will at this stage have two unknown forcesacting on it; pin E has four.
Q4.3 Because we found it to be compressive when we examined pin B. If we were'to
now draw it tensile on pin C, we would be in violation .of the action-reaction
principle!
Q4.4 Because unless FeE is tensile, LFy cannot vanish for pin C.
Q4.5 If both FDA and FDE were to turn out positive as indicated, then: (a)the first sketch
leaves LFy =1=0; (b)the second sketch leaves LF" =1=O.
Q4.6 The force exerted by the inside of the hole (in member EBI onto the pin comes
from the right side of the hole; that is, it pushes to the left instead of pulling to the
left. Nonetheless, because forces may be translated along their lines of action, the
free-body diagram of the pin is often drawn as in the second sketch.
Q4.7 For other external loads, the member may carry a force. Also, the member really
does carry load due to the (neglected) weights of the members. It also adds stability
to the structure.
Q4.8 First, FBE has to be tensile so that LFy can be zero. Then FFE has to be compressive
in order that LF" can vanish.
Q4.9 The last vertex drawn has but two members temiinating at it.
Q4.10 Depicted as external forces on any legitimate free-body diagram thus produced
would be a cancelling pair of forces associated with each cut member. Only the
whole truss can be used as a free-body unless a part of it is completely cut away.
Q4.11 Use a vertical cut through BF, FL, and LK. LML = 0 now gives FEFJ and the 3-kN
force is not used.
Q4.12 No to all five questions.
Q4.13 Because the distance between the pair of forces P goes to zero at those two points.
Q4.14 Because on any free-body diagram such as Figure I, N is the only force in the x
direction so that LF" = 0 = N.
Q4.15 By 1:MD = 0, we get the same answer for Ay•
Q4.16. The force-alone .resultant makes the same~col1tribution to the equilibrium
equations, hence giving the correct external reactions. But it does not create the
same internal force distributions within the beam!
Q4.17 Whenever q = 0, V will have a zero slope; whenever V == 0, M will have a zero
slope. Thus if the curves are drawn beneath one another, these locations can be
projected down to advantage.
Q4.18 Yes. The larger the values of loading qo and length L, the larger the deflection.
And the larger the tension, the smaller the deflection. The reverses also make
sense, with each producing the opposite effect on deflection.
Q4.19 Equation (4.27) is based on the condition y = y' = 0 at x = O.
Q4.20 No! The parabolic curve comes from a load uniformly distributed with respect to
x; the catenary from a load uniformly distributed with respect to arclength s. In

- ~'- ~
.. ... "
' ~'-
Review Questions 355

fact, transmission cables have small deflections but hang in the catenary shape,
while suspension bridge cables have large deflect'ions with q(xl == qo.
Q4.21 Yes. The difference between cables and chains is,that there is a small bending
stiffness in a cable, and none in a chain; but because this stiffness was neglected,
" the results apply to chains. Of course, the lengths of (near-rigid) links must bevery .•--"".,,,•...
small compared to the span of the chain.

Review Questions I Chapter 4


True or False?

1 Any truss can be solved (i.e., the forces in aU its members can be found) by enough
applications of the method of joints .
.2 In a simple plane truss, a joint can always be found with only two unknown
member forces acting on it.
3 For the method of sections to be helpful, the truss has to be sectioned into two
distinct, unconnected parts.
4 It is always possible to cut a section that allows the force in any truss member to
be found by a single moment equation.
S No cross section of a two-force member ever has to resist a bending moment or a
shear force.
6 No member of a truss ever has to resist a bending moment or a shear force.
7 If a truss member pulls on a pin, then it is in tension; if it pushes on apin, it is in
compression.
8 If a truss member pulls on a pin at one of its ends, then it must push on the pin at
its other end.
9 The rate of change of bending moment in a beam with respect to x equals the axial
force.
10 The area beneath the loading curve between two points PI and P2 of a beam
equals the negative of the change in the shear force between PI and P2 .
11 If shear and moment diagrams are sketched and correct values at the right end of
the beam are ~btained, then the diagrams have probably been drawn correctly:
12 If a cable load is uniformly distributed with respect to x, then the shape of the
cable is parabolic. If the load is uniformly distributed with respect to arclength s,
the shape of the cable is a catenary.
13 For a cable under any vertical loading, the horizontal component of tension is
constant across the cable.

Answers: 1. F 2. T 3. T 4. F 5.F 6. T 7. T 8. F 9. F 10. T 11. T 12. T 13. T

.:;

L.
.:

Friction

5.1 Laws, Coefficients, and Basic Applications of Coulomb Friction

5.2 Special Applications of Coulomb Friction


• Wedges
Flexible Flat Belts and V-Belts
• Screws
• Disk Friction

356
5.1 Laws, Coefficients, and Basic Applicatiohs of Coulomb Friction 357

5.1 laws, Coefficients, and Basic Applications of


CouRomb Friction
The world would continue to go round and round without friction, but
the lives of just about.everythiJ;lg.liYing on it ,would change drastically if. ,_,
friction were suddenly to disappear. Friction is therefore a very important
topic, worthy of much study. It serves both good and bad purposes'.
Without friction, automobiles, trains, and bic;ycles would not work at all
because their wheels need friction to produce acceleration of the vehicles
to which they are attached. And without. friction on the soles of your
shoes, you could not walk out of the room in which you are sitting or
lying. We wish to maximize the friction force in situations where it is
being used to our advantage, such as -in the use of tires, brakes, wedges,
screws, belts, clutches, and in simple but important applications such as
the foot of a ladder. Conversely, friction has many deleterious effects:
it causes mechanical parts to wear out, and energy is expended in over-
coming frictional resistance to motions of machines and vehicles.
We shall noW examine the law of dry friction between a pair of
surfaces; this law was first set forth by Coulomb in the 18th century and
is called Coulomb's Law of Dry Friction.
For two bodies 151 and 152 in contact, the friction force on, say, 1511
is the component of the resultant force (exerted on 151 by 152) that lies in
the tangent plane of contact (see Figure 5.1). The other component,
perpendicular to this tangent plane, is called the normal force exerted on
151 by 152,
,
! ..

. n \Olce)
f ~hict\o
1\Olce)
tl ~noltna

Figure 5.1

Let us now imagine a crate resting on a horizontal surface. The free-


body diagram is shown in Figure 5.2 (a). The resultant of all the "pressures
times areas" at each of the points of contact beneath the box is depicted as
a single resultant force N, which equals the weight mg of the crate and
which acts upward through the mass cente~ C. In this state, there is no
friction at all. Friction resists the tendency to slide and if there is no such
tendency, it will not be present..
Now suppose a child exerts a slowly and linearly increasing (with
time t) horizontal force P in an effort to move the crate to the right as
358 Chapter 5 Friction

f
-- 0
N
Figure 5.2

shown in Figure 5.2(bj. Two things of importance to our study then


happen at once. They are:
1. A friction force I develops simultaneously that opposes the tendency
of the crate to slide t? th~ right. ----J.
2. The normal force N ISstIll equal to rug because there is no vertIcal.
acceleration and the y forces are still in balance. But the position of
N-that is, of the resultant of the many pressing forces under the
crate-shifts to the right. The way this happens is by tne pressure
increasing on the right and decreasing on the left. The reason it
happens is that N must act so as to overcome the tendency of the
crate to tip forward. Said another way, the moments of I and N about
A * must add to zero to prevent rotation. Thus N moves to the right " ",
so that the moment IH ~ can be balanced by the moment N~ D ,
~,

f
where ~ is (see the figure) the shift, to the right, of the force N from
,
;1
':',
the centerline of the crate. %.

As the applied force of the child grows, so do I and fJ. If the crate is
wide enough and/or short enough, it will not tip before sliding. Let us
assume this to be the case for now. The friction force I will continue to
balance the force P for as long as it is able.
At some point, however, a maximum value is reached for I,. as
Figure 5.3 suggested by Figure 5.3, and the crate will start to slide. This value was
observed by Coulomb to be proportional to the normal force:

(5.1)
in which the proportionality constant fls ii) the coefficient of static
friction, which depends upon the types of materials in contact and upon
the roughnesses of their surfaces. The value of fls varies from near zero for
very smooth surfaces to 1 or even more than 1 for very rough surfaces. ,.
Now let us return to the child and the crate. As most of us have
experienced, it is harder to "get something moving than to keep it

*Note that Pand mg produce no moment about A.


5.1 Laws, Coefficients, and Basic Applications of Coulomb Friction 359

moving." When P reaches a value equal to the static maximum of I,


motion is said to be impending. Any further increase in P will cause the
crate to slide, and the friction force normally drops off to a second, usually
slightly lower value.

This reduced value of I is assumed to be the normal force N times a


different coefficient of friction, this time called the coefficient of kinetic
friction, J1.k:
(5.2)

where h, is the value of friction to be used if sliding has started and is


actually ongoing. The force A is thus used in dynamics. There, if no
subscript is placed on the J1., it is to be assumed that J1.s ~ J1.k' But actu-
ally, J1.k is usually dependent upon the speed of sliding, and, as the
above discussion suggests, it is less than J1.s' A simple model that is often
used is to take J1.k to be a constant (independent of sliding speed) less
than J1.s'
In statics, however, we are usually only concerned with J1.sJ so that
we may drop the subscript on J1. in most cases. The student should always
remember that
o :::;I :::;IMAX = J1.N
and that in the absence of relative motion, f = J1.N only if the bodies in
contact are at the point (on the verge) of slipping relative to each other.
This is called impending slipping. When surfaces are called smooth,"
II II II

it means J1. is very small, approaching zero. In this case,


O:::;f:::;J1.N~O

so that no friction is available,andJ-is to be.assumed as zero.


The static coefficient of friction may be determined for two mate-
rials by constructing a plane of one of the materials and a block of the
other (see Figure 5.4l:
As the angle fJ of the plane is increased:from zero, the friction force
beneath B1 will increase (see the free-body diilgram of B1) because it has to
Figure 5.4
equilibrate an increasing component of the'weight (mg) down the plane.
Friction always acts on each body in the direction that opposes sliding of
the two contacting bodies past each other.
Using the indicated directions for x andy, the equilibrium equations
of B1 are seen to be
y 'LFx = 0 = mg sin fJ - f => f = mg sin fJ .

\' 'LFy = 0 = mg cos fJ - N => N = mg cos fJ


360 Chapter 5 Friction

(Note that the moment equation would give us the location of the normal
force N.)
If we slowly increase () until the block: slides, then at this critical
angle (call it ()s), we have f = pN. Therefore, at () = ()s,

f = mg sin ()s = J.lN = J.l mg cos 8s


or
J.l = tan ()s

so that the tangent of the angle at which slipping begins is the coefficient
of friction for the two materials of interest. The angle ()s is called the
angle of friction, or angle of repose. "
In the following table, some typical values of J.ls are presented for
various selected pairs of materials.

Approximate* Values of the Coefficient of Static Friction


for Clean, Dry Surfaces

Surface Materials Coefficient of Friction

Steel on steel 0.75


Rubber on concrete 0.8
Rubber on asphalt 0.85
Rubber on ice 0.1
Oak on oak 0.6
Wood on metal 0.4
Cast iron on cast iron 1.1
Cast iron on copper 1.05
Teflon on teflon or steel 0.04
Copper on steel 0.5
f
Aluminum on aluminum
~,~ r

1.1 <t. ~. ~
'i

Steel on lead 0.95 l 'i'


Aluminum on steel 0.6 ~:J ,il
Brass on brass 0.9 !
Hemp rope on wood 0.7 :.
" .1;,
Metal on stone .. 0.5 ' [

Metal on leather 0.45


Metal on ice . 0.04
Oak on leather 0.6
Bonded carbide on iron 0.8
Glass on glass 0.9
Copper on copper 1.2

We now proceed to some examples in which we have friction acting


between various flat surfaces.

*Values of Jl may vary widely for a pair of matetials, depending on such things as
whether the steel is mild or hard, whether the friction on the wood is with or across the
grain, and so forth. The tabulated values are intended just to give the reader a general idea of
values of Jl.
~

E x A M p L E

In Figure 1 find the smallest force P that will prevent the block from sliding down
the plane. The block weighs 200 lb and the friction coefficientbetween it and the
plane is 004.

Figure 1

SOLUTION
We note first that the block will not remain in equilibrium without force P
because tan 30° = 0.577 > Jl = 004. For the sniallest forcep, the friction force
will be at its maximum, acting up the plane as shown in the free-body diagram of
Figure 2. Equilibrium requires

<\ <j

Figure 2

. ' LFx = 0.= 200 sin 30° -OAN ,- P (ll

and

LFy = 0 = 200 cos 30° - N (2)

From Equation (2), N = 173 lb. Substituting this result into Equationll) yields

P = 31lb

361
b

E x A M p L E
Find the condition for which the uniform crate, pushed to the right by a slowly
increasing force P, will slide before it tips over.

SOLUTION

The equations of equilibrium are seen to be (note the free-body diagram) ,


\.
-,

....:t... I:.Fx = 0 =P - f => P =f


+t I:.Fy =0=N- mg => N = mg

~ I:.MA = 0 = No - Ph => No = Ph
If we assume that the block slips before tipping, then at the point of slipping
jor sliding), f = pN. The equations then give

No = Ph = fh = /lNh
or
and P = /lmg

To check whether our assumption of slipping was correct,. 'we must


determine whether or not N acts between thei"centerline and the lower right
corner of the block; that is, 0 must lie between zero and b/2 for slipping to occur
first.
b
o=/lh<-
-2
){

or
':'. ~
'}

"l":"
Thus if the base dimension b is greater than 2}Jh, the block will slip. Equality
of band 2/lh means slipping and tipping occur simultaneously, and 2}Jh > b
')
,. -0

implies that tipping happens first. ~~


>.

~f

P-:""f ,';..
,
h
r
• ,~r

.i,
Let us check these results by starting over with a different assumption: that ~l

tipping occurs first: If so, both Nand f act at the lower right-hand corner of the
block as shown in the figure above. We obtain, from the moment equation,

b
I:.MB = 0 = N - - fh
2
(Continued)

362
30(9.81)N 3. All three blocks are about to move together, with impending slipping of
.A on the plane.

+t = N3 =491 N
I .1 ~9:'1: LFy = 0 = N3

i3 = i3MAX
- 50(9.811

= /13N3 = 0.25(4911 = 123N


RI5(9.81)~
.J:..... LFx = 0 = F - i3 = F = 123 N,

the highest of the three values of F. Therefore the maximum F for equilibrium
of the stack of three blocks is 103 newtons.
Intuition tells us that the case of .A and C sliding to the right with B
remaining stationary is impossible. It is interesting to study the reason why.

1,--
6,
-t-
The free-body diagrams at the left show the directions of the various friction
forces iI' i2' and i3 upon application of F. Note that if B remains stationary, then
i. balances i2 in the figure. But for .A to slide to the right relative to B would re-
quire i2 to be opposite to the directions indicated on both .A and If, opposing
the relative sliding. Two contradictions would then result: lal .A would no
30(9.81)N
longer have an applied force to the right to tend to make it move that way; (blon
B, i. and i2 would add and thus not balance; B could not then remain stIll.
130(9.81)N' Thus B cannot remain stationary unless .A does also.
-I-t-- I,
5(9.81)N ~ IJ

l35(9.81)N

15(9.
B1
1N\
134
t I. A- (
-
50(9.81)N

IE. X L E
We have seen on page 360 (without the rope and body .A present I block B will
slide down the incline if tan <p > /1. Let <p = 600 and /1 = 0.6, so that

tan <p = J3 > /1 = 0.6


Determine the range of values of the weight of .A that will result in
equilibrium of the system.

SOL UTI 0 N
There is a minimum value of .A that will prevent B from sliding down the
plane; similarly, there is also a maximum weight of .A above which .A will pull
(Continued)

364
B up the plane. Between these two weights, the bodies will be in equilibrium
with the friction force fbeneath B lying between its maxima up and down
the plane, respectively. In the first case, the motion of B impends downward' -
(see the illustrationl. Equilibrium requires

:EFx = 0 = mg sin 60° - T - fMAX (1)

~Fy = 0 = mg cos 60° - N = N= mg (2)


2

T
T

/'

... T == WA

But fMAX = J.l.N = 0.6mg/2 = 0.3mg, so that, from Equation (1),

-
T = mg~
J3 - fMAX = mg
(J3)'
2 - 0.3 = 0.566mg
'
and
W""M1N = T == 0.566mg

T In the second case, the motion of B impends upward, with friction resist-
ing this tendency, acting downward (see the illust~ation):

:EFx = O=mg sin 60° - T + fMAX (3)

mg
:EFy = 0 =' mg cos 60° - N =N = -. - as before. (4)
2

Again, fMAX = J.1.N ~ 0.3mg. This time, the first equation (3)gives

i' :: mg ';;'+ fMAX = mg(';; + 0.3) ~ 1.17mg


and
W..••
MAX = T = 1.17mg

Any weight of .A between 0.566mg and 1.17mg will result .in equilib-
rium of the two bodies. Note that one of the values of the friction force, for
W..••in this range, is zero. This happens when T ,,; mg sin 60°, or for W..••==
mgJ312 = 0.866mg. ,.

Our first four examples have been concerned with impending sliding
of flat surfaces in contact. Friction also develops in rolling situatio~s, as

365
366 Chapter 5 Friction

we shall see in the following two examples. Then we shall close the
section with four additional examples.

,
•j
,i
E x A M P L E
Three identical cylinders are stacked as shown ih the figure. Assuming that the
friction coefficient J1 is the same for all pairs of contacting surfaces, determine the .,
minimum value of J1 for which the cylinders will remain stacked in equilibrium.
~,
~.

SOLUTION
From the free-body diagram of the lower-left cylinder B in the figure, we can
sum moments about B and immediately find a relation between the forces f
andN:
f
~MB = 0= N!- - fT(1 + 1312)
2

from which

f= O.268N (I)

Since f is limited by J1N, the above result means that the critical (smallest) value
of J1 that will prevent Band C from rolling outward on the ground (while each
slips on.A) is 0.268.

,.
~,

We must also investigate the possibility of Band C skidding outward on the


plane while maintaining rolling contact wit~. the upper cylinder .A. To this end,
we sum moments about A on the same free-body above:

!MA =0 = (mg - N') !- + I'TlI -+ 1312) (2)


2

If we knew the relation between mg and N', we could obtain the relation between
I' and N' that we need. From the "overall" free-body at the left,
~Fy = 0 = 2N' - 3mg

N' N' (Continued)


;I
t '
'P-~-7-, ',-S' t"'.' -~-.:;J'"""l:::iic;iiJ"""',"'.~"'r.
..,,= c~---------------------------
.. _ ••

so that

2
rug = -N'
3

Substituting tN' for rug in Equation (21gives:

f' == 0_268N, = 0.089N' (3)


3

Therefore, to prevent "skidding" of Band C on the plane, the friction coefficient


J1 needs to be only one-third that of the other case. Therefore the value needed to
prevent both motions is 0.268 _Note that Equations (1) and (3)together tell us that
t1- ,
the two ways in which equilibrium can be lost cannot occur simultaneously.

E x A M p L E
A truck is preparing to push a drum up an 80 incline. The coefficient oUriction

I
i
between the drum and the incline is 0.3. Find the range of positive values ofthe
friction coefficient J1 between the drum and truck' bumper so that the drum rolls
(does not slip) on the ground.
I"
t

I
!

SOLUTION
. Let us assume.that the drum is in equilibrium----'that is, that its motion has not
quite yet begun:

LFx = 0 = N, - f2 - rug sin 8 0


III
LF}, = 0 = N2 - f, - rug cos 8 0
(2)

LMc = 0 = f,l - f21 = f, = f2 (31

Thus we have three equations in the four unknowns N" N2, f" and f2 .
Assuming that slipping impends at A (which it will if the drum is about to roll on
the ground) means

f, = J1N, (4)

(Continued)

367
---- -------------------~
I
I

Thus (I) and (3) give

Ntl1 - Jll = mgsin 8°


(5)
mg sin 8°
Nt =----
1-Jl

Equation (2) gives

N2 = JlNt + mg cos 8°

= --mgsm
Jl . 80 + mgcos 80
1 - Jl

N2 = mg(cos 8° + _Jl_ sin 80)


1 - Jl

And from Equations (3), (41,and (5) we have

Jlmg sin 8°
f2 = ft = ----
I - Jl

If the drum is not to slip on the incline, then


,,
f2 $: 0.3 N2

Substituting for f2 and N2, and canceling mg,

_Jl_
I-Jl
sin 8° $: 0.3 [cos 8° + _Jl_
1-Jl
sin 80J
0.7 _Jl_ sin 8° $: 0.3 cos 8°
1 - Jl .

JlO.0974
$: (1 - JlHO.297)

0.297
Jl < _.- = 0.754
- 0.394

E x A M 1Ft L 5.7~
"I
~
A

The uniform beam Bweighs 1000 N, and is attached to the light shoe S, which
rests on the cylinder C. How much weight can be suspended in equilibrium by a
cord wrapped tightly around the hub of C as shown?
(Continued)

368
,~----------------------------------------------------------
."'.

SOLUTION
Free-body diagrams of (a)beam plus shoe, and of (b)cylinder plus weight are shown
in the figure.

IOOON

-j~:
O.15m
-t 1
1 ;-

If we imagine the weight Wwto be slowly increased, it will eventually reach


a critical value at which cylinder C will begin to turn counterclockwise. This has
to happen because the free-body diagram of (C + W) shows that the only moment
about C available to equilibrate that of Wwis the moment of the friction force f.
That moment is bounded by /IN times the radius 0.3 m. Thus we first need N.
From the free-body diagram of (B + S), we see that:

1. ° °
Writing ~Fx = and !:Fy = will not be of help because they will include
Ax and Ay, which are undesired unknowns.
2. Moments about A will allow us to determine N without finding Ax and Ay:

'+) ~MA = ° == (1000 newtons)(l ml - N(l.5 m) - f(0.15 m) (1)

At the critical value of Wu!' which we seek, fbecomes fMAX, or /IN, so that

.1000
N=--
1.53

= 654 newtons
We return to the free-body diagram of (C + WI. The same remark (I) above
again holds, this time with the pin reactions Cx and Cy unknown and unwanted.
Thus again we proceed directly to the moment equation that eliminates these
reactions from consideration:*

'+) ~Mc = ° = WwIO.2ml - (0.2N)(0.3 ml (2)

(Continued)

* If we were seeking Cx and Cy in addition to Ww' we would still writeI:Mc = 0 first to


get Ww. Then, writing I:Fx = 0 and I:Fy = 0 would yield the pin reactions.
so that

(0.2)(654)(0.3)
WU'=----- 196 newtons
0.2

It is instructive to consider what changes might occur in the solution if the weight
Wcomes off the right side of.the hub of the cylinder. instead ofthe left side. 0_-

}
j

i .
.
E x A M p L E , 'f

The front wheels of a certain car support 55% of its weight Won level ground.
The coefficient of friction between the tires and an inclined road surface is 0.3.
What is the largest incline angle P for which th~ car will not slip when parked?
(Only the rear wheels are locked.)

SOLUTION
In equilibrium on level ground, by summing moments about R we find that the
... 't,
mass center C of the car is 4.95 ft forward of the rear wheels (see the free-body
diagram):

~ ~MR =0 = 9N1 - xW
= 9(0.55W} - xW

x = 4.95 ft

Putting the car on the incline, we sum moments about the bottOm (F) of the
(3..
front wheel and determine N, (see the free-body' diagraml:

~MF = 0= W cos P(4.05 ftl + W sin P(2.5 ft) - N,l9 ftl (II

N, = W(0,45 cos P + 0.278 sin Pl


Also,

~F" = 0 = f - W sin P
f == Wsin P
But for the car not to slip,

(Continued)

370
Thus

W sin /3 ::;; 0.3W[0.45 cos /3 + 0.278 sin /31


or

0.917 sin /3::;; 0.3(0.45) cos/3

tan /3 ::;;0.147
i /3 ::;; 8.36°
i
I
I
I
Note that, all other parameters remaining equal, raising the mass center
(increasing the 2.5-ft dimension) increases the "W sin /3" coefficient in Equa-
tion (1). This then has the effect of raising both Nr and the angle /3, that comes
out of the inequality above. Physically, a larger Nr with the same J.I. gives
more maximum frictional resistance to slipping.

E x A M p L E 5J9~
The angle ()is slowly increased from zero. Find the value of ()at which the slotted
cylinder of mass m will begin to slip.

,
"
SOLUTION
The cylinder cannot roll on the plane because it is:restrained by the cord, which is
assumed to be wrapped tightly and not slipping in the slot. At a certain value of (),
the cylinder will slip downward on the plane while pivoting about the point A
shown on the free-body diagram. It will do this when the value of I needed for
equilibrium exceeds the maximum value possible for I, which.is J.l.N.

~Fx =0 = mg sin () - T - I III


~Fy = 0 = mg cos () - N =N = mg cos () 121
Img sin ()
~MA = 0 = - Img sin () + I(R + I) =I = ---
R +I
(3)

(ContiilUed)
Enforcing the no-slipping condition,

f ~ j1.N

rrug sin 0
---- ~ j1.mg cos 0
R +r
j1.(R + r)
tan O ~---
r

Oslip = tan
_l(j1.(R +
r
rl)
This angle could be substituted into Equations (3)and III to yield the values
of the friction force f and the tension T at the slipping angle Oslip'

E x A M p L E 5~10~
-b- The pipe wrench (or Stillson wrench) is gripping a pipe P that is about to be
unscrewed from a fitting by the force P as shown in the figure. The member I]
is pinned to JJ and is loose-fitting on :J The friction coefficients j1. between :J
and P and between JJ and P are given to be equal. Find the relationship between
j1., D, h, b, and L for which the pipe wrench will be "self-locking," meaning that it
will not slip on the pipe regardless of the size of P.Neglect all weights, and for the
purposes of this example, make the assumption that the teeth of the wrench do
not significantly dig into the surface of the pipe.

SOL UTI ON
Separate free-body diagrams of the wrench as well as two of the parts of the
.J/_I wrench are shown in Figure 1. We see immediately from Figure 1(al that
L .rr. '1
~ ~~=O=h-~+P=P~~-h (1 )

"+t' ~F;c=O = NT - NB ~ Ni:~ N;' 121


t :;
~ ~MT = 0 = (D + h + LIP - fBD (31 '1 ~
Notice that we are not yet stating that either of the friction forces f is equal to 1lN.
We shall first get the equilibrium equations written, then investigate the slipping
possibilities.
p Using the free-body diagram [Figure 1(b))of the combined body II] + :Jl, we
get

~ ~Fx = 0 = fT - Rx = fT = Rx (41

+t ~Fy = 0 ~ NT - Ry = NT = Ry (5)

~ ~MR = 0 = NTb - fT(D + hI (61

(Continued)

372
(b)
.J.J___..

p p

(a) (c)
Figure 1

Using Equation (6) we can immediately obtain the minimum value of the
coefficient of friction J1. needed to prevent slipping at the top of the pipe (point T):

(7)

or
~' --Jr"., -'.

b
,,>-- 18)
r-D+h
We proceed to obtain a similar condition on J1. for which slipping will not
occur at the bottom point (EI of the pipe. Equations (1I and (3)give, eliminating p,

fBD
fB - fT = D +h +L (9)

or, using Equation (7),


.....':.".....,....
( h+ +h L)+ L . = fT b
,
i
(10)
t,
I
, fB D = NT D + h

I (Continued)

Ii.
L ,.. '
0.5 in.

I in.

J in.
1.25 in. J"
) in.

7.5 in.

10.3 in.

" ("

i
i,
i
I

But NT = NB by Equation (2); thus

h+L) b (11 )
fB (D + h + L = D + h NB

or

i __ b_(D_'_+_h_"_+_L_l N < i-uN (12)


IB - (h + LHD + h)B - IBMAx - r B

so that, this time,

biD + h + Ll (13)
IJ. ~ (h + LHD + hI
This coefficient is slightly higher than the one given in inequality (8), so there
will be no slipping on either contact point (T or B) with the pipe if IJ. ~
biD + h + L)/[Ih + LHD + h)).
For the larger wrench in the photo, for example, we see that D, h, b, and L
are, respectively, 1 in:, 1.25 in., 0.-75 in., and 10.3 in. Therefore, the inequal-

(Continued)

374
r

ities (8) and (13) give

0.75
J1 :2: ---- = 0.33 needed to prevent slip at T
I + 1.25
and

0.75(1 + 1.25 + 10.3)


J1 :2: (1.25 + 10.31(1 + 1.251 = 0.36 needed to prevent slip at B.

With the teeth that are actually built into the jaws of a pipe wrench, 0.36 is easily
obtained.
Note from the photos that the jaws are not exactly parallel; resistance to slip
is further increased because of this angle. Note also that this result, J1 = 0.36, is
fora I-inch pipe (D = I in.l; for other diameters, different answers for J1 will be
obtained.

The reader may wish to show that for the smaller wrench in the figure, less
friction is needed for self-locking against a I-inch pipe. Another follow-up
exercise is to solve Equations (11-(6) for tT' NTJ'iB' NB, Rx' and Ry in terms of P,
and then to show that the equilibrium equations for body jJ [Figure 1(ell are
satisfied by these results.

PRO B L EMS / Section 5.1

5.1 The electronics cabinet is 7 ft high, weighs 200 lb and 5.2 The friction coefficients for the bodies in Figure P5.2
has its mass center at C. The coefficient of friction between are: 0.4 between 81 and 82 and 0.2 between the floor and
the cabinet and the floor is 0.3. Without disturbing 82. The mass of 81 is 10 kg. Find the minimum force P
equilibrium, how large a force P can be applied needed to disturb equilibrium if lal mass of 82 =8 kg;
(b)mass of 8'2 == .12kg:'
a. to the right?
b. to the left? 5.3 If each of WI and W2 weighs 100 Ib, and if the
coefficient of friction for all surfaces of contact is 0.2, find
the angle a for which WI begins to slide downward. (See
Figure P5.3.)

••
Figure P5.1
1
Sft
Figure P5.2

Figure P5.3

I .. 375
L
376 Chapter 5 Friction

5.4 In Figure P5.4 blocks A and Bweigh 50 lb and 1001b,


respectively. The coefficients of friction are: /11 = 0.2:
between A and B, and 112 = 0.35 between B and the
Figure PS.4
horizontal plane. Find the smallest force P that will cause
A to move to the left.

5.5 Block A, weighing 501b, rests on block B, which


weighs 100 lb. (See Figure P5.5.) The coefficient of friction
between A and B is 0.4. Neglecting friction between Band
the floor, find the largest force P for which B does not move.

5.6 In Figure P5.6 find the smallest force P for which the
blocks will slide if angle cP is 20°.

5.7 Find the maximum-force P for which no slipping will


occur anywhere for the system of blocks in Figure P5. 7.
Figure PS.S
5.8 Repeat the preceding problem, if the values of /11 and
J.l3 are reversed.
p 5.9 Two identical triangular blocks, each of mass ill, are
put together to form a rectangular solid, and they are being
held in equilibrium by the force P (see Figure P5.9). If P is
then increased until motion impends, at what value of P
will this occur? Consider all possibilities.
Il ,,= 0.3
5.10 In Figure P5. 10 a small box rests on the parabolic
Figure PS.6
incline at the point (1, tl. If the box is on the verge of
slipping, what is the coefficient of friction between it and
the incline? Also, find the force P that must be exerted on
the box, tangent to the incline, to start it moving upward.

5.11 In a strength test of fiber optic cable, the nut in Figure.


P5.11a was attached to the jacket of the cable, which was
then slipped through a slot in the channel (Figure P5.11 b).

Figure PS.9 Y
x2
Y=T
I

p 2

'Figure PS.7 )
x

Figure PS.10

Figure PS.11

lal (bl
----
T
Ie)
T
5.1 Laws, Coefficients,and Basic Applicationsof Coulomb F'riction 377

Figure P5.13 Figure P5.14 ,

JJ.2 = 0.5
Figure P5.1 2

..-..-----.... ---..
/
/'
/'
""-
/ \
/ \
/ \
I \
I I Frictionless 0

\ J pulley
Figure P5.15
-I
D

t
Figure P5.16
Figure P5.17

The coefficient of friction between nut and channel was-


0.1. The load T in Figure P5.IIc was then slowly a. Is theblock necessary?
increased, and at 200 lb it was observed that a bend b. If so, what is the minimum block mass to keep the
induced in the channeL caused the_nuLto ,slip .0fLthe.- .• .man andJadder.from ~i1ipping?
channel. At what angle (J did this occur?
5.15 In Figure P5. IS block'B weighs 50 lb. Determine the
5.12 What force Pwill cause motion of the IOO-Ibcylinder range of values of the weight of..A that will maintain
in Figure P5. 12 to impend? equilibrium of the system. (Besure to check all cases!)

5.13 In figure' P5.13 bodies ..A and B respectively weigh 5.16 In Figure P5.16 a bug falls into a bowl that has the
'.., 2000 Nand 1500 N. The various coefficients of friction shape of an inverted spherical cap of radius R. The
are: between..A and the plane, 0.2; between..A and B, 0.3; coefficient of friction between the bowl and the bug's feet
;'.",-: between B and the plane, 0.4. Find the magnitude of cou- is J1 = t. The bug is barely able to crawl out without
ple Cthat will cause body B to have impending motion. slipping. Find the depth D of the bowl.

5.14 A IOO-kgman climbs a 30-kg ladder that is II min 5.17 The cylinder in Figure P5.17 weighs 200 lb and is in
length and makes a30° angle with the vertical. (SeeFigure equilibrium on the inclined plane. What is the friction
P5.14.1 The man has placed a solid block at the foot of the force at its point of contact? If it is on the verge of slipping
ladder to prevent slipping. on the plane, what is the c~efficient of friction?
378 Chapter 5 Friction

5.18 A slender uniform rod of length 17 in. and weight


50 lb is smoothly hinged to a fixed support at A and rests on Light, thin
bar
a half-cylinder block at B, where the contact is smooth. (See
Figure P5.18.j
a. Find the smallest coefficient of friction )1 between the
70-lb block and the plane for which the system can be
in equilibrium as shown.
b. For this value of )1, find the location of the normal
force resultant between the block and the plane.

ri
Figure P5.20
'.~

"

5.22 The 240-N . m couple is applied to the bar DB. Bar


5 in. AC rests on DB in equilibri~m as shown in Figure P5.22.
Find the minimum coefficient of friction between the two
~
bars for which the equilibrium can exist.

Figure P5.18

5.19 The coefficient of friction between the drum and


brake shown in Figure P5.19 is 0.5. la) Show that there is
-r
adequate friction for equilibrium. (bl Determine the hor- 0.35 m
izontaland vertical components of the pin reaction at 0 on
the brake. The drum weighs 100 lb and the weight of the
brake is 10 lb/ft. The brake is a uniform bar. D ...:
f-O.2 m+-o.3 m-I
1
Figure P5.22

5.23 Repeat the preceding problem if the 240-N' m


couple is replaced by an upward force of 480N at D. Tell
why the answer for )1min is the same in the two problems,
but not the reaction at B.., .~- .

5.24 In Problem 3.272, suppose the bar and block are


made of the same material and their coefficients of friction .,
it '
)1 with the plane are the same. What is the minimum value
of )1 for which the equilibrium can exist?
1:
5.25 The homogeneous cylinder in Figure P5.25 weighs 1
'j
FigureP5.1 g 425lb and the block weighs 300 lb. The coefficient of
f
i
5.20 As P increases from an equilibrium value, where does
the cylinder slip first, and at what value of P does this
occur? (SeeFigure P5.20.) Figure P5.25
2ft
t
5.21 Repeat the preceding problem with the friction
coefficients reversed.
~
5.1 Laws, Coefficients, and Basic Applications of Coulomb Friction 379

friction for all surfaces of contact is 0.25. Moment To is


slowly increased from Zero. Determine the value of To that
will cause the cylinder to have impending motion, assum-
~ . ihg that the block does not tip.

5.26 The 140-N triangular block in Figure 1'5.26 is pulled


at the top by a gradually increasing horizontal force P. The
coefficient of friction between the block and the floor is
J.l = 0.4. Find the smallest base dimension B for which
the uniform block will slide instead of tipping.

Figure P5.28

5.29 The man in Figure 1'5:29 is trying to ride a one-speed


bicycle up a steep incline .. He is tiring, and the bike is
mo,:ing very slowly. Find the normal forces beneath each
tire, and the friction force exerted up the plane by the
pavement on the rear tire. Neglect the friction beneath the
non-driven front wheel, and take the total weight of bike
and man to be 190 lb with mass center at C. Also find the
minimum coefficient of friction between tire and incline
for which the wheel will not slip.

F.igureP5.26

5.27 Find the weight of the block that will prevent


slipping of the 300-N ladder if the coefficients of friction
are: 0.3 at A, 0.1 at B, zero at C, and 0.4 between the block
and the floor. (See Figure 1'5.27.1

5.28 Given a homogeneous block on an inclined plane as


shown in Figure 1'5.28 find the angle e for which the block

I .' is on the verge of tipping. Then find the coefficient of
friction J.l for which the block is also on the verge of sliding.

~i:~g~~:rt a~~ r:~~:~:;:re;:~e~"~;~~:~l~:t;:si;~~~:;~~" ~',' ' \.,

'\
Figure P5.29

5.30 Extend the preceding problem. Knowing the friction


force, use the free-body diagram of the reat wheel to
determine the force T in the upper segment of the chain.
Figure P5.27 Then use the free-body diagram of the pedal sprocket to
380 Chapter 5 Friction

FigureP5.31

Figure P5.30 Figure P5.32

Figure P5.33
find the force F that the man's, left foot is exerting at the
instant shown in Figure P5.30.

5.31 The girl in Figure P5.31 is learning to use a wheel-


chair. What force must she exert tangent to the handwheel
(radius IIto make the wheelchair begin to roll up the incline
(angle 4>)1 The radius of the large wheels is R; the masses of
patient and wheelchair are M and m, respectively.

5.32 In Figure P5.32 body A weighs 270 N and body B


weighs 180 N. The coefficient of static friction between
body A and the plane is 0.30. Determine the range
of values of the fo!ce P f9r *which body A will be in Figure P5.34
equilibrium. ' .," ' -~."
.•.- ,. ',., ,. .

5.33 In Figure P5.33 what is the largest angle 4> for which
the slotted cylinder will remain in equilibrium when
released from rest on the inclined plane? The coefficient of
friction between the cylinder and the plane is J1 = i.

5.34 The slotted cylinder is in equilibrium on the inclined


plane in FigureP5.34. If the spring is stretched 4 in., what
is its modulus? What is the minimum friction coefficient
between the cylinder and the plane for which the equi- Figure P5.35
librium can exist?

5.35 In Figure P5.35 a solid circular disk of weight 65 lb is


connected to a 130.lb block by a light bar AB. Find the
5.1 Laws, Coefficients, and Basic Applications of Coulomb Friction 381

smallest coefficient of friction between the block and the b. Argue from new free-body diagrams that if M is
plane for which the system will be in equilibrium. Assume applied instead to K?, the same answer is obtained for
that the block is wide enough so that it will not tip. the required couple.

5.36 In Figure PS.36, the light rod is connected to the


drum and the block at pins. The block weighs.26Jb and ..... _ .._ ..
the coefficient of static friction for all surfaces is 0.6. If
the system is to be in equilibrium, what is the maximum
allowable value for the weight of the drum?

Figure P5.39

5.40 In Figure PS.40 the coefficient of friction /-l is the


same between A and IJ as it is between IJ and the plane.
Find the minimum value of /-l for which the system will
Figure P5.36
remain in equilibrium, and the tension in the cord for this
value of /-l.
5.37 The coefficient of friction between the homogeneous
5.41 Repeat the preceding problem with the altered pulley
beam and the comer at A is /-l = 0.3. (See Figure PS.37.) arrangement shown in Figure PS.41.
Find the coordinates of the mass center of the beam if it is
in equilibrium but on the verge of slipping at A.

Figure P5.40

FigureP5.37

5.38 Using Figure PS.37, suppose that the distance from A


to B is 1.6 m and that the beam is now uniform. Find the
minimum coefficient of friction between the beam and the
comer support that will prevent slipping of the beam.
Figure P5.41
5.39 The cylinder C and ring I< in Figure PS.39 are of
the same radius r and weight W, and are connected by the
light rod AB.
a. Find the couple M, which when applied to C results
in equilibrium. Assume no slipping.
382 Chapter 5 Friction

5.42 In Problem 5.40, suppose the common friction 5.49 In Figure P5.49 a heavy man of mass ill has placed a
coefficient is J1 = 0.2, and the angle of the plane is now light ladder of length L at a dangerous angle, and has begun
variable. Find the largest angle of the plane for which the to climb it. If the coefficients of friction are 0.25 between
blocks will remain in equilibrium. both the ladder and ground, and the ladder and wall, find
5.43 In Figure P5.43 Mr. A ~eighs 160lb and Mr. B
how far up the ladder IlL in the figure) the man can climb
weighs 200 lb. Each man pulls on the rope with a force of before the ladder slips. ShOw further that prior to the
20 lb. slipping position, the groun4 and wall reactions cannot be
found (Le., that the problem is statically indeterminate).
a. What force .does the fish scale register?
b. What are the magnitudes of the friction forces on
Mr. A and Mr. B?
c. If Messrs. A and B are wearing the same kind of shoes
and if there is impending slipping, which man is
about to slip? What is the coefficient of static
friction J1s? Figure P5.44

5.44 In Figure P5.44, the 50-lb rod is pinned smoothly p


to the ceiling at A, and it rests on the 100-lb wedge at B.
;.~_.
The coefficient of friction between the two bodies is 0.2,
and that between the wedge and the floor is 0.1. Find the 12ft
force P for which motion of the wedge to the left will be
impending.

5.45 In the preceding problem, is any force P needed to


prevent the wedge sliding to the right? If so, find itj if not,
find the force P to the right that will start motion of the
wedge to the right.

5.46 In Problems 3.275 and 3.276, find the minimum p


coefficient of friction between the block and the slotted
body for which the equilibrium can exist.
II- = 0.4
5.47 The uniform blocks..A and Bweigh 90 lb and 60 lb, Figure P5.47
respectively. The coefficients of friction at the three
contacting surfaces are shown in Figure P5.4 7. Find the
maximum value of P for which equilibrium is possible. Is
the answer the same if P is pushing to the left on B instead
of pulling to the !igh;t?" . I N

5.48 Repeat the preceding problem if the coefficient of


friction between ..A and B is changed from 0.3 to 0.1.

Figure P5.43 Figure P5.49


5.1 Laws, Coefficients, and Basic Applications of Coulomb Friction 383

Figure P5.50

6ft 6ft

FigureP5.51

Figure P5.52

5.50 The stick of mass m was originally in equilibrium


with the string force S acting vertically 18= 90°). The
force S is now slowly turned clockwise as indicated in
Figure P5.50, with the stick remaining in equilibrium in Figure P5.53
, ., the same position.
a. Find the force S as a function of 8 (and mg and ~), and
B
show that it must increase in magnitude.
b~ Let ~ = 30° and mg = 5 lb. If the stick is observed to
slip at 8 = 30°, find the coefficient of friction be-
tween the stick and the floor.
Figure P5.54 m.
5.51 In Figure ..PS,.5L find.the largestJorce.,that .can.be
developed in the tow cable if the coefficient of friction p
between the back tires of the truck and the ground is 0.25. 1m
The truck weighs 5000 lb and is rear-wheel driven.

5.52 The spool is in equilibrium as shown in Figure P5.52.


Find the minimum value of the coefficient of friction for
which this is possible. Hint: Angle ~ can be found from the b. At this value of P, what is the cylinder on the verge of
dimensions given. Also, assume that the rope is tightly doing?
wrapped.
5.54 A uniform ladder, mass 12 kg, rests against a wall
5.53 In Figure PS.53 the block and cylinder each weigh and against a horizontal surface, as shown in FigureP5.S4.
300N. If the coefficient of friction at A and B is 0.3, find the
a. Show that the block will be on the verge of slipping smallest force P that will prevent motion of the ladder to
when P is increased to the value 120/cos 8 N. the left at A.
384 Chapter 5 Friction

5.55 The bodies A IS, and C in Figure P5.55 each weigh


20 lb and are in equilibrium . ..A and C are pinned to B at its
ends. Find the force P (magnitude and sense) and the forces
exerted by the slot On ..A and by the circular track on C.

Figure P5.58

Figure P5.59

'Figure P5,55
dent of friction between tires and street in order to prevent
slipping of the car if only the rear wheels are locked?
5.56 In the preceding problem, let P be replaced by a
5.60 Repeat the preceding problem if all four wheels are
couple M applied to C.Find M. Also find the minimum
locked.
coefficient of friction between C and the rough track for
equilibrium.

5.57 The uniform 1O-lbrod and 1OO-lbcylinder are pinned


smoothly at C. If the 2-lb force is applied as shown in Figure
P5.57 and the system remains in equilibrium, then the
coefficient offriction between rod and floor has to be larger
than a certain value. What is this value?

21b
~ ~

FigureP5.57
Figure P5.61

5.58 In Figure P5.58 the collar C is very light compared 5.61 The rope is held securely by the self-clamping device
with the force P. his fitted over the rough vertical shaft S in Figure P5. 6 1.
with a slight amount of clearance. Show that if the distance
d is greater than H/(2/ll - r, then C will not slip down the a. Explain how both the actual and maximum hiction
shaft regardless of the size of P. Hint: There will be contact forces increase with P.
at two points of S. b. Find the minimum coefficient of friction between I' '.~

device and rope to prevent slipping.


5.59 In Example 5.8 suppose the car is to be parked on a c. Find the resultant force exerted by the pin at A onto 1 "-
15° incline. (See Figure P5.59.1 What must be the coeffi- the support if P = 500 lb. F
1
r-''i
,,
l
5.1 Laws, Coefficients, and Basic Applications of Coulomb Friction 385

(
, 5.62 In Figure P5.62if J.lI = J.l2, then as P increases from weigh W, and if the friction coefficients at the three
its lowest possible equilibrium value, will the cylinder C contacting surfaces are all equal (J.l), find the minimum.
first roll, or slide, on the plane for (al () = 30°. (bl () = 45 0, value of J.l for which equilibrium will exist. (See Figure
'~,: (cl() = 60°1 Hint: Show that for rolling, P5.65.1Assume that Bis wide enough so that it will not tip.

W sin ()
PI =---
1 - J.lI

and for sliding,

P2 = W[ sin () +
J.l2
.
cos (}J
1 - J.l2

and compare the numerical values.

. Figure P5.65

5.66 Two identical cylinders are placed together on an


incline as shown in Figure 1>5.66.Show that the cylinders
cannot remain in equilibrium. Hint: Write the three
equilibrium equations for each cylinder. Show that they
require the normal force between them to be zero. Thus
conclude that: (a) the friction between them is also zero;
(bl there can be no friction beneath the cylinders if there is
Figure P5.62 equilibrium; and finally (c)the component of the weights
down the plane is unbalanced, which is a contradiction.
5.63 The cylinder and the block each weigh 26 N. (See
Figure P5.63.) The friction coefficient is i between block
and cylinder, i between cylinder and plane, and i between
block and plane. Find the smallest force P for which motion
will begin up the plane if it is known that the block won't Figure P5.66
tip.

5.67 In the preceding problem, suppose that a normal


force between the cylinders is created by a pair of springs
(see Figure P5.67), one at! each end of the cylinders. If
ill = 20 kg, J.l = 0.6 at all surfaces, and </> = 20°, find
Jl = 1/4 at all contacting
surfaces the spring tension required for equilibrium of the two
cylinders.
Figure P5.63

5.64 In the preceding problem, find the value of the


common friction coefficient below which the cylinder will
roll on the plane and above which it will slip on the plane, Figure P5.67
provided J.l =1= O.

5.65 Without block B present, the center of the wheel C


will move to the right on the plane. If ..A,B, and C each
386 Chapter 5 Friction

Figure P5.69

Figure PS.68

FigureP5.71

5.72 The 130-lb homogeneous plank of length 6 ft is


placed on the two homogeneous cylindrical rollers, each
of weight 30 Ib, and held there by the force P.(See Fig-
ure P5. 72.1
a. Find P for equilibrium of the system, assuming no
slipping occurs anywhere.
b. If the coefficient of friction /l at all surfaces of
contact is the same, find the smallest /l needed for
Figure P5.72 equilibrium.

5.68 A vehicle of mass in Figure P5.68 is driven by a


ill In Problems 5.73-5.76 theJeft support is 1.2 m from the
gear and pinion as follows: A motor exens a constant mo- left end of the beam, and the force P is holding the. beam in
ment Mo <J onto the pinion P It then drives the gear (; equilibrium. Find the force P required to stan the beam
that is attached to a rear wheel, thereby turning the wheel moving to the left. The thin beam in each figure;weighs
counterclockwise and moving the vehicle up the plane. 2600 N. .
What value of Mo is needed to barely move the vehicle if:
5.73
a. Axle friction' is negligible?
b. Axle friction gives a constant resistive moment of Mf
opposing the rolling?

5.69 lin Example 5.61et the two friction coefficients be /l~


between drum and plane and /IT between drum and truck.
Find the reIatiortship'l:ietwe~n /lp"and /IT for which the
drum will roll up the plane (i.e., will not slip on the ground).
(See Figure P5.69.) The answer should be of the form Figure PS.73
/IT ::;; II/lp).

5.70 In Example 5.6 let both friction coefficients be 0.3.


5.74
Fihd the maximum incline angle 1> for which the drum will
roll (i.e, not slip on the inclined plane).

5.71 In Figure P5.7l the cylinder is in equilibrium.


Determine:
a. The stretch in the spring
b. The minimum coefficient of friction /l for which the
equilibrium is possible. Figure P5.74

" ;;
5.1 Laws, Coefficients, and Basic Applications of Coulomb Friction 387

5.75

IJ.
Figure P5.75
Figure PS.8i
Figure P5.82

5.76
B

.~ -

I
IJ.=-
3

Figure P5.76

'Figure P5.83
, . Figure P5.84
5.77-5.80 Reverse the direction of P in Problems
5.73-5.76, respectively, and find the value of Prequired to
5.85 Repeat the preceding problem if the torque T is
start the beam moving to the right.
applied clockwise.
5.81 The two identical sticks are pinned together at A and
5.86 In Problem 5.84, if T= 100 N . m and ifthe slender
are in equilibrium as shown in Figure P5.81. What is the
lowest coefficient of friction (as a function of exl for which brake arm weighs 3 Nlcm, find the force B if motion is
impending,
this can take place?
5.87 The cable exerts 320 lb on the 400-lb weight as
5.82 The cylinder has mass M, including an attached hub
shown in Figure P5.87. The truck then moves slowly to the
around which a cord is wrapped. The cord, after passing
over a pulley, is attached to a mass m. (SeeFigure P5.82.1 right, while rotating the arm ..A counterclockwise so as to
keep the tension approximately constant. At what angle <p
a. Show that if the plane is smooth, the cylinder cannot will the weight begin to move? Will it slide, or will it tip?
be in equilibrium, regardless of the values of the
.•.. ,e. ,. .•...•.
, ..
.1•.•••. :... ,,",,"'"

parameters.
b. Find the minimum coefficient of friction Jl between
the cylinder and the plane for which the system is in
equilibrium. Find the relationship between M, m, <P,
R, and r for equilibrium.

5.83 In Figure P5.83 the cord is wrapped around the light


hub attached to the 130-lb cylinder. Find the minimum
coefficient of ,friction between the cylinder and the plane
for which the cylinder will be in equilibrium.

5.84.Find the braking force B if motion impends between


the brake and drum at A, with friction coefficient 0.4.
Neglect the weight of the brake and its thickness. (See
Figure P5.84.) The couple T has magnitude 80 N . m. Figure P5.87
388 Chapter 5 Friction

5.88 The coefficient of limiting static friction between the with a turning wheel, in order to stop it byfriction. While
100-lb block and the plane is 0.5. The force P is applied the wheel is not in equilibrium (indeed, it is being
horizontally as shown in Figure P5.88. Determine: decelerated), the arm is. Find the ratio of the force exerted
a. The magnitude of the force P for which the friction by the hydraulic cylinder to the normal force between
force vanishes the shoe and the wheel if the wheel rotates lal clockwise;
b. The force P required to st~rt the bl~ck moving up the (hI counterclockwise. Neglect the weights ofarm and shoe.
plane.

p
Figure PS.88

5.89 A 200-lb cylinder is in equilibrium in the position


sho~n in Figure P5.89. Find:
a. The friction force at A
Figure PS.91
b. The coefficient of friction if slipping impends at A in
the position shown.
5.92 In Figure P5.92 find the smallest couple M that will
cause the cylinder of weight Wand radius R to have
impending motion. The coefficient of friction for all
surfaces is /l.

5.93 The blocks .A, B, C, and;]) in Figure P5.93 weigh


100 Ib, 50 Ib, 75 Ib, and 40 lb, respectively. The friction
coefficients are 0.5 between A and B, 0.3 between Band C,
and 0.25 between C and the plane. Because the peg is
smooth, the tension in the cord is the same on both sides.
Figure PS.89
Force P is slowly increasedJrom zero. At what value of P
will the equilibrium be disturbed?

5.90 Determine the force P that will move the two


identical I O-kg'cylindrical.>toHers~upthe' inclined plan'e
shown in Figure P5.90. The friction coefficient is 0.3 at A,
B, and C.
Figure PS.92

,
<',: ;;',

,-
"
~' .

Figure PS.90
Figure PS.93

5.91 The hydraulic cylinder exerts a force to the right on


the brake arm at point B. This brings the shoe into contact
5.1 Laws, Coefficients, and Basic Applications of Coulomb Friction 389

390lb
FigureP5.97

t
6ft

-~

Figure P5.94

Figure P5.98

5.94 In Figure P5.94 find the mllllmum coefficient of


friction between the bar and the floor for which the bar can
be in equilibrium in the given position.

5.95 Repeat the preceding problem if the upper angle ex


is 60°.

5.96 The weightless members in Figure P5.96 are sup-


ported by pins C and Band by a rough surface at A.
Determine:
a. The minimum coefficient of friction at A to assure
Figure P5.99
equilibrium of the structure
b. The forces exerted on member BC by the pins at B
and C.

5.97 The cylinder in Figure P5.97 has a mass of 30 kg and


a radius of 0.6 meter. The'coefficients of static friction are
0.2 between the cylinder and the inclined plane, and 0.3
between the cylinder and the floor. Find the value 6f the
counterclockwise couple Mo that will cause the cylinder
to begin to rotate.
5.98 In Figure,P5':98a,forceP is applied to the ..rope.by a
man who erroneously thinks that if he pulls to the left,
the 200-lb wheel should roll to the left. If the friction
coefficient between the wheel and both the wall and floor
is 0.35, determine the force P that will cause the wheel to
move.
5.99 A person plugs in an electric appliance, pushing the
plug into the receptacle with a horizontal force F. (See
figure P5.99.1 The prongs initially touch the contacts as Figure P5.100

shown, and when the force F reaches 3 lb, the prongs slide
past them, completing the contact. When the sliding starts, 5.100 The coefficient of friction between the rod and the
what is the component of force on each contact per- triangular block is 110' and the plane is smooth. Show that
pendicular to the prong? Assume a friction coefficient of the only condition for the block to remain in equilibrium is
0.25. that tan 0 :s:: 110' (See Figure P5.! 00.)
390 Chapter 5 Friction

'.I::~---~-- .100Ib--.
F

Figure PS.104

Figure PS.101

Figure PS.106

I~x
Figure PS.10S
A
•.•. 1,:..;" ... ' z
.
Figure PS.102

5.101 The force P is applied to the cone of mass ill as


shown in Figure P5.1OI, and gradually increased in mag-
. nitude from zero. If the coefficient of friction between
the cone and the plane is Po, find the value of P at which x Figure PS.107
motion of the cone impends. Is it tipping, or slipping? Figure PS.108

5.102 The edg; AB of block ABC in Figure P5.I02 is a


parabola whose vertex is at A. Its equation is x2 = 634y. If at the contact points A and B are each equal to fl. Calculate
the coefficient of friction between the shaft and the block the smallest value of fl for which the cylinder can stay in
ABC is flo, find the largest value of x for which the system equilibrium with the thin rod maintaining its horizontal
can be in equilibrium. Neglect friction beneath the rollers position. (See Figure P5.I 05.)
and let ill be the,mass oLthe .shaft.plus thecplate.attached .•"" t.\ •• w~..•.•.•• ~""" .."__ •. .•••• Ii. .

above it. 5.106 In Figure P5.I06 find the minimum coefficient of


friction between the small block and the wall for which
5.103 In the preceding problem, let a force P be applied to equilibrium of the system is possible.
the left on face BC oEblock ABC. Neglecting friction in the
5.107 The 200-lb block rests between the smooth Vertical
vertical slot in which the shaft slides, find the value of P, in
termS of x, for which motion of the shaft will be impending guides as shown in Figure P5.I 07. It is supported by the 50-
Ib rod. The coefficients of s'tatic friction between the rod
to the left.
and the block at A, and the block and the ground at B, are
5.104 Show that the bar B in Figure P5.I04 will not slip each 0.5. Determine the maximum angle ()for equilibrium.
to the right, regardless of the size of P, if the friction
5.108 The uniform bar of mass 50 kg is in equilibrium.
coefficient fl is greater than or equal to cot e. The weight of
The wall is smooth and the floor is rough. (See Figure
K!is W.
P5.108.) Find the reactions' at A and B, and the smallest
5.105 The uniform rod rests atop a uniform circular coefficient of friction between the bar and the floor for
cylinder of the same weight W. The coefficients of friction which the equilibrium is possible.
5.1 Laws, Coefficients, and Basic Applications of Coulomb Friction 391

5.109 In the preceding problem, suppose the stick is 5.115 In the preceding problem, suppose the force P is
sharpened at A and that this upper end is in the corner applied at height y instead of 4 ft. Find the smallest value of
(touching the 2 axis). (See Figure PS.109.) y for which the ladder will not slide upward at A no matter
how large the force P. Hint: Write the three equilibrium
a. Show that the reaction from the corner is in the plane
equations, eliminate the normal forces, and examine
containing the triangle ADB, ..
carefully the remaining equation.
b. Find, in terms of x, y, and 2, the minimum coefficient
of friction between the bar and the floor for which 5.116 The rod in Figure PS.II6 has a mass of 3 slugs. It
equilibrium is possible. rests on a rough floor at A and a smooth, fixed half-cylinder
at B. Find:
5.110 The block weighs 200 lb and rests on the 300-lb
wheel with a friction coefficient between them of 0.4. (See a. The reactions exerted on the rod at A and B
Figure PS.IIO.1 If the friction coefficient between the b. The minimum coefficient of friction for which the
wheel and the plane is 0.3, find the lowest value of T for rod will not slip at A.
which the wheel will move. How will it start to move?

5.111 Repeat the preceding problem if the coefficient of


friction between the wheel and the plane is (al0.4; (b)0.9.
i
I
5.112 Repeat Problem 5.110 if the cord comes off the top
; of the inner radius instead of the bottom.
r
r
<

5.113 The frame in Figure P5.113 consists of the bar AD,


the curved bar DC (quarter circle), and the pulley. De-
t .' termine whether the frame is in equilibrium without
i the box. If not, find the minimum weight of the box for
A
equilibrium. Assume that all the forces exerted on DC, by
the floor and by the box, act at the point C. Also find the
I
I force exerted on AD by the pin at D, for your equilibrium Figure PS.113
I, case..
i
5.114 The ladder AB in Figure PS.II4 is 30 ft long and
weighs 60 lb. If the.coefficient of friction is 0.5 between the
ladder and both the floor and the wall, find the force P
required to start the ladder moving up at A.

)
y

Figure PS.109 Figure PS.110 Figure. PS.116


392 Chapter 5 Friction

Figure PS.122

Figure PS.117.

Figure PS.120

* 5.117 The plank rests on the floor atA and on the smooth,
fixed block at G.(See Figure P5.11 7.j If the coefficient of
friction between the plank and the floor at A is 0.3, find the
range of heights H of the block for which the plank won't
slip on the floor.
Figure PS.123
* 5.118 In Problem 5.117, let the coefficient of friction be
0.3 at both surfaces of contact. Find the lowest height of
the block for which the plank will slip. x

5.119 Solve Example 5.7 if the cord to which W is


attached comes off the right side of the hub of C.

5.120 The tongs in Figure P5.120 are holding the light


cylinder in equilibrium. What is the minimum coefficient
of friction between the cylinder and the tongs such that the
cylinder can't slip upward regardless of the value of P?

5;121 In the preceding problem, find the force exerted by


y
the pin atB onto,member,ABGwhen,P,,= .•lS,.lb. Assume
a friction coefficient large enough to prevent slip. Figure PS.124

5.122 The 20-N half-cylinder is in equilibrium on the


plane, with a cord between the center of the right edge and cylindrical cavity under the action of the couple of moment
the plane as shown in Figure P5. 122. If the cylinder is on T (see Figure P5.1231. Also find the required minimum
the verge of slipping with () = 30° and cP = 20°, find the coefficient of friction J1. for which this equilibrium can
tension in the cord and the coefficient of friction between exist. It is given that WR > T.
the cylinder and the plane.
* 5.124 In Figure P5.124 find the smallest coefficient of
5.123 Find the angle cP(interms of T, Iv, and R), for which friction for which the thin rod ABC will not slip on the
the cylinder of weight W can be in equilibrium in the cylindrical surface.

* Asterisks identify the more difficult problems.


5.1 Laws, Coefficients, and Basic Applications of Coulomb Friction 393

** 5.125 The friction coefficient between the stick and the between any sphere and the ground. The three lower
wall is /1. The stick (mass ml, connected to the floor at A spheres are as shown in the lower figure, and the upper
with a ball-joint, can clearly rest on the wall at P in sphere rests on the top. What is the minimum value of /1
equilibrium. Find the largest angle between the stick and for which the spheres can remain in equilibrium? Hint:
line AP for which the stick can be in equilibrium. (See Use symmetry.
Figure P5.I25.1 Assume that the friction force at Q acts
in a direction so as to oppose movement of that end of * 5.129 Two small balls of equal masses m are in equilib-
the stick. rium on a cylinder as shown in Figure PS.I29, connected
by a light string of length I.SR. The balls are on the verge
5.126 The block of mass m is at rest on an inclined plane of slipping clockwise; their friction coefficient with the
as shown in Figure P5.I26. The coefficient of friction cylinder is 0.3 for each. Find the angles 81 and 82, the
between the block and the plane is /1, which is therefore ;::: (constant) tension in the string, and the normal forces N1
tan 4J. If a horizontal force H is applied parallel to the and N2, respectively, exerted on the left and right masses.
incline, find the largest value of H for which the block will
not slip. 5.130 Two identical uniform heavy rods AB and BD, each
f of weight W, are freely pivoted at B and stand in a vertical

I * 5.127 Repeat the preceding problem if H is applied at a 45° plane. (See Figure PS.130). End A rests on a rough

I' angle downward from its previous horizontal position (but


. still parallel to the incline). (See Figure PS. 127.)

* 5.128 In Figure P5.I28 the coefficient of friction /1 be-


horizontal ground, to whichD is pinned. In terms of !x, find
the minimum friction coefficient between AB and the
ground for which equilibrium can exist.

I
tween any two of the four identical spheres is the same as

I
i
! l

I Figure P5.127
1
!f
I
y

FigureP5.125
Figure P5.129

,,
!
\ .

A D

Figure P5.126 FigureP5.130 Figure P5.128

t.~
~_.-
394 Chapter 5 Friction

*5.131 In the preceding problem, let rx = 30° and j1 = 0.4.


Let a farce P be applied dawnward at B and turned dawn-

w"d ,lowly tow"d BD 1.0 1. Find the


slipping 'Occurs at A when () ~ 30°.
,,10, of P ;j

* 5.132 The pinned, identical bars in Figure P5.132 rest


symmetrically on the cylinder. What is the range 'Ofangles Figure PS.133
between the bars far equilibrium if L = 2R and the
caefficient 'Offriction is j1 = 0.2?
Figure PS.132
* 5.133 The frictian caefficient between the bar and the
wall is 0.5. The unifarm bar has length L = 21, where I is
the distance 'Ofthe small, fixed pulley fram the wall as
shawn in Figure P5.133. Determine the smallest angle f3
for which the bar can be in equilibrium.

5.134 In the preceding problem, find the largest angle f3 Figure PS.13S
far which the bar can be in equilibrium.

*5.135 In Figure P5.135:


a. Shaw that if the system in the figure is in equilibrium,
then ml ;::::m2/2.

Figure PS.136
b. If the frictian caefficient between the bars and the
plane is j1, find the minimum value 'Ofj1 far equilib-
rium of the system, as a functian 'Of(), m I' and m2'

5.136 This is nat a "friction problem" but an equilibrium


exercise based an Example 5.10. The free-bady diagram in
Figure P5. 136 of the pipe wrench plus pipe shaws the
L
necessary resultant 'Ofthe reactians 'Onto the pipe from its
fitting. Nate the free-bady diagram of the pipe alone, which
includes tT' NT' tB' and NB in additian ta the force and
couples in the figure. From Equatians (2), (61, and (Ill of
Example 5.10, write tT' tB, and NB in terms 'OfNT' Then
from equilibrium of forces an the free-body diagram of
the pipe, shaw that NT = P(D + h)(h + L)/lbDI. Finally,
check this result by summing moments abaut any point 'Of
p the pipe and 'Obtaining zera.
5.2 Special Applications of Coulomb Friction 395

~5.2 Special Applications of Coulomb Friction


In this section, we are going to see how Coulomb friction is used to great
advantage in four special ways: wedges, belts, screws, a.qg disks.

Wedges
Wedges are simple machines in which large normal and friction forces are
developed between two bodies by means of one (B1) being wedged be-
r• . tween the other (B2) and a fixed surface :J. This is done in order to effect
1 a (usually) small change in the position of B2, as in the following two
j.
i examples.
I~
i

I
J
,,
i
E x A M p L E 5.11••........
I Find the force P for which the 200-lb block 1.52 will be on the verge of sliding

I
i'

f
upward. The wedge 1.51 weighs 50 lb.

!"

J.L = 0.2
"
!
\'

SOLUTION
In order for block 1.52 to be at the point of impending slipping, block 1.51 must also be
at the point of slipping at both of its surfaces of contact with 1.52 and~. Sometimes,
a~ was see~ in previous examples of Section 5. I; two or more cases of slipping are
possible in a problem. Normally in wedge problems, slip occurs at all surfaces if it
occurs at all.
The free-body diagrams and resulting equilibrium equations, with each
friction force being maximum, are as follows:
For 1.52:

0.3 N2
r.Fx = N1 - :0 cos 20° - N2 sin 20° = a

0.2 N1 0.3 N2
r.Fy = -200 - p; -:0 sin 20° + N2 cos 20° = a

(Continued)
0.3 lV2 0.4 lV3
p LFx = lV2 sin 20° + Pz cos 20°.+ 16 - p = 0

-I.-fa
Na .
LFy = N3 -
0.3N2
lV2 cos 20° + Pz.sin 20° - SO

(Note that here we have not used any moment equations,~as they would merely
= 0

serve to locate the lines of action of the resultants of the normal forces; we assume
the blocks to be large enough that tipping will not: occur.)
The solution to the above four equations in the unknowns lVI' lV2, lV3, and
P is*

lVl = 175lb

lV2 = 281lb

lV3 = 285 lb

. P = 289lb

Note that all three normal forces have to be. positive in a problem such as
this, which serves as a partial check on the algebra. Also, on the system of 1]1 plus
1]2' note that (see the free-body diagram below):

== 175 + 0.4(2851 - 289

= 0 ./
+t I.Fy = 0 = lV3 - II - 200 - 50
= 285 - 0.2(175) - 250

== 0 ./

'\."

which are further checks. In the above two equations, note that 12 and lV2 are
internal forces within the system that do not therefore appear in the "overall"
equilibrium equations. Only when 1]1 and 1]2 are separated do 12 and lV2 become
external forces, and thus appear on the free-body diagrams.

*Note that if P is given and motion is not impending or occurring, the problem of
finding the three f's and the three N's is statically indeterminate.

396
E x A M p L E 5.12~
A wedge is being forced into a log by a maul in a~i effort to split the log into
firewood. What is the minimum coefficient of friction between the log and the
wedge for which the wedge will not pop.out of the log~hen it is not incontact
with the maul?

SOLUTION
Note that very large normal forces N are developed on the faces of the wedge.
Without friction, when P = 0 the vertical components of the N's could pop the
wedge upward because these components would, ordinarily be considerably
greater than the weight (wI of the wedge. (See the two diagrams.) The friction
opposes the entry of the wedge during the striking of it by the maul. But then,
when the force P is absent, the forces f change direction as shown and try to keep
the wedge within the log. (Friction is a peacemaker; it always opposes the relative
motion, or tendency for relative motion, of the contacting points of the bodies.)

N N N N

Only the vertical equilibrium equation is of value in this problem. It is:

r.F = 0 = 2N sin f3 - 2f cos f3 - w


y
111
If the wedge is on the verge of popping out, then f = JlN:

_", ,_ N(2 sin f3 .-:- 2Jl cos f3) - w = 0


If, now, the weight is neglected (it is typically much smaller than the
wedging forces in this application), we obtain

Jl = tan f3 (2)

Thus any friction coefficient ;:::tan f3 will insure that the wedge. is "self-
locking" -that is, that it will not move in the absence of the driving force P. *
(Continued)

*One of the authors has split hundreds of logs and has never seen a wedge pop out of
one, though he has heard of it happening. Wedges have half angles (P) of about 6°, so that Jl
between the wood and wedge need only be about 0.1 fort he wedge to stay in place.

L.......
Another slightly different approach to this problem, which makes use of the
friction inequality, is to use Equation (I) when w is small to obtain

f = N tan fJ

and then from 0 :s: f ::; fMAX = j1N, we get

N tan fJ ::; j1N = j1 ~ tan f3 ',~

for equilibrium.

Flexible Flat Belts and V.Belts


Belts are used to turn pulleys, sheaves, and drums in countless applica-
tions. For example, a fan belt turns the alternator shaft beneath the hood
of an automobile. A string on pulleys allows you to tune to different
stations on an old radio. A flat belt surrounds the rotating parts of a
clothes dryer and makes it turn when a motor shaft drives the belt.
In this section we shall derive the relationship between the tensions
in the two sides of a belt wrapped around a circular drum or restrained
pulley when slipping is impending.
Figure 5.5 As shown in Figure 5.5, the angle between the entering and leaving
points PI and P2 is traditionally called p. We isolate an element of the belt
at the angle (J, somewhere between a and p, and show the forces acting on
it at the left. The belt is assumed to be perfectly flexible, which means it
has no bending stiffness. Thus no moments (and in turn no shear forces)
appear on the free-body diagram. For the element, the equilibrium
equations in the normal (n) and tangential (t) directions are
~(J ~(J
'LFn = T sin T + (T + ~T) sin T - ~N = a (5.6)

~(J ~(J
'LFr=(T + ~T) cosT - Tcas T - ~f = 0 (5.7)

Dividing Equations (5.6) and (5.71by ~(J yields

(5.8)

(5.9)

398
r',
I 5.2 Special Applications of Coulomb Friction 399
i
r
I'
I
Taking the limits of these two equations as !1e .....•a gives
.;...

dN
T=- (5.10)
.,:i
de
and
dT df
(5.11)
de = de
where we have used:

.r' (¥)l1;
l:~, (~) ~
sin lim [cos
M-O
(!1e)J
2
= 1; lim (!1T)
~8-0
= a

We are interested in the case when the belt is at the point of slipping, for
which f = JiN. Thus from Equation (5.11):
1 dT dN
(5.12)
Ji de de
and Equations (5.10) and (5.12) give
1 dT
T de :=: Ji (5.13)

from which, by integrating, we find


In T = Ji8 + C1 (5.141
The constant can be evaluated to be In Ts because the tension is Ts when
(.
e = O. Therefore,

T
In - = Jie (5.15)
Ts

And the tension at any angle e (necessarily in radians!) is given by


y:.= Tsel'8 (5.161
{3=11"
The maximum T(called TL) is seen to be, when e= /3,
TL = Tsel'P (5.17)
The ratio of large to small tensions (leaving to entering the contact
surface) is therefore an exponential function"of the product of the coef-
T, 2.57T, ficient of friction and the angle of wrap /3. This can build up dramat-
ically. For example, if a r~pe passes over a tree limb and Ji = 0.3 between
the two materials, then for /3 = n,
TL = T,eO.37t = 2.57T.
But with a wrap and a half,
TL = TseO.3(37t) = 16.9Ts
T, an increase of about 5.6 times!
~ .
400 Chapter 5 Friction

As another example, just a few wraps of a guitar or banjo string


around its peg enables it to be tightened to large tensions without the
string being securely tied or clamped at all. (See Figure 5.6.) The friction
buildup, even on such a relatively smooth shaft, is sufficiently great to
prevent. slippage."

Figure 5.6

20 newtons E x A M p L E
Determine the minimum weight of block ..A for which block B will have im-
pending motion down the plane.

SOLUTION

We note that tan 4> = 152 = 0.417 < 0.6 = Ji, so that B will not begin to slide
on its own. The angle of wrap is

B = 900 - tan-1(5/121 = 67.40 = 1.18 rad


Thus the tension r. between the fixed drum and B is obtained from

20 newtons The free-body diagram of B is shown at th~ left. The equilibrium equa- I'

tions give:

I.F = 0 = N -20(22).=> N = 18.5 newtons


y 13

/ I.Fx = 0 = Ts + 20(53) - 0.6(18.51

from which
W
Ts = 3.41 newtons = ~
1.60
W.A = 5.46 newtons
This is the weight of ..A for which slipping ofB will be impending.
E A Fit p L E

The oil-filter wrench consists of a handle and a thin, flexible band that fits over
the cylindrical filter. To remove the filter, force P is applied as shown in the
figure. The tensions in the band produce friction forces around the filter whose
resultant is a force together with a counterclockwise ~ouple which lossens it. For a
given filter radius R, friction coefficient /1, and wrap angle 13 (211: - el, there is a
minimum handle length h above which the band will not slip on the filter. Find
h = h(R, /1, 13). Then for R = 2 in. and /1 = 0.3, find hmin for e = 60°, 45°, and
30°, and note that practical wrench handles are all longer than this.

t
I
I
,I

l •.
,
SOLUTION
Taking moments about the point E of the handle,(using its free-body diagram
at the left), we eliminate P and therefore can relate TL and Ts immediately:

rME = 0 = T L
cos e[h - (~cos e - R)J - T,h
E ( 11
Tsh
TL=-------
(h +. R) cose - R
~h--
This gives the relationship between the large and small tensions at the ends of
p T,
the band. Now we know that if TL never reaches T,ePfJ, the band will never slip on
I-R- the filter. Therefore,

T,h fJ
R/cos () = R/cos (3 T - -------< T,eP (2)
L - (h + R) cos e - R

or

h < epfJ[(h + R) cos e- R) (3)

The denominator in Equation (2) is positive for a practical wrench so that the
sense in inequality (3)is unchanged. Also, we note that the cosines of e and 13 are
(Continued)

.4n!l .__ --
i
tL ..
identical because f3 = 2n - e so that, rearranging inequality (3),

eP/l(R - R cos f3) < h(ep/l cos f3 - 1)

If ep/l cos f3 > I, * we obtain


h ep/l(l - cos f3)
->-- .
R ep/l cos f3 - I

as our final equation. For J1 = 0.3 and R = 2 in., we then compute h for the three
required values of e. The results are shown in the table.

() (deg) P (rad) h (in.) Sketch of Wrench

60° 5.24 3.42

45° 5.50 1.14

5.76 0.389
:I(,
,

We see that even at e


= 60°, the average handle is longer than the
minimum h calculated for no slip. This is to accommodate the hand width of .an
average personi note that h is the dimension to the resultant of the force of the
hand.

If the belt in our derivation is a V-belt instead of a flat belt, then the
relationship between TL and T. changes and much larger tensions are
possible. Suppose the angle of the "V" is 2<5as shown in Figure 5.7: It is

*Which is usually the case. For example, if () < 45° IP > 315°), it is true for any
J1 > 0.063.

402
5.2 Special Applications of Coulomb Friction 403

seen that in contrast to our previous derivation, there is now no normal


force beneath the belt at all. Instead, all the normal forces are exerted on
the slanted faces on the sides of the belt as seen in Figure 5.8.
The vertical component of the total increment li.N of normal force
acting on the element oLFigure.5.5~is.seen.now in. Figure 5.8 to be .._.__...
T,
replaced, for a V-belt, by (2 li.N' sin <5). Furthermore, it follows that the
friction increment li.f will be 2J1 /1N', which is. the total friction
developed on both faces of the element of the V-belt. Substituting these
new values of li.N and li.f into Equations (5.6) and (5.7) leads to the
following revised relation between the larg~ and small tensions for a
V-belt:
Figure 5.7
(5.18)

Typical groove angle (2<5) values range from 30° to 38°. For a leather
belt on a fixed iron pulley, J1 :::::: 0.26. With an angle of wrap of 180° and
with 2<5 = 30°, the value of TLITs is
e(O.26)(",/sin 15' = 23.5
Figure 5.8

as compared to a value for the flat belt that is less than one-tenth this
amount:
e(O.26)" = 2.26

Screws
F
We shall consider here only the square-threaded screw, which is used
with more efficiency than the V-thread in transmitting power or in
causing motion.
Suppose that a load F is to be raised by applying a moment M to a
square-threaded jackscrew as shown in Figure 5.9.
We consider the forces acting on the bottom surfaces of an elemental
segment S of the screw thread in Figure 5. 10.

Figure 5.9

, ,

Figure 5.10

We emphasize that Figure 5.10 is not a free-body diagram of the


segment S because it only shows the forces on one "face" of S. However,
the accumulation of these forces, and the load F and moment Mo, are the
404 Chapter 5 Friction

dN = P x Differential area
=p(~)wacos

dt = s x Differential area
=s(~)wcos a

Figure 5.11

only external forces acting on the overall screw. Thus they are the only
forces that are important in expressing its equilibrium equations.
In what follows we shall assume that the pressure p and the shearing
(frictional I stress s are constants. These stresses are exerted by the jack
threads onto the screw threads, over whatever number of threads are in
contact. On the elemental segment, the resultant normal and friction
forces respectively caused by the stresses p and s are simply the stresses
times the area beneath s (see Figure 5.11):
dN = pdA df = sdA (5.191
If we exert the minimum momentthat will turn the screw, then we have
impending slipping and
df = Jl dN

Summing the vertical components of the external forces on the


entire screw then gives

. +t 'LFy =.0 =, ~F'+ J cos (XdN -, f sin (Xdf


contacting contacting
threads threads

or

F = f (cos (X - Jl sin (X) dN


contacting
threads

oe prw
= (cos (X - Jl sin (X) -- dO
fo cos (X

F = rw(l - Jl tan (Xl f: e

p dO (5.20)
5.2 Special Applications of Coulomb Friction 405

where 0e is the total angle of contact between the threads of the screw and
jack. For example, if 5! threads are in contact, then 0 = 11n rad.
Equation (5.20) then gives the average pressure p in terms of the
load:
F
Pave = rwOe( 1 - J1. tan ex) (5.21)

We also have to ensure that the total of all the differential moments
caused by df and dN equilibrate the applied moment Mo. Summing
moments about the axis z of the screw,

W (LM)z = 0 = Mo - f rdNsinex - f rdfcosex


contacting contacting
threads threads

or

Mo = fOC r sin ex(prw dO) + fOc rcos ex(J1.prw dO)


o cos ex 0 cos ex
Oc

Mo = r2wltan ex + J1.)

Using Equation (5.20) to eliminate the pressure,


f 0 P dO (5.22)

Fr(J1.
Mo=-----
+ tan ex)
(5.23)
1- J1. tan ex
This is the moment required to begin to move the load upward.
If Mo is now removed, let us determine the condition under which
the screw will unwind itself. In this case, the motion is impending
downward, and in Figure 5.11 the friction direction would have to be
reversed. This reversal can be accomplish~d in Equation (5.20) by
changing the sign of the "friction force term" (the term with J1.).
Therefore, for impending motion downward with?ut Mo, v:e get
OC

F = rw( I + J1. tan


f
ex) 0 p dO (5.24)

and

tan ex = J1.

Note that this time, the friction helps the normal force to support the
load. Therefore, if

tan ex > J1.

then df will exceed J1. dN and the screw will "unwind itself," moving
downward without any applied moment Mo. Such a screw is called an
"overhauling" screw.
406 Chapter 5 Friction
. ,r)
'.:h
If we have tan IY. < J.l-that is, IY. < tan -1 J.l-then, regardless of the ~.~ tj
size of the load F, the screw will not move downward. Such a screw is '.~.1'1
said to be "self-locking." In this case, a moment opposite in direction ~}~!:l
to that of Figure 5.9 is neede~ to lower the load and the screw. Equa- ;n;;;l
tlOn (5.24 ) applIes, together ..wJth"the.momentequation.oLequilibrium: -,.,--.,j'~i!l
t:~n

~Mo - r[s;o.- "Co,.{(J>ae)rwlco,.] ~ 0 [5.251 ~i


>,;~!iJ
Thus if we use Equation (5.24) to eliminate p, we obtain
.~.'~ "

(5.26) 'i~i"
;;,
,,'
:1'
~,"'~'
which has both a smaller numerator and a larger denominator than
Equation (5.23), indicating that not nearly as much moment is needed to
lower the load as to raise it. ~: '1
We note that the path of a screw is a helix. We can think of it as an Ii.
-1 ~'. .

!.' I
inclined plane spiraling around a cylinder. A property of the helix is that
the lead L (the amount the screw will advance or retract in one tum) is a
J
!
constant. The pitch P of the screw is the axial distance between similar
~l
points on successive threads of the screw. Thus, if we imagine the
inclined plane to be "unwound," we have (see Figure 5.12) tan IY. =
~L L/(2nr). For a single-threaded screw, we have L = P, but for a double*
21rr or triple or n.threaded screw, we would have L = nP. Equations (5.23)
Figure 5.12 and (5.26) may be rewritten in terms of the pitch P:

2nrJ.l + np) (5.27)


Mo (advancing against the loadl = Fr ( . ,
2nr - nPJ.l

Mo (retracting away from the load) = Fr(_2_nr_J.l_-_n_p) (5.28) ,


:.l
2nr + nPJ.l

In these formulas, we would use the static coefficient of friction J.ls 1


1
to compute the moments required to begin the turning. For very slow
rotations so that inertial effects are negligible, the kinetic coefficient J.lk
must be used, but the equilibrium equations still apply.
A final remark prior to some examples,is that screws, like wedges,
are statically indeterminate when motion is not impending. We can see
this from our analysis, because without the relationship dt = J.l dN, we
would not be able to relate Mo and F.

* A double-threaded screw is one that has two separate threads intertwined.


E x A M p L E
The scissors jack is supporting an automobile while a flat tire is being changed.
The screw of the jack has a single square thread with S-mm radius and 2-mm
•. pitch, with ah:ic.tion coefficient of 0.3. Th~owner.of the car not!ces that. ~e hils
jacked it up high enough to remove the flat tire, but not high enough to put on the
inflated spare. What force B will begin to raise the car higher?

SOLUTION
The two free-body diagrams and the accompanying equilibrium equations show
that the force F developed in the jackscrew is S 140 N:

'£Fy = 0 = 2F1 cos SSe - 3600


FI = 3140 N F = 5140 N

In other words, more than the 3600 N supported weight of part of the car must
be exerted by the jackscrew because of the angles involved.
(Continued)

407. __
Next, the moment that turns the jack is seen from the original figure to be
0.158 N . m, with B in newtons. To raise the car higher, Equation 15.27) gives

O.lsB = sI40(0.00sl(2n(0.00sI(0.3) + 1(0~002))


2nI0.00s) - 1(0.0021(0.3)
B = 63:5 N " •.. '.

When the tire has been changed and the owner is ready to lower the car, the
force B required then is obtained from Equation (5.28):

O.lsB = 5 140(0.00s)(2n(0.00sI(0.3) - 1(0.002))


2n(0.00s) + 1(0.0021(0.3)
B = 39.7 N

about 63% of the load that was required to lift the car.

E x A M p L E

A single-threaded (or single-pitch) bolt has squarethreads* with O.s-inch mean


radius and t'6-inch pitch, The bolt is being used to securely fasten two plates
which have kinetic coefficients of friction with the bolt of Ilk = 0.3. (a)Find the
aXIal force in the bolt if it is slowly turning under a 12 lb-ft torque. (bl If the
rotation stops, what torque is required to start the bolt turning again if the static
coefficient of friction is Ils = 0.4?

SOLUTION
(a)Equation (5.27) gives

F = Mo(2nr - nPIl) = 12(12) (2n(0.SI - II-h)0.3)


r 2nrll + nP 0.5 2n(0.sI0.3 + 1(/61'
t.:\- ~\,', ••
. ' '

F = 89slb

(bl To begin again, we would use Ils instead of Ilk' The same equation, but
this time solved for Mo, gives

Mo =
2nrll
Fr ( --'----
+ np) = (0.5
(895) -
)(2n(0.s)0.4 1
+ l' (/6))
2nr -nPIl " 12 2n(0.s) - 1(16)0.4

15.8 lb-ft, almost a 32% increase!

i
*Note that most fastenings are made with V-threaded screws, which provide slightly ~.
more friction. See, for example, Statics by Goodman and Warner, Wadsworth, Belmont,
CA, 1963, p. 325.
!
"

408
5.2 Special Applications of Coulomb Friction 409

Disk Friction
Disk friction is developed when two flat circular or annular surfaces come
I into contact, with each either turning or tending to turn relative to the
I.
other, about a central axis normal to their contact plane. Common
examples are disk brakes and'clutchplates."f--:-- -_.' . ~~-._.- .
I-
Suppose we wish to know how much moment resistance to the
I relative motion (or tendency toward it) is developed over the area of
I, contact by the friction. We shall assume that:
1 a. the contact is over an annular area having inner and outer radii Ri
and Ro;

I b.
c.
the friction coefficient and pressure p are constant over the area;
there is no lubrication between the surfaces.

Figure 5.13

With these assumptions, the normal force on a differential area r dr de


of one of the surfaces, say 5, of body B\, is (see Figure 5.13)
dN = pdA
so that the moment developed on B\ by all the elemental friction forces
I
at the point of slipping is
'f f21t fRO (5.29)
M =(f.1 f dN)r = f.1pr dA = 8=0 r=Ri~p~2 ~r de
. '-------'
df
Therefore, after integrating we find
f.1p(R6 - R[)2n (5.30)
M=~~~~-
. 3
I'

f Or, in terms of the compressing force P between the bodies, which is

P = pA = pn(R~ - Rf) (5.31)

we may eliminate the pressure p from Equation (5.30) and get


2f.1P R6 ---:
M = -- 2 2 = --
Rr
2f.1P (R~ + RoR, + Rf)
-------- (5.32)
3 Ro - Ri 3 Ro + Ri
410 Chapter 5 Friction

where it is seen that if R j = 0 (solid circular area of contact!,

(5.331

which is the moment that can be.transmitted by-friction across two'" ,_.,
-circular disks, It decreases somewhat as the surfaces wear, as one might
expect. Initially the material at larger r will wear faster because it has
greater speed. This will alter the pressure distribution. If instead of
constant pressure we reach a condition of constant rate of wear, it can be
shown that the moment is reduced by 25 % to !J1.PRo .

EX A M p L E 5.17~
A clutch is being tested. Its contacting surfaces are annular disks of inner and
outer radii two and three inches, respectively. If with an axial force of 500 lb the
clutch slips when the transmitted moment reaches 400 lb.in" what is the coef-
ficient of friction!

SOLUTION

We use Equation (5,32), which when solved for !J. is


3M Ro +R j

!J. = 2P R~ + RoR + Rfj

Therefore,

= 3(400) 3 + 2 = 0.316
!J. 2(500) 32 + 3(2) + 22

P R OBL .. - -_. EMS


-:,-',
-.... / Section - 5.2",
..

5.137 The bodies A and B in Figure P5. 137 have masses


50 kg and 75 kg, respectively. The various coefficients of
friction between pairs of surfaces are shown in the figure.
Find the force F that will cause A to begin to move up the Figure PS.139
wall.

5.138 In the preceding problem, let the force F be instead


applied downward on the top of A. Show that regardless of !J. = 0.3
the value of F, block B will not move to the right. Figure PS.137

5.139 In Figure P5,139 block A, mass 20 kg, rests on


block B, mass 30 kg. If the coefficient of friction is 0.25 for
all surfaces, find the force P necessary to raise block A.

;'J.
5.2 Special Applications of Coulomb Friction 411'

5.140 In Figure P5.140 find the smallest force F that will equilibrium is possible if force F is removed. If not, how
raise the body W, which weighs 1000 lb. The friction much force is needed? '
coefficients are 0.2 between JJ and B, and 0.15 between JJ
and W Neglect the weight of JJ. 5.144 Find the range of values of the weight of JJ that will
hold B in equilibrium. The friction coefficient is 0.45
'between the cableand.the.fixed drum." (See Figure P5.144.) .~,~•."

5.145 Homogeneous blocks JJ and B are connected by a


belt that passes over the fixed drum C. (See Figure P5.145.)
The force of 130 Ib acts upon block B as shown. The
coefficients of friction are 0.4 between drum and belt, and
0.2 between B and the plane. Block Bweighs ISO Ib and is
wide enough so that it will not tip. Find the range of
weights that JJ can have for equilibrium of the system.

Figure PS.140
5.146 A I 50-lb man stands on a 60-lb bar, to which a cable
is fixed at B. (See Figure P5.146.1 The cable passes over two
fixed pegs with coefficients of friction as indicated. Assume
5.141 In the preceding problem, willthe system remain in that the normal force exerted by the man on the bar acts
equilibrium if force F is removed? If not, how much force is downward through A and find the smallest force the man
needed? can exert on the cable if the bar is to remain horizontal.

5.142 In Figure P5.142 find the smallest force F that will 5.147 In the preceding problem, find the largest force the
I, cause the center C of the cylinder to move up the inclined man can exert on the cable if the bar is to remain
plane. The cylinder has a mass of 300 kg, and the wedge is horizontal.
~..
light.
I
I....•
'
~.. 5.143 In the preceding problem, determine whether
\

I
;
Figure PS.144

SOON
Figure PS.142

130 Ib

B
"~"'~~"¥.W",~~4':~_P j
1--4 ft~I---I-. 2 ft
---10 ft .1
Figure PS.14S Figure PS.146
412 Chapter 5 Friction

5.148 In Figure P5.148 a rope is wrapped around the fixed


drum 1i times. If the girl can pull with 40 lb, what max-
imum weight W can be held in equilibrium if J1 = 0.3
between rope and drum?

-'"5;149 In Figure' P5~149-the block'weiglis"40 lb. The .•••".


f~~ctioncoefficients are 0.5 between the rope and drum,
and 0.35 between the block and the plane. Find the lowest
force T that will cause the block to move.

FigureP5.148 5.150 In the clothes dryer, the pulley of motor m turns


counterclockwise as shown in Figure P5.150. If the dryer
drum :J) is turning at constant angular velocity, then the
moments of the large (lowei) and the small (upper) tensions
are in balance with the moments due to friction at the
support wheels (not shown). At the point of impending
slipping of the belt on the drum, find the ratio Oflarge to
small tensions with and without the idler pulley .J present.
'Let J1 = 0.3.

5.151 The homogeneous block A in Figure P5.151 is


connected by a belt to an,other homogeneous block, IJ.
Block A weighs 350 lb, and the coefficients offriction are
0.3 between A and. the horizontal plane and 0.2 between
Figure P5.149 the belt and the fixed drum C. Find the maximum weight B
can have without disturbing the equilibrium of the two
bodies.

5.152 In Figure P5 .152 body Bweighs 1200 lb and body C


weights 680 lb. The coefficients of friction J1 are 2/n
between rope and drum, 0.4 between A and B, and 0.3
between Band the plane. The force Pis 350 lb. Determine
the minimum weight of A that will prevent downward
motion of C. Negl~ct the possibility of tipping .

.Figure P5.151

Figure P5.150 Figure P5.152


~r
-----~~
5.2 Special Applications of Coulomb Friction 413

I":

Figure P5.154

11 = 2hr 11 = 2hr

p
Rope

~--18in. .\

-J~._--s
Figure P5.153

ft~--.I~ .•--'---Sft
2ft

, Peg
I
l
f p
Figure P5.156

Ij
Figure P5.155

5.153 The coefficient of friction between the drum and


the band is 0.4. (See Figure P5.153.) Find the friction
moment on the drum developed by the band brake, in pz

.
~ "
terms of force P, for impending slipping. The drum is held
fixed by a couple that is not shown .
5.154 Can the 100-1bpainter in Figure P5.l54 stand on Figure P5.157
1.3m
the left end of the scaffold and drink his lemonade with-
out the rope slipping and dumping him to the ground
below? torque Ta turns A clockwise, and the belt then turns A in
the same direction, "with the load (being rotated by Al
5.155 In Figure P5.'155, given that A weighs 1000 lb, B resisting the rotation with torque TR. The friction coeffi-
weighs 750 lb, and the coefficients of friction 11 are 0.3 cient between the belt and edch pulley is 0.25. If the largest
between A and B, 0.3 between Band the plane, and lin safe belt tension is 200 N, find:
between rope and drum, find the minimum force P that
a. The maximum torqueTa that can be applied without
will cause A to move.
slipping the belt on PI, .
'5.156 Repeat Problem 5.78 if the peg is not smooth but b. The maximum load torque TR to which the pulley A
has a friGtion coefficient with the rope of 0.4. (See fig- should be connected.
ure P5.l56.) The blocks A, B, C, and .2lweigh 100 lb, 50 lb, Note: Strictly speaking, this is a dynamics problem. How-
7S lb,'and 40 lb, respectively. Force P is slowly increased ever, for the pulleys turning at constant rates, the equilib-
from zero. At what value of P will the equilibrium be rium equations hold for each pulley. Conditions. in the
,
;"- disturbed? belt are as developed in this section provided the inertia of
the belt can be neglected, which really means that the
5.157 The flat belt in Figure P5.157 is transmitting power
from the small pulley A to the large pulley pz. The applied pulley speeds are not too large.

,
L..
414 Chapter 5 Friction

5.158 The two cables are on the verge of slipping in the equilibrium if he weighs 150 lb and if the coefficient of
indicated directions in Figure P5.158. Find the reactions friction is 0.3 between his shoes and the ground?
exerted by the wall onto the left end of the flexible beam,
5.160 The ferry boat in Figure P5 .160 is being docked, and
whose weight is to be neglected.
exerts 1000 lb on each of the ropes. How many wraps
5.159 The sandbag in Figure P5:159'weighs 50 lb: The - ..around,the posts (mooring bitts)'must each'dock'worker"-
coefficient of friction between the rope and the tree branch make in order to hold the beat in position with less than
is 0.4. What range of force can the man exert and maintain 40 lb? The friction coefficient is J1 = 0.3.

5.161 The band brake in Figure P5. 161 is designed to stop


the drum :J) from rotating. Alinkage provides the force Po
as shown. The friction coefficient between the band and
the drum is J1 == 0.3. Neglecting inertial effects, what is
the friction moment that can be developed by the brake, in
terms of Po and R?
6
J.I.=- 5.162 The man in Figure P5.162 is trying to pull the block
511"
over the round hut. He is able to exert 50 lb of force on the
rope; in the position shown he is unable to lift the block.

Figure PS.1S8

Figure PS.159
Figure P5.160'" .•.. - ..•.... - )
'I,

Figure P5.161 Figure PS.162


I
, .
j .~.
5.2 Special Applications of Coulomb Friction 415

Viewaa

Figure P5.163
Figure P5.164

Hebacks up until the angle e becomes 35°, and finds he is a. What are the forces exerted at Band D onto guide 9,
then barely able, with his 50 lb, to raise the block. The neglecting friction at Band D and the weight of the
friction coefficient between rope and hut is O.I. Neglecting vise? .
friction between the block and the hut, find the weight of b. What was the force at H on handle .JJ just before the
the block. tightening was completed?

5.163 The shade tree mechanic in Figure P5.163 is 5.165 In the preceding problem, after the handle was
removing some concrete blocks from beneath an engine, released (clamping force =200 lbl, it was decided to in-
preparatory to reinstalling it in his car. The engine weighs crease the clamping action.
700 lb, and is being held in equilibrium by a 150-lb a. What force must be applied at H to initiate further
apprentice. If the coefficient of friction between rope and compression ohhe board?
shade tree is 0.5, determine the minimum tension the ~,.,.."" .. b.o.What force must be~pplied to initiate a reduction
apprentice can "exert"without' the -engine';.moving-..down- (from 200 lb) of the clamping force?
ward. For this value of tension, find the normal and friction
forces exerted by the ground on the feet of the apprentice. 5.166 The power input while raising the load of Figure 5.9
at a constant angular speed Wo rad/sec of the screw is Mwo'
5.164 An old bench vise has a square-threaded screw S The "power output" is the product of F and the (vertical)
with a mean diameter of 1inch and a pitch of i inch. (See velocity of the load, or F[(rwol tan ex]. If the efficiency of the
Figure P5.164.) The screw turns within the guide 9, jackscrew is the ratio of the power output to the power
which is fixed to one jaw {Jd of the vise. The other jaw, input, find the efficiency in terms of ex and /l.
J2, is part of body .A, which is clamped to the bench as
I ; shown. Body .A contains the threaded sleeve C that en- 5.167 In the preceding problem, if /l = 0.3, find the pitch
gages the screw with coefficients of friction 0.25 (kinetic) angle giving maximum efficiency to the jackscrew. What is
I
I and 0.3 (static). this maximum efficiency?
When the vise is clamping something between its 5.168 In Example 5.16 find the torque required to retract
jaws, the guide 9 contacts .A at B and at D, and the head the bolt when it has stopped with the 895-lb tensile force
of the screw bears against 9 at A. If a board is being held
in it.
in the vise with a clamping force of 200 lb:

I...
~_ .
..__ .-

\
~
416 Chapter 5 Friction ~ '.

'i'
-t
.A ,
t",.

ISOOlb ISOOlb
~c~,~-
~ "."
eM' , . .' .".... . ..
~.,~
~
;

~ ~
~ :

L=~~~'JL=O.~~~'~'~
~.
Figure P5.169 {~
~"
:v

5.169 The single-threaded turnbuckle in Figure P5.169 is


being used to move the heavy crates slightly closer. When
the tension in the turnbuckle is at the point of causing
impending slipping of the crates, what moment Mo applied
to the turnbuckle will start them slipping? The turnbuckle
data are: pitches = 0.06 in., radii = 0.25 in., and static
friction coefficients = 0.35. Assume symmetry.

5.170 In the preceding problem, let the respective weights


of ..Aand Bbe 1200 lb and 700 lb, with friction coefficients
of 0.2 between ..A and the floor and 0.3 between Band
the floor. Find the couple being applied to the turnbuckle at
the point when one of the crates moves. FigureP5.17.3

5.171 Show that for the square-threaded jackscrew, the


moment M to turn the screw against the load F may be
expressed in terms of the angle of friction <P as M =
Fr tan(a + <Pl.

5.172 Prove the statement, made in the text just prior to


Example 5.17, that the moment transferred once the
condition of constant rate of wear is reached is reduced by
25% to /lPRo/2. Hint: Constant rate of wear means that
the rate of work done per unit area by friction is constant,
so that

ppv == constant = k] Figure P5.175

where v = velocity of the point in question. This velocity


magnitude is the radius times a constant angular speed. between the sandpaper and the surface, find the moment
Therefore, pr = constant = k2, and the constant k2 can the motor must exert to overcome the developed friction.
be found by P = Lp
dA. Then the moment M is balanced (See Figure P5.173.1 Assume the pressure to be constant
by over the surface.

M=..
f A
r(p dAl/l

Integrate and find the required moment.


i
== . 2n
9=0
f'. R
r=O
r k-2 /l r dr dO
r
5.174 In the preceding problem, assume the pressure
drops off linearly with radial distance from the center to a
value at the rim that is 60% of that at the center. Find the
new value of the moment, and compare.

5.173 If a person pushes a disk sander against a surface •. 5.175 Find the largest torque that may be applied to the
with a force of 10 lb, and if the friction coefficient is 0.4 shaft in Figure P5. 175 without it slipping in the hemispher-
5.2 Special Applications of Coulomb Friction 417

ical seat. Assume the pressure varies as ke t


~ 28) and
5.178 The collar bearing inFigure P5. I 78 supports the
200-lb thrust in the shaft. What is the value of the torque T'
the coefficient of friction is J1.. at which the shaft will begin to tum?

5.176 Repeat the preceding problem if the pressure dis- 5.179 Repeat the preceding problem for the new support
tribution is cosinusoidal: p= j{'cos 8. cOnditiohS on the flange shown in FigureP5: I 79:"

5.177 Find the maximum torque To that may be applied to 5.180 In Figure P5. 180 the shaft .A and diskB are welded
the shaft in Figure P5. I 77 without it slipping in the conical together to form one body that is free to rotate around its
seat. Assume a uniform pressure distribution. Check your horizontal axis in bearings not shown in the figure. Disk C,
ansWer .against Equation (5.30) for the case when d = 0 larger than B, is free to slide on .A but is normally held
and fJ= 90°. apart from B. When the separating forces are removed,
the tension in four equally spaced springs (two are shown)
cause C to contact B. The friction then stops the rotation.
The lugs attached toC slide in fixed slots and prevent
rotation of C during contact., If the coefficient of fricti~n
between Band Cis 0.3, fiAd the force in each spring
required to produce a disk friction moment of 5 N . m .

.if

-t
T8in.
Figure P5.177
6r 1
,"

Figure P5.179

Figure P5.17.8

Figure P5.180

'--
l
418 Chapter 5 Friction .
I
l
j
I
.J
COMPUTER PRO B L EMS/Chapter 5 I
5.181 Work Problem 5.129 with the help of a computer.
that these coefficients are equal, and plot the value of p
Hint: There will be two equilibrium equations for each
required for equilibrium of the stick as a function of O. Note
mass. The fifth equation in the.five unknowns is simply
from the equatiori that for values of o smaller tnan 45°, no
that 1.5R = R(O, + (2), After eliminating the tension, the ~
amount of friction can result in equilibrium.
two normal forces, and the angle O2, you should be left with
the following equation in the remaining unknown 01 : 5.183 Solve Problem 5.132 with the help of a computer.
Hint: In each case (impending slipping up and down), after
0.3 cos 01 + 0.3 cost 1.5 - (1) + sin(O, - 1.5) + sin 01 = O. eliminating the two normal forces from your equilibrium
Use the .computer to solve for 01, then backtrack through equations for one of the sticks, you will obtain a cubic
your equations to obtain O2, T, N1, and N2• equation in sin rx to solve, where rx is half the angle between
the sticks for equilibrium.
5.182 Read Problem 3.133. Note that if the bar is on the
verge of slipping at both surfaces of contact, then the 5.184 Solve Problem 5.133 with the help of a computer.
friction coefficients at these surfaces are p and a. Assume

Answers to Questions I Chapter 5


Q5.1 A plot of P versus time will be identical to I vs. t except during the short interval
~t during which the block accelerates up to constant speed. (Seethe figure.)
During that time P is larger than I, and their difference, P - I, produces the
acceleration of the block.

/P> f while block accelerates


J.L,N

Q5.2 If 0 = 200 sin 30° EB OAN - P, then the block would be on the verge of slipping
up the plane. This would result in the maximum P, for then P would be opposing
friction as well as gravity, instead of being aided by friction as in the example ..
Q5.3 Because when F reaches 103 N, we have impending slippage of B on .A; how-
ever, 118 N is required to simultaneously cause impending slippage of Con B.
Q5.4 For impending motion of B to the left and Cto t~eright, the force of Con B has
gone to zero.
Q5.5 No, it won't. The friction will reverse direction on both C arid S, and Equa-
tion (I) will then give a different (larger) N. Thus I will be larger and so will Ww.
Q5.6 The smallest D, using Equation (13), will give us the minimum Jl needed.
Q5.7 If w is larger than zero, then I must decrease for the equation to remain valid.
,-
I

Review Questions 419

Therefore, the maximum f need not be as great, and so tan fJ was a conservative
answer for p.
Q5.S Its bending stiffness, which has a slight effect on the angle of wrap. If the belt is
moving, its inertia also becomes a factor. .
Q5.9 Because if Bis to slide downward; thentne rope must be at thepoinfof slipping
counterclockwise on the ,drum_

Review Questions I Chapter 5


True or False?

1 The Coulomb law is appropriate for surfaces lubricated by grease.


2 The coefficient of friction for two surfaces is independent of the materials com-
_prising the surfaces.
3 The friction force-always has magnitude pN, unless it is zero.
4 Friction is never helpful.
5 Friction is a force that resists the tendency of two bodies to slide past each other.
6 If the normal force between two bodies is zero at the point of contact, there can be
no friction force developed there according to the Coulomb law.
7 It is not possible for an object to start to slide and tip at the same time.
S If body .A is rolling on body B, the friction force between .A and B has to be zero.
9 In working friction problems, we always know in advance whether or not there is
slipping between two surfaces.
10 Suppose we assume a certain state of impending slipping exists between two
bodies, and we then draw the force as pN on the free-body diagram of one of the
bodies. It is OK if we get the direction of the force wrong because the sign of the
answer will indicate it is really the other way_
11 The formula TL = Tsep(J relates the small and large end tensions in a flat belt
wrapped on a drum, at the point of slipping of the belt.
12 The formula in the preceding question applies only for angles fJ between 0 and
2rr radians.
13 For the same coefficient of friction and angle of wiap, the quotient Ttl Ts of the
large and small tensions, at the point of slipping, is smaller for a V-belt than for a
flat belt.
14 It is harder (takes more moment) to advance a double-threaded screw against a
load than a single-threaded screw, but it is easier (less moment needed) to retract
the double-threaded screw away from the load than the single-threaded screw.
15 The moment that can be transmitted by disk friction is independent of the radii of
the contacting surfaces.

Answers:1.F 2.F 3.F 4.F 5.T G.T 7.F 8.F 9.F 10.F 11.T 12.F 13.F
14.T 15. F

l,,~,
b-L.. _
Centroids and Mass!!Centers

6.1 Centroids of Lines, Areas, and Volumes/The Recurring Integral


6.2 The Method of Composite Parts

6.3 Center of Mass

6.4 The Theorems of Pappus

420
6,1 Centroids of Lines, Areas, and VolumesrThe Recurring Integral 421

~6.1 Centroids of Lines, Areas, and Volumesl


The Recurring Integral
In this section we shall learn the meaning of the word centroid, or, more
generally, the "center of Q," where the letterp denotes any scalar quan-
tity associated with a region of space. The most commonly occurring
quantities Q are arclength, area, volume, and mass; we shall examine
the first three of these in this section. The Center of mass will be ex-
amined in Section 6.3; however, it occurs s~ naturally in the develop-
,.
ment of dynamics that it is presented in additional detail in our dynamics
r .. volume.

;
v'
i
o Region If
'. .~ Figure 6.1

The "center of Q" is a point C whose location (see Figure 6.1) is


defined by the equation

- ;
Qroe = Lr dQ (6.1)

where Q itself is given by

Q = L dQ (6.2)

and t< denotes the region in space over which the integration is carried
out; it may be a one-, two-, or three-dimensional region. If, for example,
we are seeking a "center of volume," the region of integration is three-
dimensional and Equation (6.1) becomes
t,.

Vrae = II? r dV
where t< is the spatial region whose volume isof interest. Similarly, when
Q denotes an area the region of integration is two-dimensional, and wllen
Q denotes length of a line (either curved ,or straight), the region of
integration is one-dimensional. When, as in each of these cases, the
quantity Q is geometric-that is, arclength, area, or volume-then the
"center of Q" is more commonly known as the centroid (of,arclength,
area, or volumel. An important fact is that the centroid is unique; that is,
point C located by Equation (6.I) is independent of the choice of origin O.
The proof of this is left to the student as Problem 6.1.
To locate the centroid, we use the scalar orthogonal components of
the vectors in Equation (6.1). If we set up a rectangular coordinate system
422 Chapter 6 Centroids and Mass Centers

with origin at 0, then f = xi + YJ + zk and fOC = xi + )1J + zk where


x, )1, and z are the coordinates of the centroid C in the rectangular
coordinate system. Thus

Qx = f xdQ

Qy = f ydQ (6.3)

Qz = f zdQ

Scalar integrals* of the type in Equation (6.3) (and occasionally the


J
vector form f dQ) naturally arise again and' again in different areas of
mechanics as well as in other branches of 'mathematical physics and
engineering. The recurrence of these integrals is the principal reason for
identifying and studying centroids. Suppose, for example, that we
J
encounter and must evaluate the integral ydA, which naturally arises
in the analysis of stresses in solids. If we know,the location of the centroid
of the area in question, we need only multiply the total area by the y
coordinate of the centroid to obtain the value of the integral.

We now proceed to a series of examples in each of which we shall


locate the centroid of a line, an area, or a volume. It will be seen that
sometimes one or more of (x, )1, zl will be zerO by symmetry, and we will
not have to perform as many integrations in order to find the centroid.

E x A M p L E

Locate the centroid of the area of a rectangle of base B and height H.

H SOLUTION
Our use of the term "center" suggests that the centroid will be at x = BI2,
y = H/2. We are simply going to confirm that this is so.
)
B x (Continued)

* The integrals are often called" first moments" of Q because of their relationship to
certain properties of distributed force systems. See Section 2.9.
To find x shown in the figure, we have
y

-1r- dx
Ax= f xdA

Using the vertical strip of width dx and height H for our differential area dA, and
substituting A = BH, ...•. ,..

2
x
x = J:_X_H_dX = _H_ 2_ _1:_= ~
BH BH 2
x
L.
I-x--I We see that the centroid is, as expected, on a vertical line halfway across the
I'
! rectangle.
To obtain y, we shall use the horizontal strip shown in the figure so that
I dA = B dy this time:

Ay = f ydA

r
y
BHy = f: yBdy

l )
x
_ 2
y='BH=2
B-
y21H
0 H

and so the centroid is indeed in the middle of the rectangle.

,
i
I
I For readers familiar with double integrals at this point of their study, then
with dA = dx dy we have
I - (BHlY = JH'r B.
0 Jo
I
,
r
Ay = ydxdy

(. . ...:

.i. •

Thus

so that y = H/2 as expected. Note that the first integration (on xl generates
coverage of the horizontal strip, B by dy, that was used above in the single
integration solution.
E x A M p L E
y
Find the centroid ofthe semicircle shown in the figure.

o x

SOLUTION
We want to find the point C for which

Sloe = f Ids

or, for which

sx = f xds sy = fYdS sz = f z ds

where the arclength s is nR.


In this problem, by symmetry, x = O. This is because for every elements ds
to the right of the y axis with x coordinate "x," there is an element to the left of
the y axis with x coordinate" -x" land vice-versa). Thus the entire integral
vanishes. We never have to use symmetry, however. Letting ds = R dO, and
noting that x = R cos 0,

=0
or

x=o
The. z' coordinate .•of C is.also zero, this time because. the line is in the xy
plane, so that the zcoordinate of each "ds" is zero. Thus z ds = (0) ds = O. J J
Thus the only coordinate not immediately obvious is y. Noting that the y
coordinate of ds is R sin 0, we find, from sy = nRy = Y ds, J
r ~,', . (0=" yRdO
J: 2
OIl:
_',ll~
R sin 0 dO R2(_~OS

o
J_' -
y =
Jo,,"o
--n-R--
nR nR
which is slightly less than two-thirds of the way from 0 to the top of the semi-
2R

circle shown in the figure.

424
E x A M p L E

Find the centroid of the triangular area shown.in the figure.

w SOLUTION
The differential area dA may be taken to be Y dx, as suggested by the shaded strip
in the figure. .
1
.I-x-I
. • B .1
)
x Ax= f xdA

BH x = fB x(Ydx)
2 0

. H
But Y is a function of x, given by Y = - Xi therefore,
B

or

2
x =-B
3

Therefore the centroid of a right triangle is ~ of the distance from a vertex on


either end of the hypotenuse to the opposite side:

)
x.

We shall later see in Example 6.12 that the "~rule'.' stated above is in fact true for
any triangle.

A.ft2 .
y E x A M p L E

Find the centroid of the shaded area beneath the parabola shown in the figure.

SOLUTION
Using the vertical strip as shown, we have

B x
Ax= f xdA

y
= J: x(Ydxl

or

HB2
x Ax =--
4

But

B H
=
f o
lx2
B
dx

BH
A=-
3

3
so that x = - B.
4
For the y coordinate, we may still use the above area element Y dx, provided
we recognize that its own centroid is at Y/2 (see Example 6.1). We shall "add
up" the areas times the y coordinates ori:he 'c'eil'troids 6f the vertical ~trips:

(again, Y/2 is the centroid distance for "dA"l

Thus

Jr~Ydx
2 .
y=
BH/3

= --
3 fB 2
H x4 dx
__
4
2BH x=O B

(Continued)

.~
426 __ ._-""1

l,
rr""--------------------------------------------- j

So
3 H2 B5 3
Y = 2BH B4 5 == 10H
The student is encouraged to confirm this result either by double integration
or by single integration using a horizontal'stripdA.- -"' -..~....~.'--

{,
1-
E x A- M p L E

Find the location of the centroid of the area enclosed by the semicircle and the
(diametral) x axis as shown in the figure.

SOLUTION
Lr For the same reasons as in Example 6.2, x = z = O. To find y, we again use
~- .
Equation (6.1) where this time the quantity Q is areaj and we shall also use the
!l. horizontal strip, shown in Figure I, for our "dA." We get, then, with dA =
! 2.JR2 - y2 dy,
}
}

Ayc= f y dA

l' nR
2 fR
_y = y(2.JR2 - y2 dy)
Figure 1 2 0

2 3
= -R
3
• ~. • ',01' ~ -',<,.

And therefore

4R
y=-
3n
This centroid is lower than the centroid of the semicircle (see Example 6.2).

For the reader familiar with double integration, polar coordinates offer a nice
alternative to the solution above. This time, the differential area is seen in the
(Continued)

427
I~--~---'!!!I"-----"''''-;;-'l-=---::~-.,-'-'-'''-------------------------------------

y illustrations to be dA = r dr dO. We obtain

Ay = f ydA

a x
(n~2)y = f: f (rsin Ojr dr dO

r31R
f e dO
n
= - sin
030

3
R
=- (-cos OJIn
3 0

2R3
3

4R
y=_. (as before)
a 3n
~.'.
,
;~"
~,

E x A M p L E
Find the centroid of the volume of a hemisphere of radius R.

SOLUTION
By symmetry it is clear that x = y = O. To find Z, we shall use the volume }
element shown in the figure, which is .a disk having the differential volume
dV = nr2 dz. But we see from the ~haded triangl~ that

R2 = r2 + Z2

428
(Continued)

J
---!'1111------~...
f•.• 1r~_-.--.___------------------------------------------

so that as a function of z, the radius is expressible as


r=.jR2~z2

Therefore, because all points of the disk have equal z coordinates,

VZ =-f z dV =' Iznr


2dz .

= n f: 2 2
z(R - z 1 dz = n J: IR2z - Z3) dz

Because the volume Vis tltnR3), we obtain

1tR4/4 3
z=---=-R3
2nR /3 8

The centroid of the hemisphere may also be computed using the spherical
coordinates Ir, ({J, 8). For those familiar with multiple integration, this alternative
approach is presented next: ~

z dV

so that

(as before)

429
z E x A M p L E

Find the centroid of the conical volume shown in the figure.

-1
H

1 y

SOLUTION
As in the previous example we shall use a disk as the element of volume for
integration. The radius of the disk is (R/ HI z so that the element of volume is
n[(R/ HlzJZ dz. We take this opponunit)[ to derive the cone's volume:

v = f: n(~z)2dZ
=nR
2 .fH Z2 dz
2
H o.

nR2H
3

Noting next that x = y = 0 by symmetry, we proceed to compute z:

Vz = f z dV

.= f. H z(nR22 Z2) dz
•• '0 ... H , 'lor '.

2 2
nR H
4

Thus

nR2H2
4 3
z=---=-H
nR2H 4
3

Therefore the centroid of a conical volume is .on its axis, three"fourths of the
distance from the vertex to the base.
The reader familiar with multiple integration is encouraged to reproduce
this result using cylindrical coordinates.

430
6.1 Centroids of Lines, Areas, and Volumes/The Recurring Integral 431

y y

fix)
Equipollent
'2 >

'-----L---- x
~x,-I x

We close this section by reviewing some results from Section 2.9


about distributed forces. In that section, we saw that for a distributed line
loading of parallel forces, the resultant force F, equals the "area" beneath
the loading diagram (see the figure), with a line of action given by
~. Equation (2.26):

f: xf(x) dx .

x, = f: fix) dx
(6.4)

By comparing Equations (6.41and (6.II, we now see clearly what we were


only able to mention in Chapter 2-namely, that the force-alone
resultant acts through the centroid of the "area" beneath the loading
curve.
For pressure acting on a plane surface we found,' in Section 2.9, a
! '
force-alone resultant as indicated in the illustration:
t
t,

x
/ y

F, = f pIx, y) dA

with

F,x, = f xp(x, y)dA and FrY, = f yp(x, y) dA

Thus we see that the resultant force is, this time, the "volume" of the
region for which the loaded plane surface is the base and the altitude is
432 Chapter.6 Centroids and Mass Centers

the varying pressure pIx, yl. Furthermore, the resultant has a line of
action through the centroid of this volume.
A special case of the above with great practical importance arises
when the pressure is uniform. Then, we obtain

Fr = p f dA = pA

and

AXr = f xdA

AYr = f ydA

so that the resultant force has a line of action through the centroid of the
plane area on which the pressure acts.

E x A M p L E

Find the force-alone resultant of the distributed line loading shown in the figure,
and its line of action.

f(x)

701b/ft

SOLUTION
The resultant is:

F == Area beneath
r loading curve
'--v----
70 x216).
= ~16J70
2 ( or --
6 2 0

== 210 lb
(Continued)
This force acts downward (parallel to the given loading) at the centroid of the area
beneath the loading curve:
7 0
6 X
6,-.-.A--, 3
.° Xt.(X) dx 70 X .16)
{
6 3. °
210 210

=4ft

Therefore, we have found the following with the aid of centroids:

210 Ib

----.6 ft---.I
70lbfft
Equipollent
<2 >
LJ
1-4ft-

It is interesting to contrast this problem with Example 2.32, which


consisted of the six forces on the left-hand side of the figure, and which was
shown to be equipollent to the single force shown at the right-hand side. We see
that even though the six forces sum to the same 210 pounds, and even though the
forces' magnitudes are proportional to their distances from 0, the single force
resultants of the two systems are not the same because the lines of action differ.

60lb
50
40
30

O.
I.
10
t
20

~ 1
6ft
I j
.1
O.
1-4.33ft-1
1210 lb

PR 0 B L E MS I Section 6.1

6.1 Show that the centroid C of the area in Figure P6. 1 Figure P6.1
does not depend upon the choice of origin in the reference
frame. Hint: Consider the two centroids resulting from
Equation (6.1) with two separate origins 01 and O2

and relate r1 to r2• Show then that ro,c, = ro,cz' which


means that C; and C2 are the same point.

433
434 Chapter 6 Centroids and Mass Centers

y y

x
x

x
_a_I Figure P6.3

Figure P6.2 ----xl----


FigureP6.4
6.2 Find the centroid of the area bounded by the parabola
and the line x = a in Figure P6.2.
*6.7 y(ft)
6.3 Find the centroid of the circular arc shown in Fig-
ure P6.3. r = a cos 28 ft

6.4 Which of the following expressions may be used to Figure P6.7


find the centroid location 51of the area under the curve
y = Y(x)? (See Figure P6.4.)

f'
o xY dx. t f' o
xY dx
.
(a) f' Ydx
(b)

f' o Ydx 6.8 y(m)


0
5

f'
o xY dx t f' 0 y dx
2
4
(d)

f:'
(c)

f'
.0 y2 dx Ydx
Figure P6.8
3

6.5 In Problem 6.4, find 51if Y(x) = eX and Xl = 2.


In Problems 6.6-6.9 find the centroids of the shaded areas.

4 5 x(m)

10

6.9 y(ft)

Figure P6.9

o x(ft)
Figure P6.6 4 2

* Asterisks identify the more difficult problems.

---~
6.1 Centroids of Lines, Areas, and Volumes/The Recurring Integral 435
. )'

6.10 Find the y coordinate of the centroid of the shaded In Problems 6.16 and 6.17 find the x coordinates of
area shown in Figure P6.10. the centroids of the lines by integration. Hint: ds =
.J dx2 + dy2 = .J 1 + (y')2 dx.
y
6.16 y(m)
" -x'
y=-+6
..-/ 9

Figure P6.16

1--------'-6 in.- x
Figure P6.10 8 x(m)

6.11 In Problem 6.10, find the x coordinate of the 6.17 y(ft)


centroid.
2
InProblerns6.12 and 6.13 find the x coordinates of the
centroids of the shaded areas.
Figure P6.17
6~12 y 3em
I' 'I
. ~-U>~~ 2 x(ft)
I
6em

I
6.18 Find the centroid of the shaded area in Figure P6.18.

6.19 Find the centroid of the shaded area shown in Fig.


ureP6.19.
1--- 6 em -------I x

'Figure P6.12 y

6.13 y
Figure P6.18

y(mm)

~---9in.~,1 x

FigureP6.13

Figure. P6.19
6.14 Find the y coordinate of the centroid of the shaded
area in Problem 6.12.

6.1'5 Find the y coordinate of the centroid of the shaded


area in Problem 6.13.
436 Chapter 6 Centroids and Mass Centers

y(m) y

---I.
I 2
.\ x(m)
x

Figure P6.23
Figure P6.20
x
Figure P6.21
z

Figure P6.24

y(m)

x(m)
6.20 Determine the x coordinate of the centroid of the
Figure P6.25
shaded area in Figure P6.20. The equation of the curve is
y2 = x.

6.21 Find the centroid of the volume of the paraboloid of


r(;volution shown in Figure P6.21. and that the area is

*6.22 In Problem 6.21 find the centroid of the outer surface n .


~-bh.
area of the paraboloid. Hint: You'll need integral tables! n + 1
6.23 Show that the centroid of the area under the "nth 6.24 Find the centroid of the volume of the pyramid
degree parabola" in Figure P6.23 is at shown in Figure P6.24.

Ix
,
y)= (1?++ b" n.++ 'h).' -
2n
.1 .
1 J 2(n
1
2) J
6.25 Find the centroid of the volume of the tetrahedron
shown in Figure P6.25.

~6.2 The Method of Composite Parts


When two (or more) familiar areas * A I and A z are to be considered
together as a single area, we do not have to integrate over this combined
area in order to find the centroid. Fortunately, an approach called the
method of composite parts allows us to use the known areas and centroid
locations of the separate parts to establish the centroid of the combined
area (AI + Az. =' A).

*Or volumes or lines, as the case may be.


6.2 The Method of Composite Parts 437

The theory goes as follows, with areas At and A2 having respective


centroidal x coordinates xt and x2:

Ax = LXdA
..Butthe integral over I< may be taken separately over subregions I<t and 1<2
for which the areas are, respectively, A! and A2:

(At + A2)x = r x dA + Jr
J 1<, 1<2
x dA

Now separately invoking the basic centroid equation (6.3) over each
of Ar and A2, we obtain
(6.S)

and we see that all need for integration has been circumvented if we know
Atl A2, xt, and x2. Any number of areas may be combined in this way.
In the more general form (where the quantity whose centroid we
seek is any scalar Q), the steps to the similar vector expression are

Qroc = fr,dQ

L
= t, r dQ + t2 r dQ

r~
I,. where

I
We now consider a number of examples of the use of "composite parts."

y E x A M p L E 6.9~
-r 0).
Find the centroid of the three line segments connected as shown in the figure.

[ SOLUTION
Labeling the segments as shown, we have, using composite parts,
l--bl2 --1-- bl2--1 x
lh + b + hLY = Ltf! + L292 + L35'3

== h(~)+ b(O) + h(~)


(Continued)

I
j

!-
L
where the middle term vanishes because the y coordinate of the centroid of
element @ is zero:

so that
'h2
Y=2h+b

To illustrate the vanishing of "first moments" when measurements are


made from the centroid, consider the special case in this example for which
y b = h; then y = h/3. Now let us set up new axes with origin at the centroid as
shown in the figure.

Relative to these new axes, we see that the "first moment" is

C, C3
_! h/6
f yds = h(h/61 +h(-h/3) + h(h/61
,
C ..-
x =0
1 h/3 as, of course, must be the case since the y coordinate of the centroid in this system ~
c, ~~. t
I
1
~

E x A M p L E 6.10~
The circles in the figure represent cross sections of bolts, with the bolts holding
down a gear-box cover whose shape is the curve shown. It is important to locate
the centroid C of the bolt pattern because, fat example, the distance from C to
each bolt is used both to compute the stiffness of the overall pattern against
twisting and also to calculate the shear stress in each of the bolts. Letting each
bolt's cross-sectional area be A, find the centroid of the pattern of ten identical
bolts.
1
l
-J
-1
4 in.

i
1

1~
~.
j
I

y
I x

(Continued)

438
r.

SOLUTION
Using composite areas,

-
~x. I
x=-
10

and similarly,

while z is zero because all of the areas lie in the x-y plane.
Substituting (using the bolt-numbering system shown in the figure), and
moving counterclockwise from bolt CD at the origin,

x = (0 + 4 + 7 + 11 + 11 + 7 + 4 +0 - 4 - 4)/10

x = 3.60 in.

Continuing,

y = (0 +0 +4 +4 + 8 + 8 + 8+ 8 +5 + 1)/10

y =.4.60 in.
Therefore (x, y, zl = (3.60, 4.60, 0) in.

E x A M p L E 6 •.11~
0.5 in.
Find the centroid of the cross-sectional area (of a channel beam) shown in the
figure.

6 in. SOLUTION
The axis of symmetry (y = 3 in.) allows us to conclude by inspection that y =3
---,. 0.5 in.
in. We shall determine the x coordinate of C in two ways. First we note that the
area in question is the composite of three rectangles as shown in the figure on
0.5 in. page 440.
)
1--3.5 in.--I x (Continued)

439
[3(0.51]12) + [3(0.5)]12) + [6(0.5)]10.25)
x= 3(0.5) + 3(0.5) + 6(0.5)

6.75
= -- = 1.12 in.
6

An alternative approach is to note that the area of interest is obtained by


c, removing a 3 in. x 5 in. rectangle from a 3.5 in. x 6 in. rectangle as shown in the
figure below:

0.25 in. ~I~ .LJ


2 in. 1.5 in ..
y

W(whole) x .)1 (hole) x

Applying the method of composite parts to the whole (3.5 in. x 6 in.) rectangle,
we have

where A and x refer to the (net) area of interest. Rearranging,

[(6)(3.51 - 5(3)]x =[6(3.5)](1. l5) - [513m2)

or

x == (3.5)(1.75) - 5

= 6.12 - 5

= 1.12 in. (as before)

*Some like to see this as a direct application of the method of composite parts by
..~
thinking of the "hole" as a negative area; that is,
;.~.;l...•.
,'.L

440
'.,''d

y IE x A M p L IE
Show that the centroid of any triangle is on the line that is two-thirds the distance
from any vertex V to the opposite side and parallel to that side.

SOLUTION

x This example provides an opportunity to obtain a useful, general result without


using integration. Consider the actual shaded, scalene triangle to be made up of
the right triangle OVD less the smaller right triangle EVD. Then, using what we
have learned about the centroids .ofright triangles,

.or

(~BH)Y [~IB + = QIH J~- (~QH)~


BH2/6 H
(again)
Y = BHI2 ="3
This means that all three of the lines 11' 12, 13 below will intersect at the
centroid C of the triangle:

44'1
E x A M p L E
Use the method of composite areas and the result of Example 6,4 to find the
centroid of the shaded area.

SOLUTION

I.e, I
H.1----
JHf2 1
· ".~-fH 1
~~
~B .1.1+
A,=BH A =BH
p 3

The shaded area is obtained by subtracting the area beneath the parabola (p) from
the rectangle (I). Their areas and centroid locations are shown in the figures.
Therefore,

A = Ar - Ap
BH 2
= BH - -"-
3
= - BH-
3
and
'~. '••.••' .l. •...••.
--.•.. ,~~ jil •.b.
Arxr = AXt Apxp

(BH{~)=(~BH)X+ e:)(~B)
3
x=-B
8
The same procedure will yield the y coordinate of the centroid:

ArYr = Ay + ApYp

IBH{~) (~BH)Y + e:)C30H)


=

3
Y=SH

442
E x A M p L E

Locate the centroid of the volume of a hemispherical shell of inner radius Ri and
outer radius Ro'

SOLUTION
.From symmetry we recognize that x = y = O. The volume in question may be
viewed as a hemisphere of radius Ro out of which a hemisphere of radius Ri has
been removed. From Example 6.6, we know that the z coordinates of centroids for
these are 20 = iRo and 2j = iRj• Using the method of composite parts,

or

(23 3 23 3)- (23 3)(38 ) (23' 3)(38 )


- rrR
0
-.- rrR.
I
z = - rrR
0
- R
0
- - rrR.
I
--c- R.
I.

_ 3(R~ - Ril 3(R~ + RfliRo + R;l(~)


Z= ----~ ( I)
8(R~ - R~I 8(R~ + RoR + RfH~)
j

Note that if we set Rj = 0, we recover 2 = iRo'


A limiting case sometimes of Interest is that for which Ri ::::: Ro' where
the volume of interest is a thin shell. hi that circumstance we obtain from Equa-
tion (II

which is precisely the z coordinate of the centroid of the (curved) surface area of
the hemisphere. (The student is encouraged to verify this by direct calculation.)

443
444 Chapter 6 Centroids and Mass Centers

PRO B L EMS / Section 6.2

6.26 Find the centroid of the shaded area in Figure P6.26. 6.29 Find the centroid of the composite area shown in
Figure P6.29.

6.30 In Figure P6.30 find the centroid of the five circular


areas. Each area is 0.75 in.2
I in.
6.31 Find the centroid of the cross-sectional area of the
structural steel angle beam shown in Figure P6.31. Assume
a constant thickness of i in., and compare your answers
with the actual (considering rounded corners and fillets)
values of (x, y) = (1.03; 2.03) in.
I in.
6.32 In Figure P6.32 determine the height h of the "T"
3in. x (formed by two perpendicular lines as shown) in order that
FigureP6.26 Figure P6.27 the centroid be 2 em below the top.

6.33 Find the centroid of the center line of the bar


6.27 Find they coordinate of the centroid of the T-shaped consisting of three straight sections parallel to the (x, y, zl
area shown in Figure P6.27. axes as shown in Figure P6.33.
6.28 For the angle section shown in FigureP6.28, find
the coordinates of the centroid.

h
1 r
6 in.

j)
I----b----I
x 2 in.
1
t
1-1- ------1--1
V' )
x
Figure P6.30

2 in. 3 in. 3 in. 2 in.


Figure P6.28 Figure P6.29
z

-, 3cm

6'in.

1t ~iC b
Figure P6.33

t 4in.--1
Figure P6.31
I

Figure P6.32
a.2m
j

~
6.2 The Method of Composite Parts 445

y
1
6cm
3cm
t
H
~
-t ~ )
Scm x
t Figure P6.35

!4cm
Figu~eP6.36
x
1~1'6cm:I'4 6cm'\
Figure P6.34

Figure P6.38
l'. alO.
1
. 1.
3alO. i
l1i-in.
. O.3m

y
3cm
I 13:.L-

I. 3+in. .\
~
17.
-10.
32

t
)
x

l--o.6m--1 Figure P6.40

Figure P6.37 x
Figure P6.39

6.34 In Figure P6.34 locate the centroid of the shaded t2 in.


area.

~6.35 In Figure P6.35 show that the centroid of the trap- -1


ezoidal area has a y coordinate of ~ (aa : ~), indepen- 8 in.

dent of the angles (XI and (xz •

6.36 Find the centroid of the area shown in Figure P6.36.

6.37 Find the centroid of the area shown in Figure P6.37.


.

.-4 ino-"\"'-4 in.----"


I )
x

Figure P6.41
6.38 In Figure P6.38, Al and Az are two areas with
centroids CI and Cz separated by the distance f as shown.
Show that the centroid of the composite area (AI plus Az) 6.40 Find the centroid of the tee rail cross section shown
A in Figure P6.40. (We are neglecting the fact that the web
lies on line CICZ' and is a distance z ffrom CI.
Al + Az . 0-. w •• sides and the top of the head are actually segments of large
circles for this particular rail.)
6.39 Find the centroid of the three lines in Figure P6.39,
and compare its location with the centroid of the enclosed .6.41 Locate the centroid of the shaded area in Figure
area. ..P6.41.
446 Chapter 6 Centroids and Mass Centers

6.42 In Figure P6.42 locate the centroid of the homoge- 6.48 Find the x and y coordinates of the centroid of the
neous square plate with a triangular cut-out. shaded area in Figure P6.48.

6.49 The circular cutout shown in Figure P6.49 has an


y
area of 12 in.2 Find the centroid of the shaded area.

-TIft
-t
2ft
1
l (

pft )
3 in.
I
J--I-. -.,1--1
I ft
Figure P6.42
2 ft I ft
x

Figure P6.46
,
.~

j
i
6.43 Find the centroid of the shaded area in Figure P6.43.
y IDcm
37cm
f-i-I. 3D cm .1

x
Figure P6.47

Figure P6.43

6.44 Find the centroid of the shaded area in Figure P6.44.

-1
6 in. ___ Equilateral,
t
3 in.
side 6 cm
y
_1
1 x

1---1~6in.-1
2 in.
Figure P6.44 Figure P6.45 -r
6 in.

6.45 Find the centroid of the shaded area in Figure P6.45.


1
6.46 Find the centroid of the shaded area in Figure P6.46.
I. 12 in.
__~.Ii -

6.47 Find the centroid of the shaded area in Figure P6.4 7. Figure P6.49

\
"

l-
f
",

6.2 The Method 0.1Composite Parts 447

6.50 Find the centroid of the shaded area in Figure P6.50.

6.51 In Figure P6.51 find the centroid of the shaded


square, which has a circular and triangular cutout as
shown.
Figure P6.50
6.52 Find the centroid of the shaded area in Figure P6,52.
x
Diameter 6.53 Find the centroid of the shaded area in Figure P6.53.

6.54 Find the y coordinate of the centroid of the shaded


area in Figure P6.54.

6.55 In Figure P6.55 find 'the centroid of the shaded


triangular area with the semicircular. cutout shown.

6.56 In Figure P6.56 the holes in the trapezoidal area have


I-in. radii. Find the x coordinate of the centroid of the area.

6.57 Find the centroid of the shaded area in Figure P6.57.


12 in.

~:..

>
x

----12 in. I'igure P6.55


.F,igureP6.~1

>x

FigureP6.52
2in-.t-
Figure P6.56 3 in.l y

2inr
--1--7 i:'-~6in.-1

Figure P6.57

..Figure P6.53 Figure P6.54 x


448 Chapter 6 Centroids and Mass Centers

6;58 In Figure P6.58 determine the height h of the cutout


rectangle that will place the centroid C of the shaded area
at the position shown.

6.59 In Problem 6.27, to what value should the 6-cm


vertical dimension be changed in order that the new
centroid be on the line separating the two rectangular x
areas?

6.60 Locate the centroid of the shaded area shown in


Figure P6.60.
Figure P6.62

6.61 In Figure P6.61 find the centroid of the shaded area.

6.62 In Figure P6.62 find x for the shaded area, cut from a
circle.

6.63 In Figure P6.63 find the centroid of the shaded area.

6.64 Find the area and the centroid of the dotted area in
Figure P6.64.
Figure P6.63
1
6 in.

6.65 Find the centroid of the shaded area in Figure P6.64.


x
6.66 A circular plate with radius and unit thickness
I I

has five holes drilled in it as shown in Figure P6.66.


The centers of the holes lie on an arc of radius I2. The
spacing between the centers is 60°. Locate the centroid y(m)
of the volume of.the plate if II = 0.5 m, I2 = 0.3 m, and
I3 = O.lm. -

Figure P6.64

Figure P6.58

Figure P6.60

eJ "O.6m

O.lm
O.6m

y
-~
: O.3m O.3m
---'------,
O.3m
,
/

x
Figure :P6.61 Figure P6.66
6.2 The Method of Composite Parts 449

\ .. '

Quarter-circle
I
Figure P6.67

b
;
~ '
----7
1
f
I. a--~.I x
y Figure. P6.68b

f t
b

t L x
Figure P6.68a

6.67 Find the ratio of Rj to Ro for which the centroid of the


semicircular annulus in Figure P6.6 7 is at the highest point
of the inner bounding circle.

6.68 Given the two results shown in Figure P6.68a for the
y coordinates of the centroids, use the method ofcomposite
Figure P6.70
areas to find the y coordinate of the centroid of the shaded Figure P6.72
area in FigureP6.~8b. Do you really need the areas in this
problem?

6.69 Find the x coordinate of the centroid in Problem


6.68. Are the areas needed now?

6.70 In Figure P6.70 show by integration that the centroid


of the circular sector is given by

2r sin a
y=---
3a
6.71 In Problem 6.70 use L'Hospital's rule to show that as
Figure P6.73
the . sector becomes a sliver (a -> 0), the result for y
approaches that for a triangle, as it should.

6.72 Using the result of Problem 6.70, show by composite


areas (sector minus trianglel that the centroid of the shaded
circular segment shown in Figure P6. 72 is given by
r
40cm

2r sin3 a ~
'1=------
3(a - sin a cos a) t x
30cm
* Show that your answer approaches r as a. -> 0, as it must.
~
* 6.73 Find the centroid of the shaded area in Figure P6. 73.

6.74 Find the centroid of the shaded area in Figure P6. 74. Figure P6.74
450 Chapter 6 Centroids and Mass Centers

lO in. Top view

/'
V ------
"'"

/ '"
V I'--- 1\
I / "\
\
I I ,i .
I-somm-I
\ "~ .-/
/
I
16 in.

\ 1/
Figure P6.75
\ ..
j
\
.
II
y ':
1\ .... j
\ 1/
'" /
Figure P6.76

Figure P6.77 .,
6.75 The flat circular cam in Figure P6. 75 rotates about an * 6.76 Approximate the y coordinate of the centroid of the
axis i in.'.fromlthe' center of the 6-in. radius circle. To (shadedl cross-sectional area of the cam, which is sym-
balance the cam-that is, place the centroid of its cross metrical about the y axis. (SeeFigure P6.76.)
section at the axis of rotation-a hole of 2-in. radius is
machined through the cam. Determine the distance b to 6.77 Find the centroid of the volume of the very thin piece
the hole center that will accomplish this. of sheet metal in FigureP6.77.

~6.3 Center of Mass


The center of mass, or mass center, of a body B is the point C defined
by Equation (6.1) when the quantity Q becomes mass: .

mroc = f rdm (6.6)

where m = JlJdm is the mass of the body and the differential mass dm
z is related to its volume dV through the mass density p according to
dm = p dv (see the figure). Thus

x y
mroc = f rp dv (6.7)
,
i
-~
6.3 Center of Mass 451

If the density p is constant, then m = p V;where V is the volume of


body Band

p VIae = P f I dV

or

VIae == f I dV (6.81

which is seen to be the equation defining the location of the centroid of


the volume of B. Thus the mass center always coincides with the centroid
of volume for a body of constant density.
~ .

The mass center of a body play~ a central role in dynamics, and the
student will encounter it naturally there. In statics its significance is that,
regardless of the orientation of a body B on the earth's surface, the
resultant of the distributed gravity forces (weight) exerted on B by the
earth will have a line of action intersecting the center of mass of B (see
page 109).
We now consider some examples of locating mass centers. The first
example contains a body with variable density, and the second uses
composite parts.

E x A M p L E 6~15~
:,l::::::J~
I. L .1 x
The mass density of a uniform, circular cylindrical rod of constant cross-sectional
area A varies linearly with x as indicated in the figure. Find the center of mass.

SOLUTION
The equation for the spatial variation of the density is
0,
P2 - PI
P = ~-L---'-":'x + PI

(Continued)
Therefore, with y = z = 0 by symmetry,

Therefore,

x- (PI ++2P2P21 )L.


3(PI

As a check, if PI = P2' then x = tl2, as it should be for a uniform rod.

1-3m-r,m-1 E x A M p L E

Find the center of mass of the body composed of two uniform slender bars and a

-t- 6 kg
uniform sphere.

2m 4 kg

_!
C2

@- . 1.5 m 1.5 m

(Continued)

452
SOLUTION
We shall use the method of composite parts. The masses and mass-center
coordinates (see the figure on the preceding page) are:

Body Mass Mass-CenterCoordinates lx, y)

1 10 kg (O,Olm
2 6 kg (-2.5,01 m
3 4 kg (-4, -lIm

We have, for the x coordinate of the mass center,

(10 + 6 + 41£ = 10(01 + 6(-2.5) + 4(-4)


20£ = -31

£ = -1.55 m

And for the y coordinate,

20y = 10(01 + 6(0) + 4(-1)


y = -0.200 m

And, of course, Z = 0 by symmetry.

I
r
I
f
,
I
I
i

IF
I
PRO BLE MS I Section 6.3

6.78 Find the mass center of the body in Figure P6.78,


which is a hemisphere glued to a solid cylinder of the same
density, if L = 2R. Figure P6.78
-t
L
_!

453
454 Chapter 6 Centroids and Mass Centers

6.79 In the preceding problem, for what ratio of L to R is 6.85 Repeat Problem 6.84 if the four legs have uniform,
the mass center in the interface between the sphere and the but different, densities, so that the masses of .A, IJ, C, and
cylinder? oJ) are respectively, m, 2m, 3m, and 4m.

6.80 In Figure P6.80 find the distance d such that the 6.86 See Figure P6.86.
center of mass of the uniform thin wire is at the center Q of a. How far over the edge can the can extend without
the semicircle. falling over? It is open on the left and closed on the
6.81 Repeat Problem 6.80 if instead of a wire, the body is a right, and the thickness is the same throughout.
uniform thin plate with the same periphery. b. How far can it extend if the closed end goes first?

6.82 Determine the center of mass of the composite body 6.87 Use spherical coordinates, as suggested in Figure
shown in FigureP6.82. The density of .A is 2000 kg/m3, P6. 87, to show that the center of mass of the uniform body
the density of B is 3000 kg/m3, and the density of Cis formed by the intersection of the sphere of radius R, and the
4000 kg/m3. cone having vertex angle 2a, is at

6.83 In Figure P6.83 find the center of mass of the rigid 3R


Z =- (1 + cos a)
frame, comprised of seven identical rigid rods, each of 8
length f. 0

6.84 Find the center of mass of the bent bar, each leg of
which is parallel to a coordinate axis and has uniform
L = 3r
density and mass m. (See Figure P6.84).

y
x C t

t Figure P6.86
"'<'::'''''''"'''''Y''

Figure P6.80
Figure P6.84

/
x
Figure P6.83

@_/-dr
"/
r dq,
\
r sinq, dO

x x

Figure P6.82 Figure P6.87


6.3 Center of Mass 455

1.5inT
1.5 in. t
Figure P6.89
Circular
Figure P6.88
lin.-I£I cutout
3m.
6.88 Use the results of Problem 6.87, together with the
(a)
knowledge that the centroid of a cone is three-fourths the (b)
distance from vertex to base, to find the mass center of a Figure P6.91
uniform spherical cap without further integration. (See
Figure P6.88.1 Thin ring
(cross-sectional area 0.01 m')
6.89 The body in Figure P6.89 is made of a single piece of
sheet metal, cut into a rectangle and semicircle and bent at
90° as shown. Find.themass center.

6.90 A trailer carries a load of specific weight 1'lb/ft3,


which varies with y lbut not withz) as shown in Fig-
ure P6.90. Triangular plate
(thickness 0.07 m)
8. Find the weight and the position of the mass center of
the load.
b. Find the reactions FA' FH, and Fc due to the load only
ldo not'includt the trailer weight in this exercisel. .

6.91 Find the angle between line OA and the vertical if the Smooth horizontal cylinder
thin plate in Figure P6.91 is suspended at A by a string, as (axis normal to plane of paper)
,. 1
shown in the sketch (bl.

6.92 The object in Figure P6.92 is constructed of a thin


but rigid uniform semicircular wire and a right triangular
plate. The wire and the plate are made of the same
material. Locate, with the angle 8, the point on the wire
that would contact a smooth horizontal cylinder if the
object was hung on the cylinder as shown. The mass center
Figure P6.92

ft.__ .\

I
FigureP6.90 y c.L
'.

,
...•
\

456 Chapter 6 Centroids and Mass Centers

C of the semicircular -wire is at a distance of 2r / n from x


point Q.

* 6.93 Shown in Figure P6.93 is a scale model of a thin


parabolic shell structure. The shell's outer surface is a
l
IOOmm

curve formed by revolving the indicated parabola about the


y axis. Assume a constant mass density p-and that the
constant thickness t of the model is very small. Find
the center of mass of the shell model. Note that because
l
of the two assumptions you are actually seeking the cen- Figure P6.93
troid of the outer (curved) surface area.

6.94 The half cylinder (weight VY.-l and the stick (Yv.) are
attached and in equilibrium as shown in Figure P6.94. Find __
the x and y coordinates of the combined mass center, and
us'e them to determine the "lean angle" </>.

6.95 When a structure is very thin (thickness much


smaller than other dimensions) and flat, it is called a plate.
If it is very thin but curved, it is called a shell. These are
very important structural elements. Give an argument
proving that if a plate or a shell is homogeneous, its center
Figure P6.94
of mass is at its centroid of volume, which in turn is
approximately located at the centroid of its (flat or curved)
surface area.

* 6.96 .Referringiito Problem 6.95, find the center of mass


':1.,
Figure P6.96
of the thin conical shell of constant density shown in Fig-
ure P6.96.

~6.4 The Theorems of Pappus


There are two very old theorems of value when we are calculating either:
a. The surface area generated by revolving a plane curve around an axis
(which doesn't cross the curve I in the plane, or
b. The volume generated by revolving a plane area around an axis
(which doesn't cross the area) in theplane.
The two theorems are due to a Greek geometer of the third century,
Pappus of Alexandria, and they are developed below.
CDThe area A of the surface .A generated by revolving curve C i
i
J
about the x axis (see the figure on page 457) is equal to 2ny ds, or

A = 2n f y ds
6,4 The Theorems of Pappus 457

CurVe C of So from our knowledge of centroids,


arclength S
A = 2nys
y where s is the arclength of the generating curve C and y is the y coordi-
nate of the centroid of C. Thus the generated area is the arclength (s)
of the curve times the distance (2ny) traveled during the revolving by
the centroid of the curve. If the surface is formed by less than a full
revolution, say by angle <p between 0 and 2ic radians, then, of course,
A = <pys, still the arclength times the distance traveled by C.
x
@ The volume V generated by revolving the shaded area A shown
in the figure below about the x axis is

V = f 2ny dA = 2n f y dA = 2nyA

,-
,
!'
x

where y is this time the centroid of the shaded area. Thus the generated
volume is this area multiplied by the distance traveled by its centroid
!
l -
during the revolving. Again, for less than a full revolution, we would
simply have V = <pyA with 0 < <p < 2n.
f We now consider five examples ofthe use of the theorems of Pappus.
I
I,
I

y E x A M p L E
I
Ll Determine the surface area of'a right circular cone of radius R and height H.

H x
(Continued)
,
.~

SOLUTIONJ

Using the first theorem of Pappus, we avoid integration completely. The desired j
e
area will be generated by revolving the line in the figure about the x axis, and '

R
A = 2nys = 2n - .JR2 + H2
2

y
E x A M p L E 6.18~
Find the volume of a'right circular cone of radiusR and height H.

H x

SOLUTION
The volume will be generated if we revolve the shaded area .A around the x axis.
Then, by the second theorem of Pappus, C

as we have seen in Example 6.7. Note that the volume of a cone is one-third that
of the smallest cylinder that will enclose it-that is, the cylinder with the same
height and base radius.

E x A M p L E 6.19~
Find the surface area of a complete torus (doughnut).

SOLUTION
x We shall generate the surface area by revolving the circle about the x axis and
using the first theorem of Pappus:

A = 2nys

= 2nR(2nr)

A = 4n2RI

458
E x A M p L E
Find the volume of a hollowed torus.

SOLUTION
We shall revolve the shaded area about the y axis to generate the required volume.
By the second theorem of Pappus,

v= 2nxA = 2nRn(R; -R?l


V = 2n2R(R;- R?l

The theorems of Pappus are also very useful in finding centroids


when volumes and areas (or areas and arcsl are known. We illustrate this
with a final example.

E x A M p L E 6J21~
Find the centroid of a semicircular annulus.

SOLUTION

x
Revolving the area about x, we obtain a hollow sphere, and from the second
theorem of Pappus,

V = 2nyA .

4 n(R2 - R21
"3 n(R~ - R~) = 2ny .0 2 I

Volume of Generating area


generated
hollow sphere

or
_ 4(R; + RoR + Rflj
y=------
3n(Ro+ Ril

459
460 Chapter 6 Centroids and Mass Centers

P R 08 L EMS / Section 6.4

6.97 Find the surface area (exterior plus interior) of the


object of revolution shown in Figure P6.97. It has a rectan-
R; = 1ft
gular cross section with a circle removed.

6.98 Find the volume of the object in Problem 6.97.

6.99 In Figure P6.99 find the surface area of the body of


revolution.

6.100 Find the volume of the body in Problem 6.99.


Figure P6.101
6.101 Find the surface area (exterior plus interior) of
the tube, which spans !00° of a circle as shown in Fig-
ure P6.101.

6.102 Find the volume of the tube in Problem 6.10 1.

6.103 The lO-m diameter parabolic antenna. in. Figure ' I~

P6.! 03 has an {I D ratio (focal distance to diameter) of 0.28.


Find the surface area of one side of the thin shell.

6.104 Find the volume of water that the parabolic shell of Figure P6.103
revolution in Problem 6.103 could hold. "
l~

6.105 The shaft in Figure P6.IOS ends in a conical thrust


bearing. Find the surface area of the truncated cone, if
its upper and lower radii are 2 in. and! in. and its height is
I in.-t- 1.5 in.

,1.5in.[ 6.106 Find the volume.af the truncated cone in Problem


6.105.
lin._t_.

Figure P6.97
'LL:~~I
2in.

2.Scm

I em
L r-I
2.5 cm-l-

l--.j.
2cm 4cm
.1.
Scm
.1
Figure P6.99 Figure P6.105
6.4 The Theorems of Pappus 461
<

i 6.114 Find the area generated by the slanted line segment


s during the revolution of Problem 6.112.
Figure P6.107 6.115 Find the approximate volume of the thin, half-
toroidal shell shown in Figure P6.IIS.

~ .• f. x

6.107 Find the centroid of the semicircular area in Fig-


ure P6.107 by revolving it about axis x to generate a
spherical volume, using the second theorem of Pappus.

6.108 Find the centroid of the semicircle larcl in Fig-


ure P6.1 08 by revolving it about axis xto generate the sur-
face area of a sphere.

Half.Toroidal Shell,
,~ ..•~thickness' t + ••
Figure P6.108
Figure P6.115

x
6.116 Refer to Problem 6.93. Find sx for the parabolic
6.109 Find the area of the surface formed by revolving an segment between the points (0,0) and (100, 1001 mm.
equilateral triangle of side s about one of its sides. Then use the first theorem of Pappus to determine the
outer surface area of the shell.
6.110 Find.,thevolume generated by the revolution de-
scribed in Problem 6. 109. 6.117 In the preceding problem, find Ax for the area
bounded by x = 0, y = 100 mm, and the segment of the
6.111 Find the volume of the three-dimensional figure curve between (0, 0) and (100, 100) mm. Use this with the
generated by revolving the area about the line x = 0.8 m. second theorem of Pappus to determine the volume en-
See Figure P6.111. closed within the inner surface of the shell.

6.118 Find the shaded surface area of the hollow axisym-

f
a.6m
metric shell section shown in Figure P6. 118.

ill

I x2
(y - 5) =-
3

Figure P6.118
Figure P6.111

x
6.112 Repeat Problem 6. III but with the revolution
about the line y = O.

6.113 Find the area generated by the slanted line segment


sduring the revolution of Problem 6.111.
462 Chapter 6 Centroids and Mass Centers

6.119 In Problem 6.118 find the volume bounded by the


inside of the shell and the planes x = 0 and x = 3 in.

Problems 6.120-6.124 deal with various parts of a thin


haH-toroidal shell. 1.
I;
6.120 Find the shaded surface area (one sidel of the shell
shown in Figure P6.120.
Figure P6.123

,F,igureP6.120
Figure P6.124

6.121 Find the shaded surface area (one sidel of the shell
shown in Figure P6. 121. 6.125-6.129 Find the volumes enclosed by the shells of
6.122 Find the shaded surface area (one sidel of the'shell Figures P6.125-P6.129, respectively.
shown' in Figure P6.122.

6.123 Find,the,shaded surface area (one side) of the shell 6.125


shown in Figure P6.123.

6.124 Find the shaded surface area (one side) of the shell
shown in Figure P6.124.

Figure P6.125

Figure P6.121 6.126

Figure P6.122 Figure P6.126


6.4 The Theorems of Pappus 463

6.127 6.130 The dam spans 90°, as shown in Figure P6.130.


Find the volume of concrete that was required to build it.

Figure P6.127

6.128

30 ft

'Figure P6.128

6.129

Figure P6.130

,,
,.

1---- R ---"'"

Figure P6.129

C 0 'M'P UTI; R PRO B L EMS / Chapter 6

6.131 Write a computer program that will accept any y (ft)

number of triplets of numbers Ai' Xi' Yi' where Ai is a plane


area and (Xi' yd are the,x and y coordinates of its centroid.
The program is to read N(the number of areas making up a
composite areal, then read N tripletslAi! Xi! yd, then
'Figure P6.131
compute and print the total area A and the coordinates of
its centroid. Use the program to find the centroid of the
area in Figure P6. 131.

1.0 2.0 3.0 x 1ft)


464 Chapter 6 Centroids and Mass Centers

* 6.132 A crossed log-periodic dipole array (antennallooks Figure P6.132a


like a Christmas tree [see Figure P6.132aJ with tubes
extending in four directions from a mast. One of the four
sets of tubes is shown in Figure P.6132b. Not shown are
identical sets downward and both into and out of the paper.
The figure shows a simplified model of such an antenna;
the tubes actually get smaller from left to right and the
mast is in sections that also decrease in size.
Find the mass center of the antenna. The x coordi-
nates of the elements lin inches) are the numbers below the
mast. Assume the elements are attached to the outside of
the mast. Would the mass center be the same point if the
entire antenna was covered with an inch of ice?

Figure P6.132b
.,..
Assume each of these eight
elements to be a tube with
outside diameter (OD)
:'.~,.•..
1.40 in. and wall thickness Mast = tube with OD = 1.81 :f

0.085 in .... and wall thickness = 0.091 in.


~r

... and these twenty have


OD = 0.380 in. and wall
thickness 0.050 in.
61.6 in.

.j

r::
-~ -----~---------------------------------------------------
•...

.~

~ a <n ~ 00

1
00
r<) .<'1 00 "<t '-0 "<t 0- "'1 <'I I'-- <'I
"'1 <'I
" <'I "<t 0-
.5 0-
v-i

- --
v-i 0\ -0 '<i- ("1j
.--.: c:i ("1j -0 00 -0 ("1j 00 0\ c:i
~ ~ ~ ~ ::!: ::!: ::!:
00 00
00
"" "" "" a ;j ;j ::!: ~ <n ""
.<n
<'I r<) <n '-0 I'-- 00 00 0- ~ r<) "<t

c:i

J
157.7 in.
Answers to Questions 465

* 6.133 A man built a basketball goal for his children out of


a 15-ftsteel pipe weighing 3.03Ib/ft, and having an inside
diameter of 3.33 in. The goalvibrated too much when the
basketball hit the backboard; so the man filled the pipe
with concrete, which increased the bending stiffness of the
pole. The concrete weighed 144 Ib/ft3. How full (percentl
was the hollow pipe when the combined mass center of
pipe and concrete was at its lowest point? (See Figure
P6.133.) Hint: Write a program that will list or plot the
mass center height versus con~rete height, and pick off the
answer from the list or plot. Check your answer by.using
calculus to obtain the exact solution.

*6.134 The density of the earth varies greatly through its


depth. It can be fairly well approximated bya graph con-
sisting of straight lines through the following points:
Depth from Surface Density
ofEanh(miles) .•• Jg/e<e<' .
o 3.50
400 4.30
1000 5.20
1800 5.70
1800 (jumpl 9.60
2500 .10.8
3100 'II. 7
3300 14.0
3500 15.0
3960 16.3

Assuming spherical symmetry, write a computer program


to calculate the mass of the earth, and use that result to
calculate the average density of the earth.

Answers to Questions I Chapter 6


Q6.1 Zero, because y = o.
Q6.2 •. The coordinatey would not be the,same for all parts of the element.
Q6.3 No. The integrals of arclength times y-distance are what are balanced; i.e., the
"first moments" are balanced above and below the:line t-I.
Relabeling dimensions, i = 2H/3 in the figure at the left.
This time it is "y-distance times area", which is b:ilanced above and below the
horizontal line through the centroid. The semicircle is now "filled-In" and there
,ismuch more quantity Q (area)lower than there was when Q was arclength. This
time, the "first moments of area" are balanced.
This time it is the first moments of volume that are balanced above and below the
plane z = iR. There is proportionately much more volume near the plane z = 0
than there was area near the line y = 0 in Example 6,5 .
•,Q6.7Because, with the four areas equal, two +4.coordinates cancel two -4
coordinates.

L_.
466 Chapter 6 Centroids and Mass Centers

Q6.8 No. As the thickness gets smaller and smaller (let Rj ~ Rol, the volume elements
get closer and closer to the surface of the outer hemisphere.
Q6.9 A sphere whose density depends only upon the radius.
Q6.10 Yes; the answers are 1L and iL. These solutions are the same as for the thin plates
A and ~ of uniform density. The solutions should match, for with these
plates we have a linear variation of height instead of density.
Q6.11 No, it does not. A length of straight pipe has a mass center on its axis, in the
space inside it.
Q6.12 Bend a pipe cleaner. The mass center before bending is on the axis of the pipe
cleaner, while afterwards it is not.
Q6.13 Yes. AsRj '-+ 0, Y ~ 4R';3nandasRj ~ Ro' Y ~ 4(3R;)/[3n(2Roll = 2Ro/n. We
obtained these results in Examples 6.5 and 6.2, respectively.

Review Questions I Chapter 6 ,


True or False?
]'
•.. ...•...

1"

1 The centroid of an area depends upon which reference point is used to locate it in
the defining equation Aroe = r dA.J ~
2 The centroid of a quantity Q is the point where all of Q could be concentrated
with the same resulting first moment as has the actual distribution of Q.
3 All centroid calculations for areas require two integrations; centroid calculations
for volumes require three integrations.
4 Any line through the centroid of an area divides the area into two equal parts.
5 The centroid of a semicircle is farther from the circle center than is the centroid of
the semicircular area contained within it.
6 Suppose that in an arbitrary triangle we drop a perpendicular from a vertex V to
the opposite base lline I, extended if necessaryl and find that the distance from V
to f is 9 in. The centroid of the area of the triangle is then 3 in. from f.
7 IfanareaAismadeupoftwopartsA1 andAzsuchthatA! + Az = A,thenthex
coordinate of its centroid is given by x = IA!x! + Azxz)/(A! + Azl.
8 Finding centroids of areas with cutouts is often simplified by the method of
composite parts.
9 The center of mass ofa body has to be a material (physical) point of the body.
10 The mass center of a rigid body is always the same point of the body (or of a rigid
extension of the body).
11 The centroid of the volume of a body coincides with its center of mass only if its
density is constant ..
12 The integral J.",r dm equals the mass of B multiplied by the vector from the ori-
gin of r to the mass center of B.
13 The use of the Theorem of Pappus always involves the revolution of a curve or an
area about a given axis in the same plane.

Answers:l.F2.T 3.F 4.F S.T 6.T 7.T S.T 9.F 10.T 11.F l2.T l3.T
Inertia Properties of
Plane Areas

7.1 Moments of Inertia of a Plane Area

7.2 The Polar Moment of Inertia of a Plane Area

7.3 The Parallel-Axis Theorem (or Transfer Theorem) for Moments


of Inertia/The Radius of Gyration
7.4 The Metho<tof Composite Areas

7.5 products of Inertia of Plane Areas


7.6 The Parallel-Axis Theorem for Products of Inertia

7.7 Moments and Products of Inertia with Respect to Rotated Axes


Through a Point/Mohr's Circle

467
468 Chapter 7 Inertia Properties of Plane Areas

.
In this chapter we study the inertia properties of plane areas. One reason 1
for studying this topic in statics is that ,these properties arise in the
formulas for locating the resultant of hydrostatic pressure forces on' a
submerged body (which we shall examine in Section 8.2). A more
j
important reason for this study is that it is sometimes considered a.
prerequisite for courses in strength of materials (or deformable bodiesl,
1
which follow statics. In these later courses the student will find that
stresses in a transversely loaded beam are, under special but important
circumstances, inversely proportional to a moment of inertia of the cross-
sectional area of the beam. Deflections of the beam will likewise be
inversely proportional to this moment of inertia, which forms part of the
resistance to bending of the beam. Similarly, the "polar moment of
inertia" is a factor in the resistance of a shaft to torsion, or twisting.
The first four sections of this chapter can be read by a student who is
familiar with no more than single integration. These are the sections
normally needed in the first course in the mechanics of deformable solids.
The last. three sections,. however, .utilize,double..integrals when dealing
with products of inertia.
Moments and products of inertia of mass are needed in dynamics i we
shall cover this related topic in our second volume at the point where the
subject arises naturally.

~7.1 Moments of Inertia of a Plane Area


For the plane area shown in the figure, the moments of inertia * with
respect to the x and y axes are defined to be

Ix = 1/ 2
dA (7. Il

and

Iy= JA x2 dA (7.2)

These definitions illustrate why a moment of inertia is sometimes called


a "second moment" - because of the square of the distance from the x
axis for Ix (and from the y axis for Iy)' We have seen "first moments" in
Chapter 6 in relation to the concept of centroid. Because a moment of
inertia is made up of areas multiplied by squares of distances, it has the
dimension (length)4.
I;
*Many prefer to use the term "second moments of area," feeling that the word
"inertia" suggests mass and should be reserved for .similar integrals that reflect the mass
distribution of a body. Nevertheless, the terms "area moment ofinertia" and "moment of
inertia of area" are widely used in texts on the mechanics of deformable solids.:
7.1 Moments of Inertia of a Plane Area 469
Equations (7.1) and (7.21 also tell us that a moment of inertia is
always positive and is a measure of "how much area is located how far"
from a line. If we wish to be specific about the origin of the x and y
coordinates, we may write, for example, Ixc if the origin is the centroid, or
Ixp if the origin is some other point P.
We now proceed to use the above definitions to find moments of
inertia of several common shapes in the following examples.

E x A _ M p L E 7.1~
Find the moments of inertia of the rectangular area about the centroidal x and
yaxes.

h
i
j"

SOLUTION
J
To find lxc' we need the integral yl dA. Using the horizontal strip shown in the
dA = bdy second figure for our differential area dA, and noting that the y coordinate is the
same for all parts of the strip, we get
dy

= by3lh/l
3 -hll

A similar integration with dA = h dx as shown below gives lye:

f
bll x3lb/l
ly = xl(h dxl = h -
e x= -bll 3 -bll
dA=hdx '-----'
dA

h
x

b
For those familiar with double integrals, we use them below to reproduce the
result for Ixc:

hl2 fbl2
Ixc =
f y= -h12 x= -b12
y2 dx dy
~
dA

hl2 fhl2
f Ibl2 y31hl2
= X y2 dy = y2 b dy = b-
y= -h12 -b12 -h12 3 -h12

Note that the first integration (on xl produces the strip b dy used earlier.

E x A M p L E 7.2~
Find the moment of inertia of a circular area about any diameter.

SOLUTION
Since x2 + y2 = R2 on the boundary, our dA in the figure is given by

dA = 2.JR2 - y2 dy

so that

Ixc = f y2 dA = J:R 2y2.JR2 - y2 dy

= 2 f: 2y2.JR
2 - y2 dy

Substituting sin 0 for y/R, and noting that d(y/Rl == cos 0 dO,
n/2
IXc = 4R4
f o
sin20 cos O(cos 0 dO)

(Continued)

470
where for the integral limits, y/R = 0 when 8 = 0, and y/R = 1 when 8 = n/2.
Continuing,

1
1<12 sin2 28
I = 4R4 --d8
Xc 0 4

1
1<12 1 - cos 48
= R2 . d8
o 2

For the reader acquainted with double integrals, we can obtain the above
result with somewhat less effort using polar coordinates as follows:

lxc = f y2 dA = f21<
J8=0
fR
r=O
(r sin 8)2r dr d8
'----v---'
dA

• "' •.I~ ,-,," .,.•••


_"' ..~r. _, ,,_

_ R4
--
4
1 0
2
1«1-
--- cos 28) d8--
2
_ R [ 8---
8
sin 28J121<
2 0
4

This is, of course, also lyC or I about any other diameter of the circle.

E x M p L E 7.3~
r
b A

dA Find the mOment of inertia of the triangular area about the y axis.

I x

SOLUTION
For our differential area, we shall use the shaded strip in the figure; thus, using Y
to locate the lowest (boundary) point of the strip,

dA = (h - Yl dx

(Continued)
!
'.
f
471 .

L
But Y = 2h x for the side of the triangle in the first quadrant, so that
b

dA = ( h-T2hX) dx

Therefore,

PRO BL EM S I Section 7.1

7.1 In FigureP7.l: 7.4 Review Examples 7.1 and 7.3.


Find the moment of inertia of the shaded area about the a. Find the moment. of inertia of the shaded area in
x axis. Figure P7.4 about the y axis.
Teilwhy ly is identical to lye of Example 7.1. b. Add the result to that of Example 7.3, and note that
.;;;"

the answeris, as it should be, that for the combined, I

b
rectangular area.

Figure P7.4
rh

Figure P7.1
x

7.2 Find Ix; for the shaded area in Figure P7.2.


1 x

.y(m) 7.5 Determine the moment of inertia of the shaded area in


4 Figure P7.S with respect to the xaxis.

y x = 4cm

f ;:
~,

y = 2cm

'Figure P7.2 x

7.3 In Problem 7.2 find ly for the same area. Figure P7.5

472
7.2 The Polar Moment of Inertia of a Plane Area 473

Thickness t
(t « r)

Figure P7.9

FigureP7.7
Figure P7.11

'7.6 Repeat Problem 7.S for the moment of inertia about 7.11 Show that the moment of inertia Ix of the shaded,
the y axis. - annular area in Figure P7.II is n(R: - Ri)/4 by:
7.7 Find Ix for the thin circular section shown in Figure a. Integrating as in Example 7.2 but with limits of J;;
P7.7. Hint: Usepolar.coordinates,:withdA_= tI.dO." '-on- th e in tegra tiomwi th Fespect ''to 'I.
b. Subtracting the moments of inertia of the two circles.
7.8 In Problem 7.7, find Iy.
Why does this work?
7.9 Find the moment of inertia about the x axis of the area
7.12 In Problem 7. 11, let the figure depict the cross
under the nth-degree curve shown in Figure P7.9. (n is an
section of a round tube. If the' tube is very thin (Rj -+ Rol,
integer, n :2:: 1.)
show that Ix is approximately nR3t, where R = Rj ~ Ro
7.10 In Problem 7.9, find Iy• and t = thickness of tube = Ro - Rj•
,
1"."'.l.

~7.2 The Polar Moment of Inertia of a Plane Area


In the study of deformable solids, the "torsion problem" refers to what
happens to a shaft when it is twisted. In the same way that the moment of
y
inertia forms part of the resistance ofa beam to bending, the polar
moment of inertia forms part of its resistance to,twisting. For this reason,
then, we shall discuss the polar moment ,of inertia in this section.
The polar moment of inertia of an area about a point Pis defined to
be (see the figure)

'p = f (x
2
+ y2) dA

x where the axes (x, y) have origin at P. Since the polar coordinate r is given
by* x2 + y2= r2, this simplifies to

*Hence the name "polar moment of inertia."


474 Chapter 7 Inertia Properties of Plane Areas

If we recall that both Ix and Iy are the sums (in integration) of (differential
areas) times (squares of their distances from the axis), then we see that Jp is
the same type of quantity, this time with respect to the z axis (normal to
the area) through a point P. Also,

(7.3)

so that for an area moment of inertia, the x and y (in-plane) inertias add up
to the zlout-of-plane) inertia.

E x A M p L E
Find the polar moment of inertia of the circular cross section with respect to
point C.

SOLUTION
By Ie = lxc + lyC' we obtain immediately from the results of Example 7.2:

nR4 nR4 nR4


Ie =4 +4 =-2-

Alternatively, the shaded "circular strip" in the second figure has the
feature that all its elements are the same distance r from the z axis. Thus with
dA = 2nr dr,

Ie = J 2
r dA = f: 2
r (2nr dr)

=
r41R
2n-
4 0
And if double integrals are familiar to the reader, we obtain the same result a third
time using polar coordinates:

e"
Ie =
f 2
r dA = Jo=o fRr=O r21r dr dOl
4 2
=.
1 2"

0=0
r41R
-'-dO
4 0
= -
R 01 "
4 0
E x A M p L E

Find the polar moment of inertia of the rectangular cross section at point C.

h SOLUTION

Because Ie = lxc + lYC1 we use previous results and get

or

P R OB L EMS / Section 7.2


~'.

In Problems 7.13-7.19 find the polar moment of inertia 7.15 7.16


with respect to the origin for each of the shaded areas.
y x = 4cm
7.13 7.14
y' = 4x

b y(m)
)~ ,
~;.

4 -] y = 2cm

h
h
a x

j Figure P7.16
a x
a x
FigureP7.15
Figure P7.13
o 2 x(m)
Figure P7.14
7.19

7.17
7.18

t «r y

a b x
Figure P7.18 Figure P7.19
Figure P7.17

475
476 Chapter 7 Inertia Properties of Plane Areas

~7.3 The Parallel.Axis Theorem (or Tr~nsfer Theorem)


for Moments of Inertia/The RadiuS of Gyration
Fortunately, if we wish to determine the moment of inertia I about an xp

axis through a point P other than the centroid, we don't have to integrate
if we already know Ixc' All we need do is' 'transfer" this result, adding the
area A times the square of the distance dbetween the parallel x axes
through, C and P(seeFigure 7. 1 below).

,Ixp = Ixc + Ad2 (7.4)

We now proceed to prove this very helpful theorem. We have

Ixp ~ 'f
'A
y2 dA = f A
(Y + Yl)2 dA

Ixp =,LY.i dA, + y2 L dA + 2j1 LYl dA

Ixc

Figure 7.1

where from Figure 7.1 We see that the ycoordinate of dA is j1 + Yl'


where y is the Ycoordinate of C.

The la'st integral, J Yl dA, is zero by the definition of the centroid,


and therefore
Ixp = Ixc + Ay2 = Ixc + Ad2
r
,
J
7.3 The Parallel-Axis Theorem (or Transfer Theorem) for Moments of Inertia/The Radius of Gyration

where d is the distance between the two parallel x axes through P and C.
477

This is the parallel-axis theorem that was stated in Equation (7.4). Note
from Equation (7.4) that the smallest value for the moment of inertia of
an area is with respect to an axis through the body's centroid. For y axes,
we have

(7.5) .

where this time the absolute value ofx is d, the distance between the
parallel axes through P and C.
For the polar moment of inertia, we also have a parallel-axis
theorem:

Ip (or Izp) = f (x2 + y21 dA = f [(x + XI)2 + LV + YI)2] dA

i.
= f (xi + yi) dA +y f Xl dA + x..f ~l w: ~ f (x
2
+ y21 dA

= Ie + a + a + Ad2
Ip = Ie + Ad2 (7.61

where once again d is the distance between th~ panillel axes, this time z
axes through P and C, of interest. It is also the distance between the
II points P and C in the plane of the area.

E x A M p L E 7.6~
Find the moments of inertia of the rectangular area about the indicated axes
through the lower left comer P.

SOLUTION
Using the parallel-axis theorem together with the results of Example 7.1,
3 3
p b I (or I ) = I + Ad2 = _bh_
12
+ bh(!:)22 = _bh _
x xp xc 3
3 3
I (or I )
y Yp.
= I
YC
+ Ad2= _hb_
12
+ bh(~)2
2
= _hb_
3

I•..
~.
y E x A M p L E 7:. 71Jtaee",
Find the moments of inertia of the circular area about the axes x and y shown in
x
the figure.

SOLUTION
Using the parallel-axis theorem, and the results of Example 7.2,

[because .the distance between the y axes


through the two origins vanishes. (They
are the same line!)]

y E x A M p L E

(a)Find the moment of inertia of the triangular area about the x a~Js through 0.
(b) Then use the transfer theorem to determine [xc.

SOLUTION

lal By the definition,


o b x
[xo = I y2 dA

Noting that the hypotenuse has the equation y = (~)x, we have (see the figure
at the left):

[
Xo
= (h
Jo
y2(b _ ~
h
Y) dy = bh3 3
_ bh
4
3
= bh
3

12

(b) The transfer theorem gives

bh
12
3
= [
Xc
bh
+ 2 3
(~)2 = [xc
bh
+ -1-8
3

(Continued)

478
so that

bh3
I =-
Xc 36

Notice that we don't always transfer from C to other points; in this case we
knew the answer for Ix at 0, and we used it"in reverse" to find Ixc'

E x A M p L E
Find the polar moment of inertia of the semicircular area with respect to its
centroid C.

4R t
31rL
o x

SOLUTION
At point a we have

because (seeExample 7.4) we must get half of the answer for the full circular area.
Using the parallel-axis theorem,

10 = Ie + Ad2
4 2
nR nR (4R)2
T=le +2 3n

Ie = nR
4(14" - 8 )
9n2 ."

= O.160nR4 = O.503R4

479
E x A M p L E
t,
+
section, * the manual of the American Institute
t For the C 15 x 50 channel
Steel Construction lists the area of the cross section as 14.7 in. 2 and the centroidal
of

~ -t", .moment of inertia about line x-x


as 404 in.4 If d = 15.0 in., find the moment of
inertia about the baseline axis x.
x~ ~x d

~ I~x SOLUTION

We simply use the parallel-axis theorem, and obtain, without integration,

Ix = Ixc + Ad2
= 404 + 14.7 (7.5)2
d = depth
hI = width of flange = 1230 in.4
tI = average thickness of flange
tw = web thickness
x = distance locating centroid of section

This is a good place to introduce the reader to a distance called the


radius of gyration. t The radius of gyration is a concept associated with
the moment of inertia of an area about an axis. The radius of gyration is
simply the length that, when squared and multiplied by the area, gives
the moment of inertia about the axis. Thus, fo1'example,

(a) (7.7)

For a physical interpretation of the radius of gyration, consider the


areaA in Figure 7.2(al having the moment Ofinertia Ix about the x axis. If
we squeeze the same area into a very thin ~trip and place it a distance D
from the x axis (and parallel to it as shown in Figure 7.2(bl, then the
moment of inertia of the strip about the x axis is

(7.81

Clearly, then, there is a distance D for which I~TRIP equals Ix of the


actual area. This value of D is the radius of gyration kx and is therefore
(b)
JlxlA, as in the definition above IEquation (7.71].
Figure 7.2

*Meaning "C" for channel and that the nominal depth is 15 in. and the weight per
lineal foot is 50 Ib/ft. See the AISC Manual of Steel Construction, American Institute of
Steel Construction, Inc., 101 Park Avenue, New York, N.Y. IDOl?
t This parameter arises naturally in the analysis of buckling of columns.

480 ...... ~ ...'


7.3 The Parallel-Axis Theorem (or Transfer Theorem) for Moments of Inertia/The Radius of Gyration 481

As examples, the radii of gyration of the following areas about the


indicated lines are shown beneath the various figures.

B x

g.
kx =- k = fiB k == fiH
3
2 y 6 x

nR4 HB3 BH3


since Ix = -- since Iy = --- since Ix = _.-
,4 . 12 . 3.

Average radius

k = j2H kz ~ R
x 6
since 1,~2nR3t
BH3, (and A = 2nRt)
since I = -''-
x 36

,p .ROBLEM S I. Section 7.3._

y(m) 7.21 y
For each of the areas shown in Problems 7.20- 7. 23 find the
moment of inertia about a line through the centroid 4
parallel to x.
y = x' -1
7.20 (See Problem 7.2.1
Figure P7.20 h

_I
x

Figure P7.21
482 Chapter 7 Inertia Properties of Plane Areas

7.22 (SeeProblem 7:5.)

)
x
y2 = 4x
Figuref»7.28
Area = -Tcrn2
Figure f»7.29

that
x(crn) 2
h == ~(R2 + RI +1 )
4 3n R +I
FigureP7.22
7.30 The.shaded region .A in Figure P7.30 has an area of
10 ft2• If the moment of inertia of the area about the y axis
7.23 (SeeProblems 7.9 and 6.23)
is known to be 600 ft4, what is the moment of inertia of the
area about the YJ axis (i.e., about the line x = 3 ft)? The
y centroid C of this area is located at coordinates (7, 3) ft.

Figure P7.30 Y

~I ft

2 ft
In Problems 7.24-7.26, for the problem referred to, find
the moment of inertia about a line through the centroid x
parallel to y.

7.24 Problem 7.20.


7'.31 The same shaded region in Problem 7.30 has a radius
7.25 Problem 7.22. of gyration with respect to the ~axis of 2 ft. What is its
,moment of inertia with respect to the x axis?
7.26 Problem 7.23.
7.32 In Figure P7.32:
7.27 In Problem 7:8, the.,distance' from the ongm to
a. By integration, determine the moment of inertia of
the centroid of the very thin section is approximately
the elliptical area about the x axis.
Ir sin(aI21Vlal2l. Use the parallel-axis theorem to find lyc'
, b. What is the radius of gyration of this area with respect
7.28 Use the parallel-axis theorem to find the moment of to the x axis?
inertia of the semicircular area about an axis through C
parallel to x.(SeeFigure P7.28.1

7.29 Show that the centroidal moment of inertia lxc of the


annular area shown in FigureP7,29 is b
a
,Figure P7.32
4 4 0.283R2r2(R - II x
O.nO(R - r ) - ~----- b
R + r

0.3tr3 if r :::::;
Show that this result becomes lxc :::::; R. Note
7.4 The Method of Composite Areas 483
~ .

7.33 In FigureP7.33:

a. Find the polar moment of inertia Ie of the equilateral


triangular area with respect to its centroid C.
b. Compare the result of part lal with Ie of a square cross
section having the same. area. Think about which
should be larger before m~king the calculation.
5
7.34 Find the polar moment of inertia of the triangular Figure P7,33
area of Problem 7.33 about one of its vertices.

For Problems 7.35-7.37 find the polar moments of inertia


of the areas about their respective centroids in the prob-
lems referred to. 5,

7.35 Problem 7.20.


,.:r-
1i ."'

7.36 Problem 7.22.


7.39 In Problem 7.20, find the radius of gyration ky of the
7.37 Problem 7.23. shaded area. .

7.38 In Problem 7.20, find the radius of gyration kx of the 7.40 Find the radius of gyr~tion kx for the sh'aded area of
shaded area. Problem 7.21.

~7.4 The Method of Composite Areas


We learned in Chapter 6 that in finding the centroid of a composite area
A, the integration could be divided into separate integrals over the
various areas comprising A. Thus, for example, if Al + Az + A3 = A,
then
..~.

Ay = f y dA = .f'. ydA + f y"dA + f y dA


A A, A2 A3

....•
or
_ AIYI + Adz + A3Y3
y=-------
Al + Ai + A3
This same idea, or method of "composite areas," may be used in calcu-
lating moments and polar moments of inertia:

Ix = f A
yZ dA = f.
A,
yZ dA + f .. , yZ dA + f
Ai A3
yZdA

== 11' + 1: + 11 2 3

where 11' is the moment of inertia of the area Al about the x axis, and
similarly for Az and A3•
We consider some examples made simpler by the use of composite
areas.

"

l
E x A M p L E
Calculate the polar moment of inertia of the ten identical small bolt cross sections
(studied previously in Example 6.101 about both the origin and point C.

~
4 in. .t j In.. C(3.6 in; 4.6 in.)

SOL UTI ON
If, as the figure suggests, each bolt diameter is small in comparison with the
spacings between bolts, then all points in one of the areas are approximately 4;.'
the same distance from the point with respect to which we desire to calculate"
the polar moment of inertia. Then

fo = f I
2
dA= f
At
I2 dA + ... + f..
. AID
I2 dA

where Ii is the distance from a to the center of the ith area. The expression above
corresponds to thinking of each bolt's area as "concentrated" at its center.
With identical areas (Ai = AI, we obtain
10
fo = A
i=1
L If .
Starting at a and proceeding counterclockwise we have
~~~.2 2. 2
fo = A[O + 4
2
+ J7 +
2
4
2
+ J112+ 4
2
+ JI12 + 82

+ ../72 + 8
22
+ J42 + 8
22
+ 82 + J(~412 + 5
22
+J(~4Ii +1
22
J
_= A(42 + 72 + 42 + 112 + 42 + 112 + 82 + 72 + 82 + 42 +82

+ 82 + 42 + 52 + 42 + 12]

7l8A in.4
(Continued)

484
.-J where A is in in.2 Note, in the above steps, thaUt is a waste. of time to take
the square root to get Ii because we are going t; then immediately square it
to get rf.
Now let us back up and put this analysis on a more rigorous basis. Let Ic; be
the polar moment of inertia of the ith area with respect to its own centroid. Using
the fact that polar moments of inertia of composite parts add to yield the polar
moment of inertia of the whole area, and also using the parallel-axis theorem for
each of the parts,

10
'0 = I
j;:;:l
(Ic; + Ajrfl

where rj is the distance from 0 to Cj• Thus

10 10
,
10 = I Ic; '+ I Ajrf
i· j::::l j::::l

and the second sum is precisely what was calculated above. For circular areas
,
)'i (see Example 7.4)

where Rj is the radius of the ith bolt's area. With identical areas, we have Rj = R,
Ai = A, and:

'0 = -_.
lOAR2
2
- + A I rf
10

i=1

= SR A 2
+ 718A
= (SR2 + 718lA in.4

and the smaller is R the better is the concentrated-area approximation. For


example, if R = ! in., then SR2A = 156A and the error associated with the
concentrated-area approximation is about 0.04%.
To get Ic (the polar moment of inertia about the centroidl, we shall use the
transfer (orparallel-axis I theorem. The centroid Cwas computed in Example 6.10
to be at (x, y) = (3.6, 4.6) in. Therefore,

where d = Irocl. Substituting,

t,
Ic = 718A - (10AIl3.62 + 4.62)
= 718A - 341A
t o-

= 377 A in.4
j~
Note that the transfer theorem can be used to find Ic if I is known with
respect to some other point such as 0 in this example.

L - 485
E x A M p L E 7.12~
Find the moment of inertia Ixc for the cross-sectional area of the channel beam
shown in the figure.

Gin. •
C

SOLUTION
We shall solve the problem by finding Ix and then using the transfer theorem. We
shall find Ix in two different ways.
])/
x

~ ------------ ....
= 3.5(63) _ [3
3
'-'

.x
12
53 + .(3 x 5H2.5 + 0.5)2 J
y
= 252 - 166

= 86 in.4
x x
W(whole) fi(hole) Alternatively,

= 310
-'- 53l
[ 12
+ (3 x 0.5H5.752) ] + [3(05
,12
3
_:_. ).- + (3 X
J
0.5HO.252)

+ [0.~~3) + (0.5 x 61132 )J


= 49.6 + 0.125 + 36.0= 85.7 ~ 86 in.4 (as before]

Using the parallel-axis theorem,

Ix = Ixc + Ad2
85.7 = Ixc + [6 x 3.5 - 5 x 3](3)2
Ixc = 85.7 - 6(321 = 31.7 in.4

The calculation of lye will be given as an exercise.

486
E E x A M p L E

Show that even if a triangle, such as the one shown in the figure, has no right
. -
angle, its moment of inertia about a centro ida I axis parallel to a base B is still
BH3/36, where H is the height of the triangle in the direction normal to
that base.
H

L F

I~ 13----1--Q--1
x
SOLUTION
Let us call the shaded triangle AED of interest :J, and denote the two right
triangles AEF and DEF by Wand fl, respectively. Note that A'7 = AUf - A", so
that

J';c = f y2 dA = f y2 dA - f y2 dA
5 10 .JJ

3 3
IB + QlH + (B + QJH W, I _.[QH + QH1d2 ).J
36 2 36' 2 \ ."5
11.'7
. .
where dW'7 and d".'7 are the respective distances between the x axis in the figure
and the x axes through the centroids of W and fl. But all three of these' x axes
are coincident, a distance H/3 above the base line ADF, because of earlier gen-
eral results for centroids of triangles. Therefore, duo = d,,_'7 = 0, and we obtain

the same result that we obtained for the right triangle in Example 7.8.

E x A M p L E 7.:14~
From previous examples, we have the moments of inertia shown in the figure.
Show that If.: for the rectangle follows from the indicated moments of inertia for
the two right triangles, using the transfer theorem and the method of composite
areas.

bh"
-t b
,.-.7:-.:-.-.-,,--,-. _J
-
36
for the dOlled area

___ :-':''-:.0:.:..,''': .
h I. C,' ..

~ : x x
\
bh3 for the shaded area
-
12

(Continued)
SOLUTION
Transferring the moment of inertia of the dotted area to the x axis gives

jdolled = bh 3 + ~ bh (~ h)2
x 36.2 3

bh3 bh3 bh3


=-+-=-
4 12 3

which agrees with the previously obtained answer for the rectangle.

E X AMP L E
o.h8 in. = t Find the moments of inertia about the x and ~ axes throug the centr?id of the?
T
d = 14.18 in.
x
f f cross-sectional area shown in the figure. ThIs cross section of a wIde-flange
beam is called a W14 x 84 because its nominal depth is 14 in. and it weighs
84 lb per ft.

d = depth
bJ = flange width
t J = flange thickness
tw = web thickness

SOLUTION

0.451[14.18 - 2(0.778IP
12

web

+ 12.02310.778)3
2 [ -----+
I
12.0230.778
1(14.18
--. ---
0.778)2J
12 . 2 2 ,

flanges

(Continued)

488
Thus

Ixc = 75.6 + 2[0.472 + 4201 = 916 in.4

Notice that the flanges contribute most of the inertia (91.7%!l because of the large
"transfer term." Continuing,

I = 2 .[0.7.78(1.2.023.)3.J
.. + [14.18 - 2(°..7781](.°.45113
'
YC, 12 , 12

flanges web

= 225 + 0.0965 = 225 in.4


Notice how tiny the web contribution is here. (Its area is all very close to the
y axis!)
The American Institute of Steel Construction: manual lists Ixc and IyC to be
928 in.4 and 225 in.4 The table values will generally be higher than the results of
."';-.'. calculations such as we have made above because of the additional material due
to the rounded fillets where the web and flanges are joined. To further illustrate
'this point,.note that wewould'compute-the-are:r of the section to be

A = 2(12.0231(0.7781 +[14.18 - 2(0.7781](0.451)

= 18.7 + 5.69 = 24.4 in.2


whereas the true area (again from the steel construction manual) is 24.7 in.2

P R OB LE M S / Section 7.4

7.41 Compute Ix for the I section shown in Figure P7.4l. 7.42 Find the moment of inertia Ixc of the hollow rectan-
gular area shown in Figure P7.42.

FigureP7.41
x

Figure P7.42

489
490 Chapter 7 Inertia Properties of Plane Areas
,1
I
7.51,7.52

~
,.
Figure P7.43
Haem

I~---b----

Figure P7.51, P7.52


7.43 Find I.e for the area shown in Figure P7.43.

7.44 In Problem 7.43 find Iy•


7.53,7.54 0.1 ft
radii\
In Problems 7.45-7.68 fiIid the moment of inertia about
the x axis (odd numbersl and y axis (even numbersl for the l~
areas. y

7.45,7.46

Figure P7.53, P7.54

Figure ,P7.45, P7.46.


7.55,7.56

y
x

+ . ,+
------I 1-- 0.5 ft J.I
7..47,7.48 3.5 in. 0.2ft -1--
0.2 ft
Figure P7.55, P7.56

Figure P7.47, P7.48


7.57,7.58

7.49,7.50 x
t
I ft
y
-~
Figure P7.49,P7 .50

Figure P7.57, P7.58


7.4 The Method of Composite Areas 491

7.59,7.60 I.
I r ~I~I
lOin. 7.65,7.66
20cm-'."""'+ .-~ 20cm

-I

r
sOcm

_1
120cm

9 in.
-I
sOcm

_1 x
Figure P7.65, P7.66

x
Figure P7.59, P7.60 * 7.67,7.68

Problem 7.61 is actually an S24 x 120 I beam, for which O.lsm


the published centroidal moment of inertia is Ix =
3030 in.4
1
7.61,7.62
+ I 8.048 in; I
(
/ /-t-~ '
--\
/ -""\!
- ~/-
0.6m

t
I.I02in.
\

,
~-
\
I
I
0.2m
./

0.1 m
['1 ....... ' .

Figure P7.67, P7.68


24 in.

-'e-0798i"
7.69 Show that the moment of inertia Ixc ofthe hexagonal
area in Figure P7.69 has the value indicated.

---7 7.70 In Problem 7.69 find IyC and then note that it is the
x same as Ixc'
,Figure P7.61, P7.62

7.63,7.64 Hexagon, side length s

Xc

~ I =-s
513,
~s __ 1 Xc 16
X

FigureP7.63, P7.64 Figure P7.69

* Asterisks identify the more difficult problems.


492 Chapter 7 'Inertia Properties of Plane Areas

y 3+in.
1.
,!D.
,!D._,_
3. -.

-I
6 in. 6 in.
C

~c y

1.
,!D.

f--4in.-1 x

o 1~3in.--1
FigureP7.71
Figure P7. 73

Figure P7.74
y

I
( 'BH3
-r 1=-
4

6 in. ~
IOmm •

I
'",
_t_
t
2 in. t -.-
lOrAm

IOmm
_t_
t x

I-I~..----I--I I-B-I ,i,

Figure P7. 76
2 in. 3 in. . 3 in. 2 in.
FigureP7.78 .
8mm
ITITI\I
IOmm 8mm
Figure P7.79
7.71 Find the moment ofinertia lxc of the cross-sectional
area of a steel angle beam. Assume a constant thickness of
i in.,and compare your answer with the actual (consider-
ing rounded corners and fillets) value of 21.1 in.4 The x and
y coordinates of C in Figure P7. 71 are (1.03, 2.03) in.
FigureP7.80
7.72 Find lyC in Problem 7.71. Compare with the table
value of 7.5 in.4 '.
7.73 Find lyC for the cross-sectional area of the channel

beam of Example 7.12. Do it in two ways as was done in the


example in finding f"c' (See Figure P7. 73.) 7.78 Prove that the mornents of inertia of the triangular
area about the two indicated lines have the values shown
7.74 Find lxc for the shaded Z section in Figure P7. 74.
in Figure P7.78.
7.75 Find lyC for the area of Problem 7.74.
7.79 In Figure P7. 79 find the moment of inertia of the
7.76 Find the moment of inertia of the composite area shaded area about (a)the x axis; (bl the yaxis.
about the x axis through the centroid shown in Figure
7.80 In Figure P7.80 find the polar moment of inertia
P7.76.
with respect to C of the annular area by (a)integration;
7.77 Find the moment of inertia of the composite area in (bl composite areas. (c)Show that your answer approaches
Problem 7.76 about the y axis. 2nR:t as Ri -+ Ro' where t is the thickness (Ro - RJ
r
t 7.4 The Method of Composite Areas 493
r
!
Figure P7.81 y y
I 0,606 in.

-1
0.390 in,
12.12 in.
X---1?;1----X X X
,
A = 19.1 in.2

0.606 in.
~.
+
1-12.00 in.I-- 1t y
I---Win.-I y

-,
Figure P7.82
y
0.650 in.
~ (average thickness) I in.-I f--

r=
T::&'%'

I
~ 5 in.
-- -0.716 in.
15.00 in. t
t
x x
Area = 14.7 in" ~
h , 7. 6 in.
lxx = 404 in,' ,In.

. ":~ , -0.799 in .
lyy = 11.0 in,'

-- _t
x

+
1-
-t 1-4in.-1 x
L
Figure P7.84

~
I. b -I t
3.716 in. Figure P7.83
b

FigureP7.86 H
t
7.81 A box beam is formed by welding together two steel
C 15 x 50 channels and two plates as suggested .by the ~ )
upper sketch in Figure P7. 81. The properties of an in- x
dividual channel are shown in the lower sketch': Find lxc
and lyC forthe area of the box beam. (Note: The designation error with a calculator, and note that there is only one
C15 x 50 means: C = channel section; 15 = nominal positive root.
depth in inches; 50 = weight in lb/ft.)
7.84 The two 6 x 4 x i angles are welded to the shaded
7.82 The W 12 x65 steel beam cross section shown in steel plate as shown in Figure P7.84. Find the moment of
Figure P7.82 has the centroidal moments of inertia lxx = inertia and radius of gyration of the combined area about
533 in.4 and Iyy = 175 in.4 Find the centroidal moments the x axis.
of inertia if two of these beams are welded together to
form the combined cross section. 7.85 Repeat Problem 7.84 for the moment of inertia about
the y axis.
7.83 (alFind Ix and ly for the section shown in Figure P7.83
in which t « b,b. (Neglect all powers of t higher than the 7.86 In Figure P7.86 show that the moment of inertia of
first.) (b)Find the ratio bib for which Ix = ly. Hint: You'll the trapezoidal area about ,the x axis is(3b + a1H3/12,
have to solve a cubic polynomial equation! Use trial and independent of the angles IXl and (X2'
494 Chapter 7 Inertia Properties of Plane Areas

y
._f
I 4 in.

T
.. 14.12 in.
J .
0.513m.
4 in .
3 in.

y
- -0.313 in.
Figure P7.87

I XI
I
y
.
1--4in.--1
---6.776 in.- Figure P7.91

Figure P7.88

7.87 In Example 7. 15 aSsume that four isosceles triangles values of 386 in.4 and 5.88 in. (They will differ slightly
form the fillets that give the difference between the true due to fillets.)
area, 24.7 in.2,. and the 24A-in.2 area of the web and
7.89 Repeat Problem 7.88 for axis Y-Y. The table results
flanges. (See Figure P7.8 7.) Show that these triangles
this time are 26.6 in.4 and 1.54 in.
(a)contribute the difference between lxc in the AISC
manual (928 in.4) and in the example (916 in.41 and that 7.90 Find the polar moment of inertia of the area of
(b)the triangles contribute negligibly to lyC so that the Problem 7 Al about its centroid.
manual and example values (225 in.4) should in fact agree
as they do. 7.91 Find the centroidal polar moment of inertia of the
bolt pattern in Example 7.11 by direct calculation using
7.88 A W14 x 38 (meaning the depth is about 14 in. and :EAjr?, where Ii = distance from Aj to C. Your answer
the weight per ft is 38 lbl wide-flange beam has the cross should, of course, agree with Ie from that example. See
section shown in Figure P7.88. Calculate the moment of Figure P7.91.
inertia about the centroidal axis X-X and the radius of
gyration about this axis. Compare your answers with the 7.92 Find the polar moment of inertia of the area of
table (American Institute of Steel Construction Manual) Problem 7.79 about the center of the hole.

.~7.5" Products of Inertia of Plane Areas


Suppose our area A of interest is again in the xy plane. Then the product
of inertia with respect to the axes x and y is defined to be*
y

X
Ixy = - L xydA

*Some authors define Ixy without the minus sign. The reason for our choice here is
X noted at the end of Section 7.7.
7.5 Products of Inertia of Plane Areas 495

Thus the product of inertia is a measure of the imbalance of the area


with respect to the two axes. Two important special cases are:

a. If y is an axis of symmetry, then IXY = 0 (see the illustration). This is


because each dA has a mirror image on the other side of the y axis so
that xy dA and (- x) y dA add to zero. Over the entire region, then,
, the integral vanishes.

t, y
~
f
I
I

1
!
}:
71r,

b. If x is an axis of symmetry, then Ixy again = 0 (see the illustration).


This time, each dA has a mirror image on the other side of the x axis,
and xy dA plus x (- y) dA add to zero. Ixy thus again equals zero.

Frequently, however, we have to deal with unsymmetrical areas for


which IXY might not be zero. Four such examples follow.

E x A M p L E 7.~6~
Find Ixy for the shaded area in the figure.

(Continued)

L
SOLUTION
Letting band h be the base and height of each rectangle, we have

Ixy = - f AI
xy dA - f A2
xy dA

= - f~or~o xy dx dy - f:~-hJ:~-b xy dx dy
20
= f. h _ x21b Y dy _ fO x .1 y dy
y~O 2 0 y~-h 2 -b

= _b2 y21h + b2 y210 _b2h2 _ b2h2


2 2 0 2 2 -h 4 4

or

E x A M p L E
Find Ixy for the shaded area in the figure.

.,1.,

SOLUTION

Ixy = - f xydA

I"y = -
1 H .f~(H-Y)
y~O x~O
xy dx dy = -
1
y~O
H
x2.1~(H-Y)
y-
2 0
dy

496
E x A M p L E 7.18~
Find the centroidal product of inertia IxyC for the thin strip shown in the figure.
The strip lies in the xy plane.

SOLUTION

We shall use the polar coordinate r as shown in the lower sketch. The differential
area will be w dr:

f f
LI2 -

IxyC = - xy dA = - r~ _~ (r cos f3)(rsin fJlw dr


2

f
LI2
= -w sin 13cos 13 r2 dr
-L12

-wL 3 sin 13cos 13


12

-wL3 sin 213


24

y(ft)

Y = 3sin -(1l"X)
-
E x A M p L E 7.19~
_- 10 Find Ixy for the shaded area shown in the figute.
3ft

SOLUTION

iY~3Sin;~
f i
X~S
5 ft x(ft) Ixy = - xy dA = - xy dy dx
x~O y~ 0

xy213sin ;~ dx
-
i
x~O
s
-
2 y~O

--
i
x~o2
s x
9 sin2
nx
~
10
dx

(Continued)
,
\

t.:L 497
1.'
f'
:~

and using integration by parts,

I
XY
= -4.5 xilS + 2.25(X Sin(nX/5 1)IS _ 2.25 fS sin(nx/5) dx
4 0 nl5 <'I x=O nl5

-28.1 + 0 + 2.25(~r cos1nX/5{

-28.1 .•.. 11.4 = -39.5 ft4

PRO B L EMS / Section 7.5

7.93 Find the product of inertia Ixy of the shaded area in . 7.95 Find the moments and product of inertia Ix, Iy, and Ixy
Figure P7.93. of the shaded area shown in Figure P7.95.

7.94 In Figure P7.94: 7.96 In Figure P7.96 show that for the shaded triangular
area,
8. Find the product of inertia Ixy 6f the thin strip bent
into two quarter circles as shown.
b. Compare your result with that of Example 7. 18 for
the case in which f3 = nl4 and when L/2 =' ..,.97 Show that Ix = Iy = 2b41 3 and lxy = b4/2 for the
2nRl4-that is, when the lengths of the strips in shaded area in Figure P7.97.
the two problems are the same.
7.98 Findlxy for the shaded area in Figure P7.98.
y

x
x
Figu,re P7.95

FigureP7.93
Figure P7.94

Y Yi
b
4
b

)
x
b
.1 x 2 x
FigureP7.97 b Figure P7.98
Figure P7.96

498
r
t 7.6 The Parallel-Axis Theorem for Products of Inertia 499

y y

Figure .P7.99

Figure P7.1 00
x

,, 7.99 Find the product of inertia Ixy of the area shown in . * 7.100 By integration, find the product of inertia Ixy of the
\, Figure P7.99. ' parallelogram shown in Figure P7. 100.

" ,

The Parallel.Axis Theorem for Prod~cts of Inertia


Ii " There is a parallel-axis theorem, or transfer theorem, for products of
r
'~"

,.
..,
inertia just as there is for moments of inertia. It is
\, Ixyp = IxyC - Axy

The proof goes as follows: Referring to Figure 7.3,

.. Ixyp = - f xydA = - fIX + xdW +yddA

~-
~.'
-".
Ixyp = -x f YI dA - yf Xl dA - f XIYI dA - xyJ dA
L.

Figure 7.3
-~::,.-:_--------------------------------------------------_ ...

500 Chapter 7 Inertia Properties of Plane Areas

The fi~st two integrals vanish by the definition of the centroid,


leaving the desired theorem:

y E x A M p L E 7~20~.
Obtain the product of inertia Ixy for the ~rea of Example 7. 16 by using composite
areas and the transfer theorem.

h
SOLUTION
x
h

b2h2 b2h2
-~---- (as before)
4 4

x p L E 7~21~
r
E A M
Find IxyC for the shaded right triangular area shown in the figure.
H

: H
1-
1
_. "3
x
-t.~~~
(Continued)
SOLUTION
_B2H2
From Example 7.17we had Ixy = --~. Using the parallel-axis theorem,
24

Ixy = IxyC -:- AXjT


_B2H2
I = -_. _
..- + -BH -H ~
B B2H2
.= --
xyc 24 2 3 3 72

.: ~,

"

A M p L E 7.22~
Determine the centroidal product of inertia I"y for the "zee" section shown in the
figure.

~4in.~1
r--
8.5 in.
)
x

SOLUTION.
We shall label the three parts of the "zee" as inditated. Using composite areas
together with the parallel-axis theorem,

Ixy- - I xyCD + I@.IQ)


xy .. + xy

= [0 ~ 0.5(3.5)(3.5 + 0.5)(4'J + 0 + .[0 -, 0.5(3.5)(.~3.5 ~ 0.5)(-4)J


'--.--' 2 '-.-' ,---, 2 . ...........,
~~ ~
ACD YCD AQ) YQ)
xCD xQ)
= -14.0 - 14.0

Ixy = - 28.0 in.4

{Continued)

I. 501
L
Note that the centroidal axes of CD, @, and CID, which are parallel to x and y
through C are axes of symmetry for the respective parts. Thus each centroidal
product of inertia is zero, leaving only the transfer terms. Furthermore, even the
transfer term vanishes for@becauseitscentroidisthesameasCsothatx2 andY2
are zero. As expected, then, the only contributiorts to IxyC are due to the areasQ)
and CID being in the "opposite quadrants" (first and thirdl so that their transfer
terms have the same sign.

PRO B L EM S / Section 7.6

7.101 What is the product of inertia IxyC of the Z section


shown in Figure P7. 101?

7.102 Find the product of inertia lxy for the hollow


T
3in.
rectangular area of Problem 7.42. ~.
)
7.103-7.114 Find the products of inertia lxy for the 12 x
areas in Problems 7.45-7.68, respectively. Figure P7.115

7.115 Find the centroidal moments of inertia and lYe lxc


and the product of inertia IxyC of the composite area shown
in Figure P7.ll5.
y
7.116 Repeat Problem 7. 100, this time using the transfer
a3b
theorem andthe.'fact that IxyC = ~--sin2!J. cOS!J..
14

7.117 Use the results of Problems 6. 18 and 7.95 to find


the moments and product of inertia lxc' lyC' and IxyC for the
shaded area in Figure P7.117. x

7.118 Find lxy for the shaded area in Figure P7. 118. (Make .Figure P7.117
use of the parallel-axis theorem'l

y
1----35 in.-~I.

fin.t l -1
3 in.
31 51
1 6 in.
_I
__ .L---=)"-

1--/--1 I x
I
---3/-

Figure ~7.1 01

502
7.7 Moments and Products of Inertia with Respect to Rotated Axes Through a Point/Mohr's Circle 503

ti..
~7.7 Moments and Products of Inertia w.th Respect
to Rotated Axes Through a Point/Mohr's Circle
,
h.
Suppose we know the moments of inertia Ix and Iy with respect to the
i orthogonal axes x and y in the plane of an area A, and also the product of
t ~
\ inertia Ixy• It is possible to derive a pair of equations that will allow us to
j

,-I then find the moments and products of inertia associated with arbitrarily
I oriented lines through the point. We shall find these equations in this
r"' (,. section, and we shall also see that they may be combined to form the
equation of a circle(called Mohr's circle), whiCh is a handy tool in finding
inertia properties with respect to rotated axes.
We begin by using Figure 7.4 to derive the formula for Ix, (the moment
of inertia about the arbitrary axis x'). It is assumed that Ix, Iy, and Ixy are
known for the x and y axes at P.

x x

Figure 7.4

Using the definition of the moment of inertia, we obtain

Ix, = f y,2 dA '= f (y cos e- x sin e)2 dA

in which y' is seen in Figure 7.4 to be (y cos e- x sin e). Expanding,

,1 Ix. = cosz e f yZ dA- 2 sin e cos e f xy dA + sinz eJ X


Z dA

or, recognizing the definitions of Ix, Iy, and I;y,


Ix, = Ixcosze+ IysinZe + 2Ixysinecose (7.7)
Using the following trigonometric identities in Equation (7.7), Ix, can
be expressed as a function of the angle 2fJ:
1 + cos 2e
cos z e = ~~~--
2
1 - cos 2e
sin z e = ~---- 2

sine cos e = -'sin'-2-2e


" .
504 Chapter 7 Inertia Properties of Plane Areas

We thus obtain

(7.8)

We will also need Ix'y' in terms of Ix, Iy, andlxy:

Ix'y' = - f x'Y' dA

-f (y sin () + x cos ())(y cos () - x sin ()) dA

= sin ()cos ()[ - f (y2 - x2)dA ] + :cos2 () ~ sin2 (){ - f xy dA ]

cos 2()
or

I ' , = -
x y (I - I)
_x__
2 y sin 2() ,+'I-.'cos
xy
2-(),. (7.9)

Equations (7,8) and (7,9) are the transformation equations that give
the inertia properties for rotated axes. They Can be manipulated into the
equation of a circle by squaring both sides of each and adding:

_(Ix
[ Ix,
+
2.
Iy)J2 + 2
Ix,y' _ (Ix - Iy
-
2
)2 + Ixy2 (7,10)

This is the equation of a circle, which the reader may recognize more
easily in the form

(x - xOl2 + y2 = R2

The abscissa (x) is seen to be the "moments of inertia axis," while


the ordinate is the "products of inertia axis." Thus a point on the circle
has the coordinates (Ix" Ix'y')' The center C of the circle is located at

ex : 0) ly,

while the radius R is given by

R= Ix - Iy
( --2-
)2. ..+ I 2xy

Therefore, Mohr's circle appears as shown in Figure 7,5.


Now that we know how to draw Mohr's circle, let us proceed to
demonstrate how to use it. Suppose we wish. to know lal the mornent of
inertia of an area about an axis x', located angle () ahead of x as in Fig-
ure 7.4, and/or (bl the product of inertia Ix'y' with respect to x' and y;
(again refer to Figure 7.4). We could, of course, use Equations (7.8) and
(7.9) tofind these inertia properties directly. But some find it easier to
r-
7.7 Moments and Products of Inertia with Respect to Rotated Axes Through a Point/Mohr's Circle 505

j'
i
i Figure 7.5 Mohr:s circle.

remember, and to make. use of, the circle that was derived from these
two equations. Point A in Figure 7.5 has coordinates (Ix, Ixy), which
corresponds
.
to x' lining-.'
up with x, and to y'.. '.twith. y(f) = 0).
- .
Point B has
the coordinates (Iy, - Ixy), which corresponds to x' lining up with y, and
hence y' with - x (f) = 90°). These points are 180° apart on the circle
because of the "2f)" in the equations-all the angles on the circle are
double what they are on the actual area. To find Ix,for an arbitrary axis x',
we rotate, from CA, through 2f) in the opposite direction (i.e., -.:> ) on the
circle. * We then reach the point P (see Figure 7.6) whose coordinates are
(Ix" Ix'y')' This is all there is to it, but before using this simple procedure,
we need to prove that the point so identified gives the correct answers for
Ix, and Ix'y,-that is, that the coordinates of the point P thus identified
agree with Equations (7.8) and (7.9).

t.o:"
,.

f 1

Figure 7.6

*If the product of inertia axis is plotted downward instead of upward, the rotation
;' ..... directions on the circle and the actual area will then be the same.

tL
.....••...
~..
I

P<;;v:i _
506 Chapter 7 Inertia Properties of Plane Areas

To do this, we first write the lx' coordinate of point P, using Fig-


ure 7.6:
Ix' = center + (radius) . [cos(24) - 28)]

y
Ix' = (Ix-2-.+ I ) + R(cos 24>cos 28 + sin 24>sin 28)

y
Ix +I ex ~ I ) . Ix
Ix' = ---y + R ---- cos 28 + R 2 sin 28
2 R R

This is the same equation as (7.8); therefore, the procedure has


yielded the correct answer, in general, for Ix" Next we check Ix'y' , From
Figure 7.6 again,
Ix'y' = (radiusHsin(24> - 28)] = R{sin 24>cos 28 - cos 24>sin 28)

= R Ixy cos 28 - R --2-.-


ex - Iy)
sin 28
R R
y .
Ix'y' = Ixy cos 20 - (Ix--2-- I ) sm
{J
28

which agrees with Equation (7.9). Therefore, reiterating, mice the circle is
drawn we can find the values of Ix' and Ix'y' at any angle ~ with the
x axis by rotating on the circle through 28 ~ from the reference line CA
and then writing down the coordinates of the point thus located.
We now work an example problem using Mohr's circle, which was
named for a German engineer, Otto Mohr IL835-1918), who discovered
its relationship to Equations (7.8) and (7.9).

E x A M p L E
Use Mohr's circle to find the moments of inertia of the cross-sectional area shown
ex
in Figure 1 about the (J. and f3 axes through its centroid C. The thickness t is very
(3
small compared to b.
Figure 1

(Continued)
SOLUTION
The steps in this solution will be:

1. Find the centroid C.


2. Find T", Iy and Ixy•
!

3. Use the transfer theorems to get IxC IyC and 'IXyc. ! !

4. Draw Mohr's circle and use it to obtain I. and Ip•

(II The centroid: Let the horizontal rectangle have area A, and the vertical
one A2:

Ax = A,x, + A2x2 = (b - t1t(b - t + t) + bt ~


.22

b2t12 + bt2/2 - t3/2 b


x= 2bt _ t2 :;::: 4" (since t« bl

and
I
b
\ '" (by symmetry I
.
, y:;:::-
4

(21The inertia properties at 0:

3 3
I = leD + Ia> = (b - t)t '
+ (b - tIt (t)2
- + -tb + tb (b)2
-
x x x 12 2 12 2

CD
. (b - tlt3 tb3
---+-
3 3

:;:::-
3

And

(by symmetry)

Also!

b-t)t .tb
Ixy = 0 - (b - t)t ( -2- + t "2 + 0 - (btl "2 "2

CD
which we shall neglect since t « b and the largest term is a "t2 term."
(31The inertia properties at C:

(Continued)

507 _.
so that

(by symmetryl

or

The reader may wish to check these values by transferring the inertia properties
of CD and @ directly to C.

A
3

-3 ---------
B

Figure 2 (4) We now use Mohr's circle to get the inertias about the (J. and fJ axes
b3t
at C. Factoring -, the circle is drawn as shown in Figure 2 in which:
24

a. Center =
I +I
xc
2
yC
= 5
(b24t)
3

b. Radius = R = Ixc - Iyc


( ~-- 2
)2 + I x~.
2
= (b3t)
3- 24

Point A represents the values (Ixc' Ixycl. while point B represents (IyC' - Ixyc)'
To get from x to (J. on the actual area, we rotate 45° 5. Thus Wego 90° +'
on the circle, and reach point D of Figure 2, where the moment of inertia is
b3t b3t
maximum, equaling Ix, = I. = center + radius = 8 - = -, and where
24 3

Next, to get to fJ, Wecontinue 90° 5 more o~ the actual area, therefore 180°
more +' on the circle. This puts us at point E, where Ix, = Ip = center - radius
b3t b3t .
= 2 - = -, and where again Ix'y' = O.Note that the area is distributed fairly ')
24 12 ;;
close to the fJ axis through point C, so that it is not surprising that Ix, is minimum
(see the circle) for this line.

508
7.7 Moments and Products of Inertia with Respect to Rotated Axes Through a Point/Mohr's Circle 509

In general, Ix will not equal Iy, so that calculation of the desired Ix"s
will be slightly more complicated than in the preceding example.
However, that example has served to ease us into the use of the circle,
and has given us a feel for the axes of maximum and minimum moments
of inertia through a point. For these two axes, called the principal axes,
the product of inertia always vanishes. The associated points on the Ix'
axis (abscissa) that are farthest to the left and right on the circle give us the
principal moments of inertia. The circle clearly shows that they are the
minimum and maximum moments of inertia at the point and that, for
their associated axes, the products of inertia vanish. This has very
important implications in the mechanics of deformable solids.

a
E x A M p .L E 7.,24~
For the rectangular area. shown in Figure l,lind tn~ values of I~, fp, and I«p at the
origin (corner 0).*

SOLUTION
, First we find Ix, Iy, and Ixy while reviewing the use of the transfer theorem. We
'.
recall that.
hb3 6(23)
I
Yc
= -
12
= -" -'
12
= 4 m.. 4

o 2in. x bh3 2(63)


I = - = --. = 36 in.4
Figure 1 Xc 12 12

IxyC = 0 (by symmetry)


Transferring these centroidal inertia properties to the corner 0, we obtain

I Xo = IXc + Ad2 = 36 + (12)32 = 144 in.4

IYo = IYc + Ad2 = 4 + 1I2112 = 16 in.4

IxyO = IxyC - AX}' =0 -(12)(+ 1)(+31 = -36 in.4

Now we are ready to construct Mohr's circle. We need:


1. The center; it is at

ex; y
I , 0) =C 44
: 16,0) = (80,0) in.4

'. ~ (Continued)

*We shall start with.five-digit.accuracyin this example so as not to obscure certain


'results with roundoff error.
2. The reference point A, corresponding
-36) in.4
to 0 = 0, is a t II.x, I.)
xy = (144 ,
.l"
"

~.

,t!
~

2,p
'Z
(1., I,y) = (144, -36)
Figure 2

Therefore the circle is constructed as shown in Figure 2. We see from the


circle that
R = .)(144 - sW +(36)2

= .)(6412 + (36)2 = 73.430 in.4


and

Next, the line ex is seen in Figure I to make an angle with the x axis of:

o = tan- J
(D = 71.565°

Thus 20 = 143.13°, and we turn on the circle through 20";:J (opposite to


the 0 direction I~) on the actual area). We reach the point Q on the circle whose
(Ix" Ix'y') coordinates correspond to Ia and laplsee Figure 3). The answers for Ia and
laP are readily and simply written down with the help of Mohr's circle:

Ix, = Ia = center - R cos 7,51 ° = SO - 73.4(0.991) = 7,26 in.4


Ix'y' = laP = -R sin 7.51 ° = - 73.4(0.131) = - 9.59 in.4

7.51 °
,
I

Figure 3

(Continued)

510
The value of Ip corresponds to the abscissa of the point Pon the circle 180°
away lor diametrically opposite) from Q:

Ix' = Ip = center + Rcos 7,51° = 80 + 73.4(0.991) = IS3in.4


It is interesting to note that ~ (the diagonal of the rectangle) is not the line of
minimum moment of inertia. That line is loeated 143.13 + 7.51 = 151°
clockwise from CA on the circle; hence 75.5° counterclockwise from x on the
actual area (see Figure 41.

Figure 4

Similarly, to get to the axis of maximum moment of inertia, we tum either


29.36° fronYCAon the circle or 151 + 180 = 331° <=' from CA on the circle.
Both these rotations (which, rounded, add to 360°)will reach the point farthest to
the right on Mohr's circle-clearly the point with the largest Ix, ,as we have seen
before. It has the value of Ix, = center + radius,= 80 + 73.43 = 153.4 in.4,
and is located 90° away froin the previously found axis of rttinimummoment of
inertia, (SeeFigure 5.)
Note that ()! and (}2 in Figure 5add to 180°, and thus they locate the same
line.

Axis of maximum
moment of inertia
/
28, = 29.36°
x
,." 29°
81 = 29.36° == 14.68,." 150
2
Figure 5

The reader may also come across Mohr:s circle again" but in a dif-
ferent context, when studying the mechanics of deformable, solids. In
that subject, two quantities (stress and strain) each have transformation

511
512 Chapter 7 Inertia Properties of Plane Areas

formulas identical to Equations (7.8) and (7.9). Therefore, they too have
Mohr's circles, by the same derivation as we have presented for inertia
properties. Actually, the equations are the transformation formulas for a
mathematical entity called a second-order tensor; inertia properties,
stress, and strain are thus three examples of second-order tensors. It is a
fact that without the minus sign in the definition of the product of inertia
J
(Ixy = - xy dA), the inertia properties do not obey the tensor trans-
formation equations, and this is why we have included it in the
definition.

PRO B L EMS / Section 7•7

7.119 In Figure P7. I 19 find the principal moments and a. Use Mohr's circle to compute the principal moments
axes of inertia through the centroid of the shaded area by and axes of inertia ILe., the maximum and minimum
using Mohr's circle and the results of Problem 7.117. moments of inertia and their associated axes) at C.
b. Give an argument that at the center C of the square in
7.120 Shown in-Figure P7.120isthe cross.sectionofa
Figure P7.121 (b),the'Mohr's circle is just a point, and
length of 3 x 3 x !"angle iron." Find the exact values of
that therefore the moment of inertia about all lines
I. and If! and compare the results with those of Example
through C is h4/ 12, with the product of inertia
7.23 in which the thickness was neglected.
vanishing for all pairs of axes there 90° apart.
7.121 From Examples 7.8 and 7.21 we have these results c; Using Part (b), argue that the moments of inertia of
for the isosceles right triangle shown in Figure P7.121Ial: the isosceles right triangle shown in Figure P7. 121lc)
about the indicated :,!xesare as given in the figure,
h4 with the product of inertia associated with these two
and I =-
72
XYc axes vanishing.
d. Transfer the results of Part (c)to the centroid C of the
triangular area [Figure P7. 121(dlL and show that the
Y
answers agree with those of part (al.

Figure P7.119

x h
4in: (al x

h
Yj I.
:lIn.

~.
24
(3

h
h
~ in .

) h h h
--7
3in. x (bl Ie) (d)
Figure P7.120 Figure P7.121
- -<.r.
rh.:; 7.7 Moments and Products of Inertia with Respect to Rotated Axes Through a Point/Mohr's Circle 513

y'
20

\ .FigureP1.124

Figure P7.122

-1----3.5 in,--"I
J.
-In.
4
T
.

h
t
3in ..
x

h 1 6 in.

~3 .
'4 In.

b
1~3.5 in.~I-t-
Figure P7.125
>x
Figure P7.123. Now use Mohr's circle to find the max-
imum and minimum moments of inertia at O. and their
associated axes. You should find that these axeS are 4 and 12
Figure."7.123 in the figure. Explain why these lines make sense. Also
note that your results are larger and smaller than ~b4 as
they must be.
7.122 In Figure P7.122, using Mohr's circle, derive a
e
formula for the angle through which we must turn from 7.124 Prove with the help of Mohr's circle that Ix+ly=
the x axis in order to reach the axis of maximum moment of Ix. + I y' for the plane area shown in Figure P7. 124.
inertia. The angle should be in terms of I", Iy, and Ixy• Test
your equation on" the'bottom"'sketch shown' in Fig" 7.125 Using Mohr's circle and the results of Problems
ure P7.122 for. which Ixc = IyC = h4/36 and IxyC = 7.74, 7.75, and 7.101, find the principal moments and axes
h4/72, and for which line 1 is the axis of maximum of inertia at C for the Z section shown in Figure P7. 125,
moment of inertia. 7.126 Find the smallest moment of inertia for any line
7.123 In an earlier problem, it was shown that Ix = drawn through C in Problem 7.72, Compare with the table
Iy = ~b4 and Ixy = b4/2 for the shaded area shown in _ valUe of 4.37 in.4

COMPUTER PR 0 B L EMS /Chapter7

7.127 Add to the computer program written for Problem is then to Use these twop10ments. of inertia to directly
6.131 the following feature: The program is to also read the calculate and print (a)the moments ohnertia Ix and Jy of the
centroidal moments of inertia Ixieand IYie of each area Ai' It total area about the x andiy axesiand (b) the moments of
. -If"', ::1

~:-<~:
~ ... _-_.
514 Chapter 7 Inertia Properties of Plane Areas

inertia lxc and lyC of the total area about axes parallel to x Write a program that will use these results, plus the
and y through the (calculated) centroid. Also 'to be printed parallel-axis theorem, to place these three areas side by side
are the results lxc + Ay2 and lyC + Ax2, where x and yare .in the six possible ways shown below and compute lyo
the coordinates of the centroid C. These results should about the left vertical edge. Take h = 2R = b = 10 in.
match Ix and ly if everything is done right. Test the program The program should contain an algorithm that does
with the example from Problem 6. 131. all six problems; i.e., .yow must teach the computer to
permute the-bodies and do the transferring of moments of
7.128 The first three examples in Chapter 7 resulted in
inertia for each permutation.
the following centroidal moments of inertia:

b b

h
-r
h

hb'
1 =_.- hb3
L
y 12 1,;= 48

Figure P7.128

Answers to Questions '.Chapter 7


Q7.1 No, it was not necessary, because if we change the names of x and y, and of band
h, then the same integral as before yields lyC = hb3/12 without having to
integrate again.
Q7.2 No, because then yis not the same for every element of the differential strip.
Q7.3 It's constant.
Q7.4 Ip = lxp + lyp = Jxc + Ay2 + ly~ +.AX2 = (IXc + lyc) + A{x2 + y21 = Ie + Ad2 -.}

Q7.5 Working from the answer to Example 7.3, to make the notations and geometry
agree, we must do three things: (1)swap band h; (2) double the old h; and (3)use
3
half the answer. Thus lin Example 7.8 is' .[b(2h)3JI
.. -.--.' bh whIch
2 = -.-, .. checks.
x . 48\ 12'
Q7.6 No! We can only transfeno or from an axis through the centroid.
wt3 . At2 . . .
Q7.7 Because lSTRIP= -- = _..-. much smaller than AD2 because the stnp ISgIven
Xc 12 12 I

to be very thin.
Q7.8 Because the fillets are so close to the y axis and so relatively far away from the
x axis, they will contribute, relatively, a lot to IxJimd very little to lyc'
Review Questions 515

Q7.9 Because r cos f3 and r sin f3 are not the Ix; yl coordinates of all the points in the
"dA." The smaller is w relative to L, the better the approximation.
Q7.10 No, because both signs change if we go the other way!

Q7.11 No.

2 ft

Q7.12 Yes; ex. appears to be the axis for which the area is distributed farthest away, in an
overall "r2 dA" sense. And by symmetry, I.p should vanish.

Review Questions I Chapter 7


True or False?

1 The moment of inertia of a plane area A, about any axis, is always a positive
number.
2 The dimension of a moment of inertia of an area is L 3, where L denotes length.
3 The polar moment of inertia of an area A about any axis (z) normal to the plane
(xy) of A through a point P in this plane is larger than each of Ixp and Iyp' and in fact
equals their sum.
4 Let the moment of inertia of an area A about the x axis through its centroid be
called Ixc' Consider the moments of inertia of A about all other axes parallel to x.
All these moments of inertia are less than Ixc'
5 The moment of inertia of a circular area about any line in its plane tangent to the
circle is inR4.
6 If we know the moment of inertia of a plane area Aabout an axis x in the plane,
then the moment of inertia about the parallel axis'through the centroid of A is
Ix + Ad2, where d is the distance between the axes.
7 The utility of the method of composite areas is significantly enhanced by the
parallel-axis theorem.
S The product of inertia of an area can be positive or negative, but never zero.
t3t
9 The product of inertia Ixy of the given letter "F" in the drawing is ~ - 2
where t is the thickness of the letter and t « t.
10 Mohr's circle is useful in finding the axes and values of the largest and smallest
moments of inertia of a plane area with respect to axes through any point .
.11 It is possible for Mohr's circle Ifor area moments oUnertia) to cross the ordinate
lproduct of inertia axis).

Answers: 1. T 2.F 3. T 4. F 5. T 6. F 7. TB. F 9. T 10. T 11.F


Special. Topics

~
I
8.1 The Principle of Virtual Work

8.2 Hydrostatic Pressure on Submerged Bodies l


~
1
.1..•.1

516
8.1 The Principle of Virtual Work 517

~8.1 The Principle of Virtual Work


In Section 3.4 we encountered bodies or structures composed of a num-
ber of rigid (or neaHigid) parts with speciarconnections such as pins.
When there is a large number of such elements, many free-body dia-
grams and associated sets of equilibrium equations may be required in the
analysis. Among the unknowns appearing in'these equations will be all
the forces of interaction between adjacent elements or members. It
sometimes occurs that most, if not all, of tl1ese interactions are not of
interest, arid we simply have to contend with them in order to get to the
results that are important. In such situations there is an equivalent and
economical form of analysis that can be used, and it is based upon what
is called the Principle of Virtual Work .. Before stating the principle we
must establish some preliminary concepts. For simplicity we restrict our
attention here to two-dimensional(plane) problems or situations.

Kinematics of a Rigid Body in Two Dimensions


In two dimensions the general change in configuration possible for a rigid
body is shown in Figure 8.1. The vector lOA is called a position vectorfor
material point A if point a is a fixed point in'the frame of referenceilOAi
denotes that position vector for the initial;location of A and lOAf the
position vector for the final location of A. The vector uA is called the
displacement that A undergoes when A m~ves from its initial to final
position. So, using the laws of vector addition, we see that
(8.1)

The rigid-body change in configuration may be accomplished by first


moving point A to its final position withoutchanging the orientation of
the body (this is called translation), and thert rotating the body about an

o
Figure 8.1
518 Chapter 8 Special Topics

axis (perpendicular to the plane) through A. The angle of rotation is cpo


The student should use sketches to verify that the same displacement UA
and angle cp yield the final configuration ihhe order of the operations is
reversed; that is, the body is first rotated about A and then translated to
the final configuration. Furthermore, the student should verify that the
same change in configuration could be accomplished by a translation
taking B to its finarposition followed by a rotation about B of the same
angle cpo Thus the most general change in configuration is characterized
by the displacement of some point plus a rotation about an axis through
that point.
Referring still to Figure 8. 1, we note that

and

Subtracting, and using Equation.(8.1J,


uB = uA + (rAB! - rAB,) (8.2)

From the preceding discussion we see that the term rAB! - lAB, is the
displacement that B undergoes when the body is rotated through angle cp
about the axis through A. This vector can be put into a particularly useful
form if the angle of rotation is infinitesimat in that case rAB! - rAB, is
infinitesimal and may be written as the differential drAB. To evaluate it
we refer to Figure 8.2 (where 1and J are fixed in the frame of reference).
e
The angle describes the orientation of the rigid body in which A and B
reside, and a change in 0 thus represents a rotation of the body. We see

L
from Figure 8.2 that
(8.3)

where we note that IrABI can't change since the body is rigid. Thus with a
i
differential change in 0 we find
drAB = IrABI(
-sin 01 + cos oj) dO
Figure 8.2
which may be e.l.'pressedas*
=lrABlldO)k x (cos 01 + sin oj)
= (dO)k x rAB (8.4)

The vector (dO)kis called the infinitesimal rotation vector and it can
be seen to conform toa right-hand rule. That is, if the fingers curl in the
direction of rotation, the thumb points in the direction of the vector. It is
important to recognize that drAB is perpendicular to rAB .

* An expression of similar form holds in three dimensions, but it is more difficult tq


derive and to visualize. It is for this reason that the development of this section is restricted
to two dimensions.
8.1 - The Principle of Virtual Work 519

We may now retUrn to Equation (8.2) to write

DB = UA + (de)k x rAB (8.5)

when the body undergoes an infinitesimal rotation de. If DB and DA are


infinitesimal,then they are differentials ofrOB and rOA and
(8.6)
This equation will be of considerable importance to uS shortly, but first
we must examine the concept of work.

Work.of a Force .and Work of a.Couple for


Infinitesimal Displacements
The increment of work done bya force F acting on point P of body B
during the infinitesimal displacement drop (see Figure 8.3) is defined to
be the scalar
dW = F . drop (8.7)

in which rap is a position vector of the point P in the inertial reference


frame !J, and where it must be stipulated that F is constant (or at most
changes infinitesimally) during the displacement. Note that the defi-
nition [Equation (8.7)] tells us that only the component of F that lies
along the displacement drop will contribute to dW; thus this mechanical
definition of work is different from our ordinary, everyday perception. *

Figure 8.3

Also, note that if the smaller of the two angles between F and drop is
less than 90°, F does positive work during the displacement drop [see
Figure 8.4(alJ. If the smaller angle is greater!han 90 'however [see Fig-
0
j

ure 8.4 (b)],then the work of F during the displacement drop will be nega-
tive. We see then that, speaking loosely, we may say that a force does
positive work when it "gets to move in the direction it wants to./I

'-*Try telling a farmer who has carried al OO"lbsackof feed across a field that hehasn't
done any work!
520 Chapter 8 Special Topics

<d,m
~F

8 < 1r12 8> 1r12


F does positive work F does negative work
(a) (h)
Figure 8.4

Note too that because


dW =F. drop = F(I drop I cos 0) = (F cos 0lldropl (8.8)

we can view the work increment as either (a) the magnitude of the force
times the component of the displacement of P in the direction of the
force; or as (b) the magnitude of the displacement of P times the com-
ponent of F in the direction of the displacement.
If we write the two vectors in Equation (8.7) in terms of rectangular
Cartesian components parallel to directions fixed in .!J, then
F = F) + F3
rop = xi + yj
so that
drop = dxl + dYJ
The following equation is another way of writing the work, this time in
terms of the rectangular components of the force (see Equation 8.7):
dW = Fx dx + Fydy (8.9)

As an example, the work of the force of gravity (or weight) is, with J and
positive ydownward,
dW = mgJ 'droc= mgJ . (dxi + dYJ)

dW = mgdy (8.10)

which is the weight times the downward component of the displace-


ment of the mass center.
The units of work or energy are seep, to be the same as are the
moments of forces. In the 51 system the N . m is used for moment, and
8.1 The Principle of Virtual Work 521

the joule (J), which is 1 N . m, is used for work so as to identify the


nature of the quantity. Often this distinction is made in the U.S. system
by using lb-ft for moment and ft-Ib for work.
The increment of work done by a constant couple (of moment Ck)
acting on a body during an infinitesimal rigid change in configuration is
defined by
dW = Ck. [(dO)k]

= C dO (8.1l)
We see that the work is positive if the sense of turning of the moment of
the couple is the same as the direction of the infinitesimal rotation, and
negative otherwise.
For this definition to be useful, we surely expect the sum of the
increments of work of a pair of equal-in-magnitude but opposite forces to
equal the increment of work of the corresponding moment of the couple.
To establish this connection, consider the situation depicted' in Fig-
ure 8.S. The net increment of work.donerby the-forces F and - F is
dW = F . drOB + I-F) . drOA
i
= F • (drOB - droA)
and using Equation (8.6)
dW =F. [(dO)k x rAB]
Figure 8.5
= (dO)k • (rAB x F)

= (dO)k. Ck = CdO

where we have recognized Ck = rAB x F as the moment of the couple


composed of the pair of forces.

, Virtual Displacement and Virtual Work


I A virtual displacement is an infinitesimal displacement that a material

II , point might be imagined to undergo. The word virtual is used to convey


that this displacement is imaginary or fictitious; the point in question
might be "tied down" (fixed)so that no actual change in configuration of
"
i~ .
\ the body would cause the point to move. However, a virtual displacement
is a displacement that could take place 'if the:constraint were removed.
The symbol (j is used to communicate "virtual" (or imaginary) incre-
ments, so that (jrop stands for a virtual displacement and (jO stands for
a virtual rotation. Thus, in two dimensions, the most general virtual
change of configuration for a rigid body is characterized by a virtual dis-
placement of a point together with a virtuaJ; rotation. The virtual dis-
placements of two points A and B are related by
, (jrOB = (jrOA + ((jO)k x rAB (8.12)
,.
where we have simply substituted (j's for d's in Equation (8.6).
522 Chapter 8 Special Topics

(b)
(a)
Figure 8.6

The virtual work of a force is the work that would be done if the
point of application of the force were to undergo a virtual displacement
and the force were not to change during th~t displacement. Similarly the
virtual work of a couple acting on a rigid body is the work that would be
done if the body were to undergo a virtual 'rotation and the couple were
not to change during that rotation. ,. " .

Principle of Virtual Work


The Principle of Virtual Work may be stated as follows. If a body in
equilibrium in an inertial frame is given a virtual rigid change in configu-
ration, starting from the equilibrium configuration, the net virtual work
of all of the external forces and couples acting on the body in its equilib-
rium state vanishes.* We use the symbol bWto stand for the sum of the
virtual works of all ofthe external (equilibrium) forces and couples. Thus
the principle is
bW= 0
Before proceeding to a proof of the equivalence of the principle of
virtual work and the equilibrium equations, we illustrate this equiva-
lence by a simple example. A coffee cup B of mass m is in equilibrium
on a horizontal table top as shown in Figure 8.6. In the free-body dia-
gram (two-dimensionality has been assumedl we show the weight of the
cup and the reaction of the tablej this reaction is assumed to have a
line of action through B whose distance from the line of action of
mg is the unknown a. The problem, of course, is to find the unknown
scalars f, N, and a.
First let us give the cup an upward virtual translation, each point in
Figure 8.7
the cup having an upward virtual displacement E1 (Figure 8.7). We note
the virtual work of f is zero since that friction component of reaction is

*Whenthe body is caused to deform by a virtual change in configuration, the


appropriate generalization of this statement is that ,the sum of the virtual work bf the
external forces and the virtual work of the internal forces vanishes. Nontrivial applications
of this form are beyond the scope of this book.
8.1 The Principle of Virtual Work 523

perpendicular to the virtual displacement E10f B The virtual work of


~.'
N is NEI' and the virtual work of the weight is - mgE1 because the mass
center C, like every other point, moves upward. Thus
JW = NE1 - mgEI =0
;~
I

k (N - mg)E1 = 0
,,
Because E1 is arbitrary, we obtain
N - ing = 0

N=mg

This, of course, is what we alternatively obtain from the equilibrium


equation "f.Fy == O.This exercise is instructive with regard to the concept
of virtual work. The force Nwill vanish as soon as the cup breaks contact
with the table top/and so an actual upward displacement of the cup
would be accompanied by zero actual work done by N.

As an alternative to the above let us give the cup a downward virtual


displacement Ez; in this case (see the figure at the left)
JW == -NEz + mgEz = 0
or

N=mg (as before)


The instructive aspect of this part of the exercise is that the virtual
displacement Ez represents a change in configuration that is impossible
for the cup to undergo in actuality because it requires the cup to penetrate
the table top. Thus in forming virtual changes in configurations, it is not
necessary that we conform to actual constraints imposed upon the body.
A horizontal translation of the cup by a displacement E3to the right
yields
JW = fE3 = 0

or
f=O
which we recognize as the result of applying the equilibrium equation
"f.Fx = O.
Finally let us give the cup a virtual rotation (Je counterclockwise)
about A (see Figure 8.8). We note from the figure that point C will be
displaced horizontally, so no virtual work will be done by the weight mg.
:...--.
•..•. ----------------------------------------------------

524 Chapter 8 Special Topics

Point Bwill move downward a be, so no virtual work would be done by f


even if f were not already known to vanish. Therefore,
bW = -N(a be) = 0
or
a = 0
which we recognize as the result that would have been obtained by
applying the equilibrium equation ~MA = O.
Figu'; 8.8

This example has illustrated the way in which virtual displace-


ments differ from real displacements and the way in which virtual work
differs from real work. And we .see that application of the. principle for
three independent rigid changes in configuration has produced the same
results that would be obtained from the three independent equations of
equilibrium:
I.Fx =0 (gives f = 0)
I.Fy =0 (gives N = mg)

~MA = 0 (gives a =0)


We are now in a position to demonstrate the generality of this
equivalence.
Consider the body B to be acted upon by the external forces F1,
FZ1 •.. and the couples C)~, Czk, ... "as seen in Figure 8.9.

'L i

Inertial frame
of reference
Figure 8.9
8.1 The Principle of Virtual Work 525

Now let the body undergo a virtual rigid change in configuration


producing changes in the position vectors (lopt' lOP2' ... j of the pOints.of
application of the forces (call these virtual ~hanges (jlOp, , (jlOP2' ... j and a
change in the orientation of the body (jl1)k. Then the virtual work of the
Fj's and C's is

(jW = Virtual Work


= Fl • (jlOp, + Fz • (jlOP2 + ... (8.13)

+ Clk . (jl1)k + Czk . (jl1)k + ...


Relating the infinitesimal virtual displacements of Pl, Pz, ... to that of
an arbitrary point of the body, say Q,

(jW = Fl . (jlOQ + (jl1k x lQp,) + Fz . (jlOQ + (jl1k x lQP2l

+ ... + (Clk + Czk + ... ) . (jl1k


but because

Fl . (jl1k x lQp,) = (jl1k x lQp,) • Fl

= (jl1k. (lQP. x Fl)

we obtain

(jW = IFl + Fz + ... ) . (jlOQ + (jl1k . (lQP. x Fl

+ lQP2 X Fz + ... + Clk + Czk + ... )


(jW = LF . (jlOQ + (jl1k .. LMQ (8.14)

Since the virtual displacement (jlOQ a} point Q is independent of the


virtual angular displacement (rotation) (jl1k, each can be taken zero while
the other is not. * Thus (jW = 0 implies

LF = 0 and (8.15)

which are the equilibrium equations. Conversely, if LF = 0 and LMQ ==


0, it follows from Equation (8.14) that (jW = O. This establishes (for two
dimensions) the equivalence of the principle of virtual work and the
equations of equilibrium. This equivalence also holds in three dimen-
sions, but we have not attempted to establish it here because of the more
complicated kinematics of rigid bodies in three dimensions.
We now consider an example of the use of the principle of virtual
work to establish an equilibrium configuration of a (near-) rigid body.

*The virtual work vanishes for any and all virtual displacements.
E x A M p L E 8. 1l1li......-
Find the angle 0 for equilibrium of the uniform bar B shown in Figure 1. Both
planes are smooth.

SOLUTION

Figure 1 This problem illustrates the way in which w:e commonly use the principle of
, virtual work. We can frequently give the body a virtual displacement in which
one or more of the unwanted unknowns in the problem do no work and thus do
not appear in the equation bW = O. We do this here with a virtual displacement
in which A and B move along their respective planes.

'Figure 2

Our sought-after angle increases by bO,as seen in Figure 3. The normal


forces NL and NR do no virtual work because each is normal to the virtual dis-
placement of the point on which it acts (see Figures 2 and 3). Hence only gravity
will contribute to the virtual work, which vanishes:

bW = Fg • broc = a (ll

With the help of Figure 4, we find roc and thenbroc:


1 '~ ~
Figure 3 roc = ~ [sin(ep ~ eli + cos(ep ~ Oli]
2

1 ~ ~
broc = - [cos(ep - Oli - sin(ep - Olj] b(ep - 0)
2
y .j
x -bO
/' 1 ~ ~
/{
1
= -
2
[cos ep cos 0 + sin ep sin O)i ~ (sin ep cos 0 - cos ep sin O)iH-bOI

-I ~,',.,.',
= -'-
2
'3
bOIlscos 0 + !sin ~
e)i - (5
4
cos 0 -
3
5
~
sin Oji] ;~
Also, the gravity force, or weight, is

Fg = mg(-~1 - H)
Figure 4
so that, substituting into Equation IlL

1 bO
bW = mg-- [~(~
cos 0 + ! sin OJ- !(!cos 0 - ~ sin OJ] = 0
2

and since bO is arbitrary, the bracketed expression vanishes:

-
7
2 5 cos 0 + ~1sin 0 = a

526
8.1 The Principie of Virtual Work 527

The real advantage of virtual work is notcin analyzing a single rigid


body, * but rather in studying problems with systems of rigid bodies,
joined together in frictionless ways such as by (a) smooth pins or ball
joints, (b)inextensible, taut cables, or (c)linear springs. With (a)and (b),
we now proceed to show that in a virtual displacement that is consistent
with the connections, no net work is done on the two connected bodies by
the pins, ball points, or cables.
If.A and B are joined by a smooth pin or ball joint,.no net work will
be done by the forces F (exerted on.A at A) and - F (exerted on Bat B), as
shown in the illustration. This is because the virtual displacements of the
points A and B are the same (they are pinned together) while the forces are
equal in magnitude but opposite in direction (in accordance with the
principle of action and reaction): Thus,

W'ork of F on .A Work of - F on B Zero

We next let bodies.A and Bbe joined by a flexible but inextensible,


taut cable, with or without the pulley in between. (See the illustration.)
Let the unit vectors U II and vII be along the cabl~t as shown, with U.L and V.L
being normal to the cable, in the respective directions of the normal (to
the cable) components of the virtual displacements DroAof A and DroB

~ .

I
I *For the remainder of this section we shall use the term' 'rigid body" as a shorthand
t expression to specify that the body is near rigid and that it is to be subjected only to rigid
I virtual changes in configuration.
f
tNote that ull = vII and U.L = V.L if there is no pull~y.
I
"j

L
528 Chapter 8 Special Topics

of B. Then the virtual work done by the cable on the two bodies is

bW = -Tull '(brOAllull + broA.lu-tJ+ TVII . (brOBllvll + brOB1v-t)

= - TbrOAIl + TbrOBIl (8.16)

= T(brOBIl - broAIIJ = 0

because the components of virtual displacement of A and B along the


cable must be equal if it is inextensible (and, of course, taut).'

If we place a spring in the cable as shown below, then

need not vanish, because the spring can stretch. In fact, if we let S be the
stretch of the spring, we obtain

the "virtual stretch" of the spring. For a linear spring, the tension in the
spring (and cable) is kS where k is the spring modulus, or stiffness, and
Equation (8.16) yields

bW = -kSbS (8.17)

This means that:

(a) Assuming S > 0 (spring is stretchedJ, then if

(i)bS> O,bW < 0


(ii) bS= 0, bW = 0

(iii) bS < 0, bW> 0


8.1 The Principle of Virtual Work 529

(b) Assuming 5 < 0 (spring is compressed), * then if


(i) <55 > 0, <5W > 0
(ii) <55 == 0, <5W = 0

(iii) <55 < 0, <5W <0


Of course, if 5 = O~that is, the spring is unstretched-then it will do
no virtual work. In the various cases above,,,the reader is urged to note
carefully that the spring does positive virtual work during the virtual
displacement if it gets to return toward its unstretched length, and
negative virtual work if it is forced farther away from it.
When we consider two or more rigid bodies connected by frictionless
pins or ball joints, and/or by flexible, inextensible cables, we see that the
virtual work done by the external forces and couples on the combined
a
system of bodies is zero (as was the case for single rigid body). This is
because <5W = 0 on each body and thus it also adds to zero on the
combination. Moreover, the virtual work done by a pin (or ball joint or
cable) on one body of the connection cancels its work on the other
provided the virtual displacements are consistent with the connections,
as we have shown. Thus such an internal interaction need not even be
considered in the equation <5 W = O!This is the huge advantage in using
virtual work: the non-working internal forces can be ignored.
For a linear spring performing non-zero virtual work on a system of
rigid bodies, we have seen that its virtual work is - k5 <55. If the spring
connects the rigid bodies .A and B, then we get the result shown in the
following equation:

IlW + IlW + IlW + IlW =0


(on ..A except (of spring on ..A ) (of spring (on /J except
for spring) on il) for spring)
o o
-kS1l5

This way of computing the virtual work done by the spring, as well as the
ignoring of non-working internal and reactive forces in computing <5 W,
willnow be illustrated by some examples. The reader is urged to mentally
consider the difficulty of solving these problems with a set of equilibrium
equations. And to remember that, if any of th~ forces acting on a body was
omitted from the free-body diagram and thence from the equations of
equilibrium in Chapters 3-;:-5, our solution was doomed!

*Of course, a transversely flexible cable (or cordI won't support compression.
However, a special case of the above analysis is that where the cable and pulley are removed
and the ends of the spring are attached directly to bodies ...Aand B. In that circumstance we
can have a compression-supporting spring connection.
E x A M p L E

Find the relationship between the forces P and mg and the angle 0, for equi-
librium. Neglect the weights of the four bars and the sizes of the two rollers.
(See the figure.)

SOL UTI 0 N'


For a virtual displacement (which will be described below), the virtual work will
vanish:
bW = 0 (II
From the equilibrium configuration, we imagine a virtual displacement bx, a
slight movement outward of the two points where the forces Pare .applied. Then
the virtual work of these forces is 2 [P bx].
The only other force that will perform virtual work is the weight mg. We
must determine how much mg' 'moves" in response to the imagined bx. From the
sketch, we have x = a cos 0 so that

bx = - a sin 0 150 (21


y -J.
.j

which relates bx and the corresponding virtual change 150 in 0 according to the
rules for differentiation and the formation of differentials. Similarly,

y
so that
= (a + bl sin 0 + b sin 0 = (a + 2bl sin 0
t,
..
i'
by = (a + 2bl cosO b8 $

which is the virtual displacement of the mass center of the weight. The virtual
I
1
work of gravity is then mg by. Thus Equation (1) becomes

2Pbx + mg(a + 2b) cos 0 b8 = 0

and using Equation (21,

2P(- a sin 0 1501+ mg(a + 2bl cos 0 150 = 0

or

[- 2Pa sin 0 + mg(a + 2bl cos 0] 150 = 0

(Continued)

530
Since (jO is arbitrary, the-term in brackets must vanish, giving

mgla + 2b) cot 0


P=--~~-~
2a

We see from this answer that as 0 -+ 90°, P-+ 0; this is because if the rods are
vertical, there is no need for any force P to maintain equilibrium,

We now work the problem of the preceding example by the usual


method of free-body diagrams accompanied!by equilibrium equations of
various parts of the structure (see the illustration). From the overall
,,'
freebody diagram~ the roller reactions are each mg/2.
,
, . ,-,',

L
i~
iI ,~

t
I
t, ..

f'i"
t
r ~mg

The forces exerted on the two lower (two-force!) members shown in


the diagram at the left are obtained from
+t 'LFy == 0 = 2F1 sin 8 - mg
mg
F1 = ---
2 sin 8
Moments about B will now give the value of P for equilibrium. Using
the free-body diagram,

p
mg mg
= - a cos 8 + -,-. -
2 2 SIll e sin(n - 28)b - Pa sin 8

sin 28

mga
P = [ -, - cos 8
2
mgb
+ --.
2
-
SIll
' ,
(2 sin 8 cos 8)
8 . '.-.
'JI a sin 8

531
',.'>
532 Chapter 8 Special Topics

or

mg(a + 2b) cot


P=-------
e
2a
as we obtained earlier using virtual work. Note now that as -+ O,cot ()e
(and therefore PI get very large. This can be seen most easily from the
°
figure, where if () -> only a small fraction (sin ej of each Pi is available to 1
hold up half of mg. I
Comparing the two approaches, we see that virtual work is in a sense ~'
a "cleaner" approach because forces (such as Pi above) that do not do any t

~l~\:i~~~a~;~:~~~ the overall structure do not have to be considered at:

" -(

E x A M p L E 8.3~
Six bars, four of length 21 and two of length t, are pinned together to form a gate as
shown in the figure. The springs are attached'at the midpoints of the indicated
members, and the modulus of each is k. The springs can be compressed, and they
are unstretched when the gate is fully retracted. Find the angle 0 for equilibrium.

SOLUTION
Provided the actual constraints are not violated, the horizontal reaction of the
slot onto the roller does no work during a virtual displacement (j() from the
equilibrium position, nor do the pin reactions at A. Only the springs and P will
contribute to the virtual work (jW, which vanishes.
The length L of each spring in the equilibrium configuration is seen in the
diagram to be

I.
1:
T'.. ff.

2 . = I

(Continued)
sin ()
Therefore, the stretch in the spring is giv~n by

S = I sin 0 ~ 2 G) = I(sin 0 - I)

'----v--'
Un stretched
length

which is negative because the spring is compressed. The virtual change in S, or


"virtual stretch," is

bS = I cos 0 bO

and the virtual work of two of them (with equal Sand bSI is

b \¥.prings = - 2kS bS = -2k/(sin 0 - III cos 0 bO

= 2kl2 cos 011 - sin OJ bO

As seen below, the force P is located at x = 51 cos 0 so that

bx = - 51 sin 0 bO

and thus the virtual work of P is

bWbyp = Pbx = -5P1 sin 0 bO

m--- x.

Using bW = 0, we find

bW = 2kl2 cos O( I - sin 01 bO - 5P1 sinO bO = 0

or, since bO is arbitrary,

. 1 - sin 0 - (:1) 0
2.5 tan = 0 ( 1)

The solution to this equation may be obtained by trial and error using a cal-
culator or a computet. If the calculator is not programmable, we could proceed as
follows for the case P/ kl = I.
A rough sketch of the functions I - sin,O and 2:5 tan 0 is shown in the
,
.
diagram. It appears that they are equal [thus satisfying Equation lIll at 8 ~ 0.4 .
Using this as an initial guess and proceeding to compute the left-hand side of

(Continued)
Equation (I), call it f(OI (with the calculator computing in radians!), we obtain:
Functions of 0

() flO)

0.4 -0.446401
0.3 -0.06il861
0.29 -0.031984
0.28 0.004759
0.281 0.001090
0.282 -0.002580 o

Therefore 0 for equilibrium is about 0.281 rad or about 16.10•

-c E x A M p L E 8:.4~
In the figure find the force exerted by the pin at D on the member BD in the rigid
frame of Example 3.19.

7 ft
B

I
1.5 ft

I
il
1501b

_. A. i
1~3ft~ ~
SOLUTION
Referring to the free-body diagram of BD, we see that a virtual rotation with B
fixed would cause Dy to be the only unknown component to do virtual work.
Remember that the angle of rotation is to be infinitesimal so the displacement of
D is essentially vertical. Letting 00 be the virtual rotation, then Isee the figure)
ow= 0

Dy(300I- 150(1.500) = 0
00
__-~~+J- Dy = 75lb
(::~6'~V;I,::;::;,;:;;;J£F';;F~lt t Of course, this is the same result obtained from applying l:MB = O.
300 (Continued)

534
Now giving the bar a virtual horizontal translation, £, as shown below in the
diagram, then

bW = 0
C 2501b --
I~I
A.,".) _:J

which, of course, is nothing more than the equivalent of applying 1:Fx = O.


To complete the solution of the problem," we now apply the method of
7 ft
virtual work in a manner that does not proviq.e such an easily recognizable
correspondence with the equilibrium equations. We focus our attention on the
composite of the bars ABC and CDE for which we have the free-body diagram
-1 B
shown. Now we choose the virtual change in configuration to be given by an
infinitesimal rotation, bO, of ABC about A and a "horizontal translation, 7 bO, of

'f" A. E
CDE so that the bars remain connected at C, and 'E moves horizontally as shown
inthe illustration below. Thus, point B moves 2.8 bO to the right and points Cand
D each move 7 bO to the right. Applying the principle of virtual work to this
assembly and noting that there is no net work done by the forces of interaction at
E
C,

-I 1-
C
700

"\
I \
M-I \\
I \
B I \
-11-2.800 \\
\
\
A

-I f--7M
bW = 0

Bx(2.8 bOl + 250(7 bO) - Dx!7 bO) = 0


or
2.8Bx + 7(250) - 7Dx = 0
and since

4.2Dx = 7(2501
or
Dx = 417lb

as was found before in Example 3.19.


,
E x A M p L E I

Twelve light rods of length R are connected to six equally spaced points A, B, C,
i
1,
D, E, and F on a circular base as shown in Figure 1. They are joined by. ball and "
socket connections to the base and to each other. The six bar-to-bar connections lI
at the top are joined by six stiff springs of unstretched lengths OAR. The springs,
which can carry compression, form a hexagon in a horizontal plane. If the spring
moduli are each equal to k, find the anglels) e, between the vertical through A and
the plane BQF lsee Figure 2), for which the system is in equilibrium under six
downward loads P applied at Q, R, S, T, U, and V. Consider only symmetrical
displacements-that is, those in which the six top connections Q, R, S, T, U, and
Figure 1
V (Figures 1 and 3) each move inward and downward identical amounts.

i J'

Figure 2

SOLUTION
The sum of the virtual work done by the forces P and the springs will add to zero:
bW = 0

From. symmetrical equilibrium positions, the forces P will each move in-
ward toward the z axis lsee Figure 1) and downward, during a virtual increase in
A D
o of amount 150. The virtual work done by P is seen in Figure 4 to be PIR/2) be sin e,
and there are six of these. .

p
Vertical
~o(J
2

Figure 3

Figure 4

(Continued)

536
Next.we consider the virtual work done by-the six springs. During the vir-
tual rotation bO of triangle BQF(Figure 2) about the line BF, the length of each
spring will change slightly. In the equilibrium position of the structure at 0, the
length of each spring (see Figure 5 and consider spring QV) is seen to equal R -
(RI2I - (RI2) sin O. Thus the stretch in each spring is O.IR - (RI2I sin O. This
means the' 'virtual stretch" occurring during the virtual rotation is - (R121 cos e be,
and the virtual work of each spring is
A

The total virtual work may now be equated to zero:

bW = ~ = 6 [ P ~ be sin e - kS bSJ
QV is the same
length as QO!
Figure 5 o~ [P: sinO- k(O.IR - ~sine)( -~cose)Jbe

or

4P
\. kR sin 0 + (OA - 2 sin 0) cos 0 = 0
)

For P = kRI4, the only roots to this equation for 0 ~ 0 ~ n/2 are = 27.2° and e
46.3°. It is interesting to note that some equilibrium positions are unstable, such
as a marble balanced on an inverted spherical bowl. It can be shown using more
advanced mechanics that the 27.2° equilibrium position that we have found
above is stable, while the 46.3° position is unst~ble.

~. PRO B LE M S / Section 8.1

Use the principle of virtual work to solve: 8.11 Find the relationship between P and F for equilibrium
of the linkage shown in Figure P8.11.
8.1 Problem 3.86
-,. ~.<
8.12 Find the force P that must be exerted for equilibrium
8.2 Problem 3. 104 ofthe double lever system shown in Figure P8.12.
8.3 Problem 3.105

8.4 Problem 3. 107

8.5 Problem 3.193 p

8.6 Problem 3.232

8.7 Problem 3.241

8.8 Problem 3.261


p
8.9 Problem 5.97 ~
SOOlb
8.10 Problem 5.106 Figure P8.11 Figure P8.-12.j -

537
538 Chapter 8 Special Topics

Figure P8.17

Differential
chain hoist

FigureP8.13

(Members such as ABC are continuous bars.)

Figure P8.14 Figure P8.18 Figure P8.19

8.13 In Figure P8 .13 find the force F that will hold the 8.17 Find P as a function of a, b, C, W, and 8 for ;
I
weight W in equilibrium. equilibrium of the system shown in Figure P8.17.

'8.14 In Figure P8.14 the light, homogeneous, I-m bars &1 8.18 Find the angles e and </J for equilibrium of the system
and &2 are pinned together at D and are also connected by of two light bars and two heavy weights shown in Fig-
the spring AB of modulus 200 N/m; the unstretched' ure P8.18.
length of the spring is 0.15 m. What is force P if the system
is in equilibrium when triangle DEF is equilateral? * 8.19 In Figure P8.19 find the force exerted on the cargo lift
by the hydraulic cylinder C if it is pinned to ,the lift at
,8.15 Repeat Problem 8.14 if &1 and &2 each have a mass of (a)point A; (bl point B. (c) Use the result of (a) to compute
2 kg with respective mass centers at C1 and C2. the reaction of pin C onto the member CDE at C. Is this
reaction along CD?
8.16 For the data of Problem 8.14, find the angle 8 for
equilibrium if the force P has a magnitude of 20 newtons.

* Asterisks identify the more difficult problems. 11


1(
~i
_ .._-0-<
..~~
.~.
8.1 The Principle of Virtual Work 539

20 in.
R

Figure P8.20

300lb

Figure P8.21

8.20 If the system in Figure P8.20 is in equilibrium, find b. Find the force exerted on the roller at C by the slot,
the couple M being applied to crank ..A as a function of using equilibrium eq~ations. Then take the four bars
the angle (J and the force P on the piston. The crank, piston, apart, and, on free-b~dy diagrams of each, show all
and connecting rod Rare light, and friction is negligible. forces acting on the separate bars.
J c. Show (with both virtual work and with equilibrium
! . 8.21 The linkage is made up of 10 light bars of length 2 ft
equationsl that with ,the couple omitted the linkage
(such as AB)and two bars of length I ft (on the extreme
F- , cannot be in equilibrium.
right!, pinned together as indicated in Figure P8.21. The
300-lb force is to be applied as shown, and it is desired to 8.24 In the preceding problem, delete the couple M and
have the_systeq1~ in equilibrium at (J = 60 Find the 0
• add a vertically attached stiff spring joining the midpoints
upward force at C on bar CD that will accomplish this. of segments CF and AF with natural length 1 ft. Find the
spring modulus if equilibrium exists at (J == 600

8.22 In Problem 8.21, suppose that instead of the force at


C, the equilibrium is to be maintained by applying a 8.25 The smooth collarsC and 2J in Figure P8.25 ensure
counterclockwise couple onto bar AB.Find the magnitude that the spring remains on the same horizontal leveL The
of the couple. spring is unstretched at 0 = O. Show that there is an
equilibrium position of bar B (besides the obvious one at
8.23 In a problem similar to 8.22, there are just four bars,
o = 0) given by
with all other data remaining the same. See Figure P8.23.
. [(khPI2)113]
a. Determine the couple M acting on AB for equilibrium
o = cos _ I
using the principle of virtual work ....•

r
I

E
300lb Figure P8.25

D
Figure P8.23
540 Chapter 8 Special Topics

8.26 This.problem is similar to Problem 8.25 except that


now the spring changes its level while remaining hori-
zontal. Find the "not-obvious" equilibrium angle e. (See
Figure P8.26, where the spring is unstretched at e = 0.1

8.27 Use virtual work to solve Problem 3.218.

8.28 Verify the result of Example 8.5 by using the equi- Figure P8.26
. ;
librium equations.

In Problems 8.29-8.32 find the relationship between P and f


W using the principle of virtual work.

8.29 8.30

8.32

Figure P8.30

8.31
p
I
... \.~.
I
I
W
p

a b

Figure P8.29

Figure P8.31

Figure P8.32

~8.2 Hydrostatic Pressure on Submerged Bodies


In Example 2.39 we recalled that in a fluid at rest:
a. The pressure is equal in all directions ,at a point (Pascal's law).
b. The pressure is constant through the fluid in each horizontal plane.
c. The pressure causes a force that is normal to every differential area of
surface on which it acts. .
d. The pressure is equal to yh, where y is the specific weight of the
fluid * (taken constant here) and h is the depth below the free surface.

*y is its weight per unit volume, related to the mass density p by y = pg.
8.2 Hydrostatic Pressure on Submerged Bodies 541

Also, in Section .6.1 we found that when pressure acts on a plane


surface, the resultant of the pressure forces has:
a. A magnitude Fr equal to the volume beneath the loading surface.
b. A line of action normal to the plane 'knd directed toward it, and
passing through the centroid of this volume.
In this section we shall first broaden our treatment to allow a
hydrostatic pressure to act on non-rectangular plane areas. It will tum out
that the formulas we obtain for Fr and its line of action will be intimately
tied to the very same properties of areas we have been studying in detail
in Chapters 6 and 7: the amount of area, its centroid, and its inertia
properties.'
We shall also look at two results that allow us to obtain the resultant
of fluid pressures on curved surfaces, often without the need for
integration. Finally, we shall complete our look at fluid statics by briefly
discussing Archimedes' principle of buoyancy.

Hydrostatic Pressure on Plane Surfaces


We begin by developing some results for hydrostatic pressure on arbitrary
plane areas. At the depth x cos () (see Figure 8.101, the pressure* on all
elements of the line lEis given by
p = yx cos () (8.181

"
'~
k/ Z

h = depth
= xcos8

Figure 8.10

* Throughout this section, we are using gauge pressure-that is, the pressure above
atmospheric, If the pressure behind one of the submerged surfaces we shall be studying is
atmospheric, then the actual front-to-back pressure difference is indeed the gauge pressure
we are using. If this is not the case, however, then bqth should be expressed as absolute
pressures or both as gauge pressures so that correct differences will be used in detennining
resultant fluid forces on' the surfaces.
542 Chapter 8 Special Topics

where () is the inclination of the plane area A with the vertical. Noting
that all the differential forces on area A due to the pressure (dF == p dA)
are parallel, we know that there will be a "force.alone resultant" in this
direction, and we may write

dF = pdAk

Integrating, we then obtain the resultant:

Fr = kf :xc~s (},dA

P
(8.191

Fr = ky cos () x dA f
But from our knowledge of centroids,

xA = f xdA (8.20)

so that

Fr = k y(x cos ())A == PeAk

Depth of
centroid
of area

Pressure at
centroid = Pc .

Thus the resultant force, normal to, the plane of the area, has a
magnitude equal to the pressure at the centroid times the area. We now
seek the location of this force-aloneresulta'nt.whichis equipollentto the
fluid pressure forces. It will not be at the centroid of the plane area unless
() = 90°.
We are looking for the x and y coordinates of a point P where Fr
(above) may be placed and have the same moment about any point (we use
the origin 0) as does the original system' of fluid forces. This point is
called the center of pressure.
Thus, we wish to find the vector rap in the equation

r opx Fr = f r x dF (8.21)

We represent the sought vector rap as


~ ~
rop = xpi +Ypj, (8.22)
8.2 Hydrostatic Pressure on Submerged Bodies 543

and we write

(xpi + YPJ) x (PeAk) = f (xi + YJ) x(p dAk) (8.23)

-xpPeAJ + YpPeAi = -J f xp dA + if YP dA (8.24)

Equating the J coefficients of Equation (8.24) will give us the x


coordinate of the center of pressure:

xp =
f xp dA f x(yx cos e) dA

PeA y cos e xA
,r.ces-tr f x 2 dA
x ~--~---
.~
p - ;r.ces-tr xA
and recognizingthe moment of inertia (see Section 7.1),

(8.25)

Now, using the parallel-axis (transfer) theorem (see Section 7.31,


2
X
p
= lyC + Ax = X + lyC (8.261
xA xA

We see that, since lyC is always positive, the center of pressure is


always lower than the centroid of the area unless = 90°, in which case e
the pressure is the same at all points of A, and then P and C are at the same
depth.

We next equate the i coefficients of Equation (8.2.4) and find the


lateral position (Yp) of the center of pressure:

YP =--
f YP dA f yyx cos e dA
PeA (y cos e x)A
f xydA
~lxyO
(8.27)
YP = xA xA

J
where lxyO = - xy dA is the product of inertia of the area with respect to
the axes x and at Yat point 0. Using the parkllel-axis(or transfer) theorem
544 Chapter 8 Special Topics

(see Section 7.6),


- (lxyC - Axy) _ IxyC
(8.28)
YP = XA = Y - xA

If either of the axes through C, parallel to x or Y, is an axis of


symmetry of the plane area, then IxyC = 0 and then YP = Y. This means
that the lateral distance from the x axis to' the centroid in this case is
the same as the distance from the x axis to' the center of pressure* (see
Figure 8.11).
If, however, IxyC -=I: 0, then the center of pressure will be displaced
laterally from Cj YP will not equal y. We shall now consider three
examples. In
the first, the area is rectangularj in the others, the width
varies with depth.

Axis of symmetry of A"

Xl (Parallel to x)

(a) (b)

Figure 8.11 la) In this case, YI is an axis


of symmetry of area A, so that IxyC = 0
and thus Yp = y. (b) This time, x I is an
axis of symmetry of A, so again IxyC = 0
andyp = y.

E x A M p L E

Show that the results of Example 2.39 follow from Equations (8.20) and (8.26) of
this section.
(Continued)

* For the case when the axis through C parallel to x is an axis of symmetry, the result
YP = Y becomes obvious when we recall that the line of action of F, passes through the
centroid of the volume beneath the loading (pressure) surface; the other case's (same) result
does not, however, follow from this knowledge'about:1F:s'lineof action:;;,",,,,••,,,, ..
SOLUTION
We have seen that the resultant force on any plane area A equals the centro idaI
pressure times Ai therefore,

Area of
Pressure at C gate

IF.I = y
, (Xl + X2)"
2' (cos <f>llx2 - xllw =
,
62.4(x~ -
1IJ2
xil-2-11O)

= 624[(60J212 - (2oJ212]/(2J21 = 1.41 X 106lb

which is the same answer as we obtained by integration in Example 2.39. For the
distance d to the center of pressure p, we use Equation (8.261:

I
x
p
=x+~ xA

o Using the figure at the l.eft, the terms become


y
lye

~
w(X2 - xl13
l X +X2 12
Xp =---+
2 IXI
___
+ xl'2 [lx - xllwJ
2
2
w
X A
1 = (base)(height)3
Yc 12 After simplifying, this results in
2 2
d =
2
_ X2
------
+ XlX2 +. Xl
= 61.3 ft
3 Xl + X2

which is again the same answer we obtained in Example 2:'39:".•.~..,_.".•...••


-.- ....

545
E x A M p L E 8.7~
Find the center of pressure of the triangular fluid gate, shaped and situated as
shown in the figure.

Yc

SOLUTION
Using Example 7.3, we can obtain
S4
I =-
Yc 24

We then obtain, for our triangle,


64 .

I
Yc
= -24 = S4 ft4

Therefore,
,.
I
x =x+.2'£
P xA ~i,
, '
.J

.r;:; S4 j •

= (4 + 3....;2) + ----~
(4 + 3J2)t(6)6

= 8.61 ft

For the y coordinate of p,

o by symmetry
- IIxyc
YP = Y - XA

Yf> = Y '- 0 = t(3J2)


= J2 = "1":4°1 ff ~

546
,I

E x A M p L E

If the same gate as in Example 8.7 is reoriented as shown in the figure, fihd the
new position of the center of pressure.

SO L UTI ON
. BH3 6(63)
This time, Iy (see the figure) = -- = -- = 1<08£t4
12 12
so that
.' '~.

lyC = 108 - Ad2


108 - !(6)6(22) =36 £t4 *

and so

I
x = x + ..1'£.
p xA
36
= (4 + 2) +(4 + 2118

== 6 + HI = 6.33 £t
The product of inertia IxyC is needed for Yp. 'From Example 7.21, it is

(base x heightl2
I =-.----
xyc 72

(6 X 6)2
72
(Continued)

BH3 6(63)
*Or we could al..tematively use lyC ==. --"36'" = -'36 36 ft4.
-.-' - =......•
~_ .•,

547
~
~

Therefore,
i
~~.

~.
.~

1..
.

:-i!.
18 .~
== 2 ~ --- 1.83 ft &
(6)(18) ..~.
-"!;,"

Hydrostatic Pressure on Curved Surfaces


If the submerged surface S is not necessarily flat, then formulas such as
(8.26) and (8.28) do not apply. Instead we may use the following two
results: Let the resultant force due to hydrhstatic fluid pressure on one
side of a curved surface* be called Fr' Then:
Result (I)The component of Fr ina horizontal direction x is the force
caused by the same pressure acting on the vertical projection of the area
..onto a plane normal to x. The proof is as fopows, with 0 being the angle
between the normal to the differential areadA and the x direction (see
Figure 8.12):

Frr= f (p dA) cos () =


"-.---'
f p(dA cose)
' .. '
Figure 8.12
dF Vertical projection
of dA, normal to x

Differential force '-----~,.


in x direction Differential force on
projected area normal to x

Result (Ill The vertical component of Fr is the weight of the column


of fluid above S.

In this second result, we see that there is no confusion if the fluid


above the surface all extends vertically up to the free surface, as shown on
the left in Figure 8.13(a). A surface S ma4e up of the curve AB and a
uniform wi~th w normal to the page is being considered, and the vertical

*With no vertical outward normals except possiblyalong.partofjtsedge;",,, ...


,..~,.,..,," M.".. ,.

548 t
8.2 Hydrostatic Pressure on Submerged80dies 549

A A
(<1) (b) (c)
Figure 8.13

component of the fluid force resultant on S is obviously the weight of


the shaded fluid. In Figure 8. I3(b), howevq, the fluid above the same
surface S does not extend up to the free surface. And in Figure 8.13(c),
it does so over some of the area elements, on others it does so after inter-
ruption, and on still others it doesn't extend upward to any free surface
at all. The point is that none of this matters. The vertical force on S
acts as though the fluid existed ini.a column up to the free surface (ex-
tended, if necessaryl in all cases because it is the pressure on the surface
that counts and the pressure in the fluid is the same across any hori-
zontallevel. .
We now present two related examples. In the second, we shall
consider the pressure on a curved surface; in:both, we shall use results I
and II.

1l!1
81i9~
-] E x A M p L E
If the dam in the figure is 200 ft wide, determine and locate the resultaht force
exerted on it by the water pressure.
60 ft

SOLUTION

j We note first that in this case, all differential forces exerted on the dam by the
water are parallel; therefore, there is a "force-alone" resultant that is normal to
the slanted surface.
The horizontal component of the resultant force equals the pressure at the
centroid of the projected (shadedl vertical area times this area:

Pressure at Projected
the centroid area
,..-~-...,.,---"--..
. Frx = (yhcl(Apl

= (62.4 :t3}25 ftl(lO,OOOft 2


) = 15.6 x 106lb
~ ,"'~ ",..~:.",
..•~ .......•...•..
~_",.ot .•, ....,..•.•.
_.""'''''-~~
..•_.';r''_''',.<1',...",,.. ..~._,., •
Projected area ='= 10,000 ft2 (Continued)
The vertical componencof the resultant force equals the weight of water (see the
figure) above the wetted surface:

. . lb
b-1 6
FrY =ylVoll = 62.4ft3 X 10ft X 50ft X 200ft) = 3.12 X 10 lb

Thus the resultant force has the magnitude

Fr = .,)15.62 + 3.122 X 106 = 15.9(106Ilb

This magnitude of Fr may be checked. As we have seen, it is also the volume


(of the triangular prism in the figure) under the loading (or pressure) surface:

Fr = tbhw

= t(5111y501(200)lb

= 25.5(62.4)50(200) lb

(as abovel

As for the direction of F" it is, as previously mentioned, normal to the


surface because it is comprised of infinitely many parallel differential forces, each
one normal to the surface with the sense ~
50 --

We note that the theorems used to obtain FrX and FrY also yield thecorrrect
direction for the resultant:

() = tan-I (15.6) = tan-I 5 (as above)


3.12

where

50
</> = tan-I - = tan-I 5 (also)
10

.'. Fr is seen again to be normal to the slanted surface of the dam.

F,y[acts at centroid of (dotted)


volume of water above the wetted surface]

~
FrY = 3.12 X I061b

FrX = 15.6 X I061b

F,x [acts at centroid of (lined)


pressure curve on vertical projection] I'
.j

.: t'
(Continued) ~-

)
:
./
? ::
...•...

550
The place on the dam where F, acts (known as the center of pressure) is at the
centroid of the distributed loading, which is in this :case a triangular prism. Thus
the load acts normal to the dam a distance i(51) =34 ft along the slant from the
water surface (see the illustration), and, of course, halfway across the dam (100
feet from each endl.
We may check this result in two ways: First, we use the components F,x and
F,y obtained earlier, and let M,o be the resultant moment about 0 of the forces
due to hydrostatic fluid pressure. Then

M,o = F,x[i(50) ftl + F,y[i(lO) ftl

= 15.6(106) IOOJ + 3.12(10 [20J


[3 "'3. lMt 6
)

,
,, = 520(106) +20.8(106) = 54111061lb-ft
i\

If this is to be equipollent to F, at distance d,then

15.9(106)d = 541(1061
1
L

I.
d = 34.0 ft (as before)

Alternatively, we may determine the distance d by using Equation (8.26):

I
(. <
d or x
P
I
=.2'£
xA
+x
f'(
, ~-

[2001(~13)J
d = ----- + 25.5 = 8.5 + 25.5
25.5[200 x 511

d = 34 ft

E X A M p L E
Repeat Example 8.9 if the dam is parabolic instead of linear.ISee the figure, noting
" the different coordinates.)

(Continued)

55'1
SO L UTI 0 N
The horizontal component of Fr is the same because the projected area is
unchanged:

F,x = - 15.6 x 106lb

For F.y we need the specific weight times the"volume of the water above the
I

wetted surface of the dam:


dA

FrY = y:~ [50(10) - I5~o'(y212~01 dY]ft2(200 ftl

= (62.4 :~3)( 500 ~ ~~~) ft2(200 ft)

FrY = 4.16 X 106lb

which is a bigger force than we found for the triangular dam because there is more
water above this one.
The above components of course add vectorially to form the resultant
force Fr:
Fr = Frx1 + FrY; = -15.6(106)1 + 4.16(1061;lb

The components act along the lines shown in the illustrati_on below:

FrY c-:!.(IO) = 6ft


. (~e Example 6.13)

1f (50) ~ 18.8 ft

Centroid of volume of
water above wetted
surface of dam

Centroid of loading curve


on projected area

We know that there is 11 force-alone resultant of all the infinitely many


differential forces acting on the dam due to water pressure because these forces
form a coplanar distributed system once the pres~ures are multiplied by
the 200-ft
width. The location of the line along which ihis force-alone resultant acts is
'shown in the next illustration.

(Continued)

552
An exercise problem will be to locate the spot S where F,intersects the
surface of the dam, and to show that F, is not in this case normal to the surface, as
would always be the case for a plane area as we have seen in the previous example.

(
x

1
33.3 ft

j ~F,x = 15.6(106) Ib
I '
FrY = 4.16(106) lb "----'--F,

To check the location ofthe resultant, we sh~U integrate to get M,o:


x

M,o == f r xdF
Wetted
surface

f
M,o

'
= [(xl
.
+ yj)x
r
'~
,yy w ds( - cosol
'----' --------
+ sin oTn*

zj
Pressure dA, Unit vector in
directiOn of

'" 'dy '-- ,_d-iff-e-re-n-ti-31-f-or-ce-

Differential force dF
dx

M,o= ywk flXY sin 0 + y2 cos OJds


But as can be seen in the iilustration,

dx dy
sinO =-,d and cos 0 =-
s ds

so that

(Continued)

*Note that (J =tan-t(dX) = tan-t(~),Which is not needed in this example.


"~dye 125', "",.~ .,.' , , ",
MrO = 12500k{[ ,/250 ~~~JI:O+ [~3JI:O}
12500k[2000 + 41700]
= 546(1061k lb-ft

The resultant Fr has the same moment about 0 as do the aboye distributed
forces:

15.6 x 106lb 4.16 x 106lb

Mro = ?!xli x SOl + ~l! x 101

= 545(1061Ib-ft,

off by about t of 1% due to roundoff.

Sometimes we need to compute fluid pressure forces beneath a


surface. The force due to such pressure is the negative of the force
produced by an imaginary column of fluid above the same surface and
extending up to the free surface level. Thus we may use our previous two
results (with all forces reversed) for such problems, if we wish. We
illustrate this idea with an example worked two ways: first with
straightforward integration, and then using the idea of "imaginary fluid
above the surface." ~

E x A M p L E

Find the resultant of the fluid pressure forces acting on the sluice gate AB in the
left diagram of Figure I, expressed as a force at the pin A and a couple. The gate is.
a thick, quarter-cylindrical surface with radius R and length L perpendicular to the
paper.

Figure 1

(Continued)

554
SOLUTION
Since the pressure varies linearly with distance from the free surface, as suggested
in the right diagram of Figure I, and since this distance is H + R sin e, the
resultant force Fr has the components:
1"/2.
Frx = J6=0 y(H + R sin e)LRde cos e
\" ,~
Pressure dA
(I)
Differential force dF

dF,

and

Fry = f~20 y(H


~~
+ R sin e)LR de sin e

Pressure dA
(2)

Differential force dF

dF,

Carrying out the integrations leads to

(3)

R A

rY;:m,m "m of ~

J.. Figure 2

For the moment about the pin A, which together withFr completes the
resultant at A of the fluid forces on the sluice gat~, we integrate one more time:

MrA = 1~20 y(H


••
+ R sin
w
e)LR de R sin e
J~

dF las above) Moment


arm

= YR L( H + iR)
2 (41

(Continued)

555'
Thus the resultant at A is the force F, together with a clockwise couple of
strength M,A'
Consider now the forces resulting from water imagined to be above the
sluice gate (see Figure 31.

1 H

-t
! A

~
0 pgLR B
Figure 3

For the horizontal component of the resultant force, we add the forces on the
vertical projection CB of the gate:

yLHR +

Volume of rectangular Volume of triangular


pan of load surface pan of load surface
(shaded, Figure 31 Idotted, Figure 31(acts
(acts at z = H.+ R/2, at Z = H + 213R,
the centroid of the centroid of
this volume) this volume)

It is seen that this force, to the right, agrees with the actual F,x of Equation (3).
x For the y component, we see in Figure 4 that the weight of the water shown
shaded is y(HRLI, acting at the center of the block, and the weight of the water
2
nR ) 4R L
shown dotted is y ( -.- L , acting at X = Z = R - - and Y = -.
4 3n 2
It is also seen that the (downward) total y force, the sum of the weights of the
two volumes of water in Figure 4, is the same a~ F,y in Equation (3).
Finally, the above four contributors to - F, may also be used, each in reverse,
to check M,A:
"Figure 4
Rectangle Quarter-circle Rectangle Triangle

f ,,.

2 "'" , "

'+) M'A = (yHRL) ~ + (y n: L)( R _ ::) + (YLHR{~) + (YL:2)(~ R)


Moments of y forces Moments of x forces

This is the same result that we obtained (Equation 4) much more directly by
integration.

556 ,
--~
8.2 Hydrostatic Pressure on Submerged Bodies 557

Buoyant Force
~' -
Archimedes' principle (220 B.C.!) is that the buoyant force on a sub-
merged body equals the weight of the fluid displaced by the submerged
portion of thebody, The buoyant force acts through the center of mass
of the displaced. fluid, a point known as the center of buoyancy.

Figure 8.14

~
For the proof of the principle, let the submerged portion of the body
be replaced by an identical volume VF, in amount and shape, of the
fluid itself as suggested by Figure 8.14. Such.'a volume is obviously in
equilibrium, so that the buoyant force B on the fluid is the weight of the
fluid volume, acting upward through the mass center CF of this fluid.
volume. In Figure 8.15, PF is the mass density of the fluid and B is the
single-force resultant of all the differential forces caused by fluid pressure
F.igure 8.15 on the fluid volume.

Now, returning tg the actual body, the fluid pressure on its sub-
merged portion is everywhere the same as we had on the substituted
volume of fluid because this pressure, a fun,ction of depth and fluid
density, is unaffected by what really comprises the volume VF• Hence the
resultant B of the forces caused by this pressure~s the same. The free-body
diagram of the actual floating body B is seen in Figure 8. 16. The body is
thus buoyed upward by a force equal to the w~ight of displaced fluid. In
Figure 8.16(a), PT and Ps are the average mass 'densities of the parts of B
above and below the free surface of the fluid. N~te that the mass c~nter of
the part of Boccupying VFneed not be at CF, or even on the same vertical.
line as CF. But the mass center C of all of B is on this line as shown in
Figure 8.16(bJ, because otherwise the moment equilibrium equation
could not be satisfied.
We now consider briefly the importance of the position of the
buoyant force on the rollingmotionsof'a'ship in' the sea!"E:onsider the; ..

J
558 Chapter 8 Special Topics

J
Center of mass of the 'tip
of the iceberg' (part of IJ ;]
above the waterline) .'.
or

.~.

(a) (b)
Figure 8.16 .

The intersection of
the displaced buoyant
force with line ACs
is called the metacenter.
Cehter of mass' -
of the ship
---- ---- .

New center
Center of mass of buoyancy
of the displaced fluid,
or center of buoyancy'
Figure 8.17 Figure 8.18

sketch of a floating ship's profile in Figure 8.17 with the mass center CF
representing the mass center of the displaced fluid volume VF.
Now let the ship roll about the z axis normal to the paper through,
say, point A in a motion caused by wind and water forces (see Figure 8.18).
)'
The buoyant force will, in general, change. There is now more fluid
displaced on the left and less on the right. Depending upon the profile of
the ship's hull, the weight of this displaced fluid mayor may not even
equal the weight of the ship. In any case, there will be a definite change in
the line of action of B drawn on the ship's profile, and a possible change in
its intersection with the line ACs (which line, relative to the ship, hasn't
changed). This intersection point.iscalled the.:.~metac.eme.r.~~figur~_8.~~.
I
j, 8.2 Hydrostatic Pressure on Submerged Bodies 559

shows us that if the "metacenter" lies above the mass center, then the
moment formed by Band mg is "restoring" -that is, tends to turn the
ship back toward the vertical. In this case, the ship would be stable.

,! -,

In ship design, the hull profile and mass distributions are carefully
computed so as to keep the metacenter above the mass center and also to
keep it close to the same spot for fairly large roll angles of the ship, up to
1.
~ 20° and more in some cases.

E x A M p L E
Compute the largest (uniforml thickness t so that a tub hollowed out of a steel
block will float (see the illustration). Let I = 7 ft, w = 3 ft, h = 2 ft, and Yw =
62.4lb/ft3, Ys = 490Ib/ft3.

w
{,

:,.....

SOLUTION
Using Archimedes' principle, we shall equate the weight of the tub (in terms of tl
with the buoyant force (weight of water displaced) when the waterline is up to the
top of the tub:

Weight of tub = Ys(Volume of steel in tub).


WT= 490[wlh - (h - t)lw - 2t)ll - 2tll

(Continued)
Substituting and simplifying,

WT = 490(4t3 - 28t2 + 61t)


The buoyant force is

B = Yw (Volume of displaced water)

= 62.4 (wthl = 62.4(42)


= 2620lb
For equilibrium,
[
+~
which gives the following equation in the thickness t:

t(tl = t3 - 7t2 + 15.3t - 1.34 ~ 0 (II

This equation has but one real root, which is found below by trial and error to be'
t = 0.0913 ft or about Ll inch:

f(tl

o -1.34
1 7.96
0.1 0.121
0.09 -0.0190
0.091 -0.0049
0.0912 -0.0021
0.0913 -0.0007
0.0914 +0.0007

PRO 8 L EMS / Section 8.2

8.33 Show that the pressure in a fluid at rest lal is constant


through the fluid in each. horizontal plane i (b)obeys the
equation dp = y dz, where z is measured downward. Thus
if z is zero at the free surface of a constant density fluid,
thenp = yz.

8.34 Explain why the fluid force is normal to every


differential area on which it acts.

8.35 Prove Pascal's law, that the pressure in a fluid at rest Px dy dz


is constant in all directions. (SeeFigure P8.35.) Hint: Sum
'forces on the infinitesimal element of fluid in the x and z
directions, obtaining Ps = Px and Ps = pz' Then note that
o and the orientation of the slanted surface around the Figure P8.35
vertical are arbitrary.

560
8.2 Hydrostatic Pressure on Submerged Bodies 561

8.36-8.40Find the force-alone resultant of the water


pressure forces on the shaded side' of the submerged
surfaces in Figures P8.36-P8.40.

".;-:,

r .!

8.41 For this third orientation of the gate of Examples 8.7 8.42 Show that the x coordinate of the center of pressure
and 8.8, find the ~enter of pressure. See Figure P8.41. of fluid force acting on the equilateral triangular area .A in
Figure P8.42 is unchanged by any rotation of .A about an
axis through the centroid no:rmal to the area. What is this
force if the fluid is water with specific weight 62.41b/ft 3
. and if XA = 3Q.ft and s = 3 ft?

FlgureP8.41 Figure P8.42


562 Chapter 8 Special Topics

8.43 The gate jn Figure P8.43 'holds back a reservoir of ,.•.. 8.46 Work Problem 8.44 if the door's shape is anequi-
water. The pressure at any depth z is yz, where y = specific lateral triangle as shown in Figure P8,46.
weight of the water. The gate is 5 ft wide perpendicular to
the paper. Thus the pressure load may be considered as a 8.47 Find the force exerted on the rectangular gate at A by
distributed line load of intensity Syz lb/ft acting on the the stop. (SeeFigurePS.47;J
"beam" shown. Draw the shear and moment diagrams as
suggested by the axes to the left of the figure.

-~-
-...h
M V

9 ft

x x Figure P8.47
Figure ,P8.43
8.48 The circular gate in Figure P8.48 has a diameter of
6 ft. It swings aboUt a horizontal pin located 4 in. below
8.44 Find the resultant force caused by fluid pressure on
its center. At what depth' h of water will the gate be in
the shaded door in Figure P8.44 if the door is rectangular
equilibrium?
with a width of wand a height of h.
8.49 Find the force due to fluid pressure acting on the gate
in Example 8.7.

8.50 Find the force due to fluid pressure acting on the gate
in Example 8.8.

f
h

Figure P8.44
r
6 ft

8.45 Work Problem8A4 if the door is circular with 1


diameter h.

1
h
4in.
--l
t
6 ft

2h/j3
_! 1
1
Figure P8.46 Figure P8.48
8.2 HydrQstatic Pressure .on Submerged BQdies 563
<--f.

i~ .. Width of
x dam.= 200ft

Figure P8.53

1-28£t--'1 .Figure P8.56


Figure P8.51

8.51 In Example 8.10 verify the resultant fQrce by in- beneath the dam due tQ tWQ~Qurces: (a)the reactiQn Qfthe
tegratiQn. See Figure P8.51. Hint: Use: earth ("intergranular, " i.e., earth particles Qnt.ocQncrete),
and (bl the' 'uplift" fQrce due tQwater pressure frQm fluid
dF= (pressurelldAIF-cQs 01 + sin 0,) that has seeped underneath the dam. The first .ofthese will
with pressure = yy, and dA = (width) ds=200 ds. Then have a hQrizQntal and a ~ertical' CQmpQnent arid act
use: sQmewhere between Band Di the secQnd is the'vertical
fQrce-alQne resultant of the upward fluid pressure fQrces
ds = .Jdx2 + dy2 = dy 1 + (dX)2
- (assumed tQ act .over approximately the entire bQttQm
dy surface .of the daml. Assume the uplift pressUre tQ vary
linearly from the full SOy at B to zero at D, and deter-
.or mine the magnitude and directiQnof the earth's (fQrce-

ds = J..
1 +. .(~.
125
)2. dy
alQne resultant) reactiQn, and the pQint where it intersects
line BD.

Further, 8.54 In Problem 8.53 find the fact .or .of safety against
.overturning .ofthe dam abQut D, defined as the ratiQ .ofthe
. . dy dx cQunterclQckwise mQment ab'QutD due tQthe weight Qfthe
CQS 0=- and sin 0 =- dam, tQ the clQckwisemQment caused by the fQrces frQm
ds ds
the water pressure .on AB and BD. The numeratQr resists
and substitutiQn gives .overturning and the denQminator promQtes"it:--
dF = 200yy[-djr1+-(ydy/1251,)"
8.55 Let H = 2R and find the magnitude, directiQn, and
line.of actiQn .ofthe single-forbe resultant .ofthe fluid forces
~j dx .on thegate .ofExample 8.11.
( .

!
f
I
which can be integrated tQ give Fr'
8.56 Find the fQrce atB in Figure P8.56 required tQ hQld
the gate AB .ofExample8.1 Lin equilibrium.
r * 8.52 In Example 8.10, find the cQordinatesQf the pQint S
I, where Fr intersects the parabQlic surface .ofthe dam. ShQW
I. 8.57 In Example 8.11 cQmpute the line .of actiQn .of the
that Fr is nQt nQrmal tQ the surface at S.
horizQntal CQmpQnent Frx Qfjthe fluid fQrce .on the sluice
.
~ 8.53 The dam in Figure P8.53, similar to that .ofExample gate.in tWQways, and check .one against the .other:
8.9, is hQlding back water, this time against a vertical 8. Using the rectangular and triangular distributicns in
(in,stead .of slanted) plane. The dam is made .of CQncrete the example
with average specific weightl 55 lb/ft 3. There are forces b.;l!sing EquatiQn(8.26). I
564 Chapter 8 Special Topics

8.58 The steel'gate has the cross section shown in Fig- ----2R--~
ure PB.5B. It is 10 ft long, and has a specific weight of
490 lb/ft 3. Determine the depth H of water that will cause
Figure P8.60 Segment
the gate to open (i.e., pivot clockwise, letting water flow
under). of a circle

8.59 In Figure PB.59 find the water level H at which the


hinged gate will open and let water spill out along the line
PQ. Neglect the weight of the gate.

8.60 A bowl with the dimensions shown in Figure PB.60 is


filled with a liquid of density p.By integration, find the
resultant force exerted by the liquid on the bowl.

8.61 In Figure PB.6l:


a. Find the single-force resultant acting on the gate q
caused by hydrostatic pressure, for the case (J == O.
The width of the gate is 5 m and the mass density of
the water ..= 19/cm3•
b. Find the reactions at the pin A and floor B. Figure P8.61

8.62 Repeat Problem 8.61 when (J = 20°.

8.63 In Figure.PB.63 find the magnitude, direction, and


line of action of the force-alone resultant caused by
hydrostatic pressure acting on a l-m width of the approx-
imately,parabolic surface.

8.64 Consider the law of buoyancy as it relates to ships


floating in the sea. For the simplified "ship" in Figure
PB.64, assume that the center of mass is at C on the
waterline in calm sea. Assume that the ship then rolls as
indicated. Locate the centroid of the displaced fluid (which
used to be at F) and show that the same buoyant force has
now shifted to the right sufficiently to cause the ship to Figure P8.63

begin to roll back counterclockwise.

T
w . ,.
..... La
f .."
c

,

w
t
4w

Figure P8.58

Figure P8.59 FigureP8.64


8.2 Hydrostatic Pressure on Submerged Bodies 565

8.65 In Problem 8.64 show that if the simplified ship is


only 2w wide, then it will continue to roll clockwise
toward capsizing if it has rolled to the point where the same
3w/2 is above the waterline on the left side.IThis angle of
roll is 26.6°, which is very large.) See Figure P8.6S.

8.66 The hollowed sphere in Figure P8.66 is sealed at its,


base and is being used as a tank. How full is it jH - zl at the
instant it lifts off the ground? IAssume the seal exerts no 2
downward force on the tank.) Figure P8.65

8.67 Huck Finn has gone to sleep with one end of his
fishing pole resting between his toes. If the density of the
pole is half that of water, find the fraction of the pole that is
under water. See Figure P8. 6 7.

Figure P8.66

Figure P8.67

COMPUTER PRO B L EMS / Chapter 8

8.68 Write a Newton-Raphson iterative algorithm (see 8.69 Use the algorithm in the preceding problem to solve
Appendix D) for solving equations of the form flxl = O. the equation
Use the algorithm" to solve the equation
fIt) = t3 - 7t2 + IS.3t - 1.34
2.SP
flO) = 1 - sin 0 -ki tan 0 = 0 =0

for the critical tub thickness in Example 8.12; compare


from Example 8.3, for a wide range of values of Plkl from with the result found in the text.
zero to very large. Compare with the text when PIlkl) = I.

L
566 Chapter 8 Special Topics

*8.70 The hollowed cone in Figure P8. 70 is sealed at its


base and is being used as a tank. How full is it (H - z) at the
instant. it lifts off the ground? (Assume the seal cannot
exert a downward force on the tank.l
H
"'Y= Specific weight of fluid

Figure P8.70

Answers to Questions I Chapter 8


Q8.1 Zero work.
Q8.2 Not at all.
Q8.3 Only for infinitesimal (j() does C move horizontally and does th'e point of
application of Nand f move vertically.
Q8.4 The distance, I, from A to B is constant during the rigid change in configuration.
Q8.5 Yes, otherwise the cable tensions on the two sides of the pulley would not be the
same.
Q8.6 If () = 90°, we have divided by zero in deriving Equation (8.261.
~.-.. .
.

Q8.7 In the prior result (in Section 2.9) the pressure was constant, and so any projection
could be used. In this chapter, the (hydrostatic) pressure varies linearly with
depth. Thus the vertical projection of the area is the only one it makes sense to
use in the theorem.
Q8.8 It is only able to exert normal/horizontal, here) forces on the column.
Q8.9 Q is the point of intersection of the components comprising Fr'
Q8.10 The resultant of the forces caused by water pressure has to be the negative of the
weight of the displaced fluid (PFgW). Thus if the resultant of these water pressure
forces at CF contains a moment, then ~MCF cannot vanish there.
Q8.11 Disaster! The moment of Band mg would then be in a direction that would
overturn the ship further.

Review Questions I Chapter 8


True or False?

1 A virtual displacement is an imagined one that constraints on the body may


prevent from occurring in reality. l'
"'i

2 An advantage of virtual work is that forces not of interest may be suppressed in q


the analysis by appropriate choices of virtual displacements.
,
"•. Cc

Review Questions 567

3 In using virtual work, we need not worry about the body being in equilibrium in
an inertial frame; any old frame will do.
4 Virtual displacements really need not be restricted to infinitesimals provided
rotations are infinitesimal or zero.
~; ,o"l.t;
5 The method of virtual work cannot be used to find the force of interaction where
two parts of a body are pinned together.
6 The Principle of Virtual Work is restricted to situations where there is no friction.
7 The resultant force due to hydrostatic pressure on a submerged plane surface
always equals the product of the projected area on a vertical plane and the
pressure at the centroid of this projection.
8 For hydrostatic pressure on a submerged plane area, the center of pressure is closer
to the free surface of the liquid than is the centroid of the area.
9 Consider the vertical plane that (a) contains the centroid of a plane area
submerged in a liquid; and (b)is perpendicular to the plane area. The center of
pressure has to lie in this vertical plane.
10 The magnitude of the resultant force on a curved surface equals the pressure at
the centroid of the curved area multiplied by this area.
11 The buoyantforce is always vertical.

- ~_i Answers:1.T 2.T 3.F 4.T 5.F 6.F 7.F 8.F 9.F 10.F 11.T
Appendices
~"
•! ..
l"

Appendix A Vectors
A.I Vectors: Addition, Suotraction, and Multiplication by a Scalar
A.2 Unit Vectors and Orthogonal Components
A.3 Scalar (Dotl Product
A.4 Vector (CrossI Product
,.
; A.S Scalar and Vector Triple Products
~ .~

Appendix B Tables Relating to Units

Appendix C Moments and Products of Inertia of Areas


"

Appendix D Examples of Numerical Analysis/The Newton-Raphson Method

Appendix E Answers to Odd-Numbered Problems

"~-. '

569
--=::l!lO••••------------------- ••• _

Vectors

~A.1 Vectors: Addition, Subtraction, and


Multiplication by a Scalar*
Vectors are mathematical entities possessing the qualities of magnitude and
direction and obeying certain algebraic rules. Although the concepts involved
may be extended to spaces of higher dimensions, our concemshall be with
vectors in ordinary three-dimensional space. In this circumstance it is possible
to represent a vector by an "arrow" -that is, a line segment whose length is
proportional to the magnitude of the vector, with directionality along the line
segment indicated by the arrowhead. "t

~ Many of the features of the algebra of vectors may be either deduced or -.t ':".
1 !:
B readily seen from the parallelogram law by which vector addition is defined i'

Figure A.1 Parallelogram Law of Addition (Figure A.I). Using boldfaced type to denote vectors, we write. :~iJ

C=A+B

Clearly from the parallelogram law there is no significance attributable to order,


so that also
B
C=B+A

which is to say that vector addition is commutative. We also note that the
parallelogram law suggests a slightly different way of depicting vector addition;
~ that is, the same result is obtained by a head-to-tail arrangement as shown in
i

Figure A.2 Figure A.2.

*Examples pertinent to Sections A.I-A3 may be found in Section 2.1 of the text.

570
r

.- A2 Unit Vectors and Orthogonal Components 571

.~
t" Figure A.3

I
r 4
We are now in a position to illustrate (at least for two dimensions) the fact
that vector addition is associative. In Figure A.3 we see that

!
~I.••
A + (B + C) == (A +B) + C
I..•. Thus, we see that, in vector addition, or.der is irrelevant.
The negative of a vector A is the vector having the same magnitude as A but
.". b.
the opposite direction. Writing (~ A) for the negative of A, the parallelogram law
dictates that
[,
I A + (-A) == 0

Furthermore, letting b be the sum of a vector Band (- A) we have

I D ==
or more simply
B + (-A)

L
I'
D==B-A

i. Thus we have the process of subtraction.


The simplest of the processes of multiplication involving a vector is mul-
tiplication by a scalar. A scalar is an entity whose measure is a real number. If
I
t;
a is a scalar, then by aA we mean the vector whose magnitude is the product ofthe
I ..
I
absolute value of a and the magnitude of A, and aA:has the same direction as A if
\ a is positive and the opposite direction if a is neg~tive. The parallelogram law
shows that multiplication by a scalar is distributive-that is

alA + B) == aA + aB

~A.2 Unit Vectors and Orthogonal Compc!.nents


The vector e == A/IAI, where IAI is the magnitude of A, is dimensionless, has a
magnitude of unity, and has the same direction as A. Thus e is called the unit
vector in the direction of A.
Unit vectors of preassigned directions provide the mechanism by which
vectors are usually expressed. Suppose we let (as in Figure A.4 on page 5721 X, y,
and z be mutually perpendicular axes, or reference directions,. and we let 1, i,
\. and k be unit vectors in those directions. The parallelogram law allows us to de-
compose a vector A into three mutually perpendicular parts written AJ, Ari,
!

and A.k so that

A = AJ + Ari + A.k
572 Appendix A Vectors

A,k
r
)y
/'
./
./
./
/'

x
Figure A.4 Components of a Vector

If we write A = lAJ + A31 + A.k, then the vector In the parentheses


extends from the origin 0 to the projection point P of A in the xy plane. This
vector, by the Py}hagorean theorem, has magnitude oj + A; A;.
If to this'vector
we then add Azk (which extends from P to the tip Q of the arrowhead of Al,
then we have succeeded in building up the vector A from its three components.
The magnitude of A, using the (shaded) right triangle OPQ, is then given in terms
of the components of A as

IAI = jojA; + A;2 + A;


= "x/A2+A2+A2 y z
(AI)

We can see that the vectors AJ, A3, and Az~ are the (orthogonal) projections
of A onto the x, y, and 2 directions. They are often called orthogonal components
y
of Ai the same terminology is also used for thJ scalars Ax, A;,'and'.Ai'. When
context does not make clear which components are intended, we distinguish
them by calling the former "vector components" and the latter "scalar com-
x ponents." If OX, 0Y' and Oz are the angles betwe'en vector A and x, y, and 2, re-
"

Figure A.S Angles Whose Cosines AIe spectively, then from Figure A.S we see that
the Direction Cosines of A
Ax=IAI cos Ox

Ay = IAI cos Oy (A.2)

Az = IAI cos ez
These cosines are called the direction cosines of A.- We note that since a'unit
vector has a magnitude of unity, Equations(A 2) show that the direction cosines of
a unit vector are its components.
Squaring both sides in each of Equations (A21, adding, and using Equa-
tion (A. I), we confirm an important property of the direction cosines of a line-
that is,

(A31

Finally, it isof great practical importance to realize that, from the properties
of addition and multiplication by a scalar enumerated in Section A.I,
..
(A41
A.3 Scalar (Dot) Product 573

IX being any scalar. Moreover, if

C=A+B

then

lA.5)

~A.3 Scalar (Dot) Product


The scalar product of two vectors A and B is denoted A - B (from whence comes.
the name "dot" productl and is defined by

~
.. A - B = IAIIBI cos (J

where cos (J is the cosine of the angle between the two vectors. This product, as its
name communicates, is a scalar, and from the definition it is clear that scalar ..
multiplication is commutative-that is,

A-B=B-A

For a unit vectore,

A- e = IAIII) cos (J

so that A - e is the scalar component of A associated with the direction of e. Thus


if, as in the preceding section, we express A by

A = A) + A + A.k y"

then

Ax = AoI
Ay = A-"
Az = A ok
A very important property is that the scalariproduct is distributive over
addition-that is,

Ao(B+Cl=A-B+A-C

We may obtain this property easily if we now define the unit vector e to be A/IAI,
so that

A 0 (B + Cl = IAle 0 18 + Cl

From Figure A.6 on page 574 it is clear that

eo(B+Cl=eoB+eoC
574 Appendix A Vectors

B I
- I
L e = AIIAI•
--'_.
~B. e .f--e. e~1
I--(B + C). e-l
Figure A.6 Distributive Property of the
Scalar Product

so that

A . (B + C) =IAle . (B + C)

= IAlle . B + e ' C)

=IAle . B + IAle ' C

=A'B+A'C

The commutative and distributive properties of the scalar product allow us


to obtain a result of great practical importance. S~ppose we express the vectors A
and B as

f\. = A) + Ayj + A.k


B = B) + B3 + B.k
Then

A . B = (A) + Ayj + A.kl ' (B,1 + B3 + B.k)

= AxBxli ' i) + AxBy(i ' j) + AxB,(i . k)


+ AyBx(j , i) + AyBy(j . jl + AyB,lT . k)
+ A,Bxlk . I) + AzBy(k . j) + A"Bz(k ' kl
But, since i, j, and k are mutually perpendiCular unit vectors, then by the
definition of thescalar product ~e.obtain
"':'" -:- -:- -:- •••• A

l'I=J'J=k'k=1
.•..•. .•. k~ .•. k~
I'J=I' =J' =0

Substituting above, then, we obtain

(A. 6)

This important result tells us how to compute the scalar product of two vectors in
A4 Vector (Cross) Product 575
i
\.
terms of its scalar components. When we apply it to B = A we obtain

A. A = A~ + A; + A;

which was already available by the definition-that is,

A . A = IAIIAI cosIO)
= IAI2
r Finally, before leaving this section we point out that the scalar product gives
I us an elegant and practical way to express the angle between any two vectors,
and, of particular importance in mechanics, it gives us a compact way to compute
the component of a vector in any given direction.

,
I.
f
~A.4 Vector (Cross) Product
A second multiplicative operation between two vectors is called the vector
product and is usually denoted A x B from whence comes the term "cross"
product. The vector product is defined to be a vector so that, if C = A X 8, then
ICI= IAIIBI sin 0, where 0 is the angle (not greater than 180°) between Aand B,
and C is perpendicular to both A and Bwith its direction determined by the' 'tight
hand rule" as indicated in Figure A. 7. The assignment of the direction of C may
be visualized as in the figure by first giving A and B a common point of origin or
"tail," and then by rotating A about its tail so as to line it up with B. If the sense of
turning of A (through angle 0) to line it up with B is mimicked by:curling the
fingers of the right hand, then the extended thumb points in the direction of
C = A x B. This is also the correspondence between the turning and advance
of a screw or bolt with the customary right-hand threads. A sometimes useful
geometric interpretation of the cross product is that its magnitude is the area
of a parallelogram whose sides have as their lengths the magnitudes of A and B.
(SeeProblem A 13 at the end of this appendix.)
From its definition, vector multiplication is not commutative, and, in fact,

B x A = -A x B

f .-

Figure A.7 Direction of the Vector Product

L
576 Appendix A Vectors

--- ----------.--

Figure A.8 Distributive Property of the


Vector Product

Vector multiplication is, however, distributive, and this is an important


.J
property of which we shall take advantage marly times. The property is that -~.
<c"'
.~

A x (8 + C) = A x 8 + A x C (A.71
...
;
To establish this property we begin with a special case. Let 8 and C be
vectors, each perpendicular to A as shown in Figure A.8. Because A is per-
pendicular to 8, to C and to 8 + C, then the vector product of A with each of
these is obtained by rotating the original vect~r through 90° about A as shown,
and multiplying its magnitude by IAI. Thus by the parallelogram law of addition,

Ax (8 + C) = A x 8 + Ax C

We now need to relax the restriction that 8 and Care perpendicular to A. To


that end we decompose the vector 8 into two parts: one part, 81, in the direction
of A imd the remainder; B2, necessarily perpendicular to A. Thus

We now need the result that

A x 8 = A X 82

The illustration shows us that the direction of A x 82 will be the same as


that of A x 8. As for the magnitudes, we have

IA x 81 = jAllBI sin 0

and this equals IA x 821, since

A IA x 821 =IA11821 sin 90°

=IA11I81 sin Oil

Now let us call D the sum of 8 and C and similarly decompose C and D. Thus

A X B2 direction D=8+C
..'.
A,4 Vector (Cross) Product 577

and

DI = BI + CI
and thus

Because B2 and C2 (and hence D21 are each perpendicular to A, we use our
restricted distributive law, which is that if B2 and C2 are normal to A,

A X (B2 + C21 = A x B2 +A X C2
or

A X D2 == A X B2 +A X C2
But

A X B2 = A x B

and similarly,

A X C2 = A x C

and

A X D2 = A x D

so that A x (B + C) = A x B + A x C with any restriction, which completes


our proof.
We close this section with a formula for the cross product in terms of
components of the multiplying vectors. As usual; let 1, ;, andk be mutually
perpendicular unit vectors in the x, y, and z directions, but let us also specify that
this is a right-handed system; that is,

1x;=k

kx1=;

and

;xk=1

It then follows that


-:- ~ .....
J x 1 = -k

1 x k = -;
and

k x; == -1
- ",

and of course

1x1=;x;

=kxk=O
578 Appendix A Vectors

Expressing A and B in terms of components along x, y, and z,

A x B = (A,I + Ayj + A.kl x (B,I + B) + B.k)


= AxBy(I x j) + AxBz(I x k) + AyBx(j x I)
+ AyBz(j x k) + AzBx(k x I) + AzBy(k x j)
= (AyBz ~ AzBy)I + (AzBx - AxBzlT + (AxBy - AyBxlk (A. 8)

Sometimes it is useful to express this result in the form of a determinant,

i j k
Ax Ay Az
Bx By Bz

which will be recognized, when expanded, as the right-hand side of Equa-


tion (A.8).
lt is necessary for the reader to become thoroughly familiar with .the vector,
or cross, product, because it is this operation that lies at the heart of the concept of
the moment of a force (see Section 2.2).

~A.5 Scalar and Vector Triple Products


Two different multiplicative operations, each involving three vectors, naturally
.arise in mechanics. The first of these is called the scalar triple product. lt is, as the
name implies, a scalar, and it results from the sc~lar (dotl product of a vector with
the vector (cross) product of two others, as in e . (A x BI. A sometimes useful
geometric interpretation of this product is that:its magnitude is the volume of a
parallelepiped whose sides have as their lengths the magnitudes of A, B, and C. Of
greater importance to us here is that the value of the scalar triple product is not
changed by certain exchanges of elements; in particular,

e . (A x Bl == A • (B x C) = B • (e x A) * (A.9)

Equation (A.91may be established by returning to the expression for Ax B


given by Equation IA.8):

A x B = (AyBz - AzBylI + (AzBx - AxBzlj + (AxBy - AyBx)k

If we similarly express e by
e = c,I + C) + C.k
then
e . (A x Bl = (AyBz - A.ByICx + (AzBx - AxBzlCy
(A.lOl
+ (AxBy - AyBx)Cz

.This third expression may also be written Ie x A) . B so thai Equation (A.9)


expresses that an identity follows from "interchan~ng" the "dot" and "cross." It is of
course understood that the parentheses, indicating the order of the operations, always
group the elements of -the cross product.
A.S Scalar and Vector Triple Products 579

The right-hand side of Equation (AI 0) may be expressed as a determinant so that

Cx Cy Cz
C. (A x Bl = Ax Ay Az (All)
Bx By Bz

Because the value of a determinant is not altered by two interchanges of its rows,
the right-hand side of Equation (AlII has the same value as

Ax Ay Az Bx By Bz
Bx By Bz or Cx Cy Cz (A121
Cx Cy Cz Ax' Ay Az

The first of these may now be recognized as A • (B x C) and the second as


B . (C x AI, which establishes the identity (A9). We note that, because
B x A= -A x B, then C . (lJ x A) = -C. (Ax BI. This result is also im-
mediately available from the determinant form because one interchangeof rows.,,-
changes the sign of a determinant.
Finally, we point out a feature of the scalar triple product that the reader may
already have deduced. That is, if any two of the three vectors are proportional, *

r
i.
~-" .
the scalar triple product vanishes. This follows from the fact that the cross'
product vanishes if the two vectors are proportional, since one of the forms of
Equation (A.91 will have the two proportional vectors in the cross product. The
result may also be deduced from the determinant form since a determinant
vanishes if two rows are proportional. It is also see,r from the fact that, in this
case, the parallelepiped mentioned in the opening paragraph has zero volume.

!•
f"

r.~. E X A M p L E
For the vectors:

A= 21 --3; + k
B = 51 + ; - 4k
C = 31 + k
find the scalar triple product C . (A x B).

SOLUTION

C • (A x B) = C • [(21 - 3; + kl x (51 + ; - 4k))


(Continued)

*For example, A = (XB, where (X is a scalar.


and omitting the zero cross products such as 1 x I,
= e . [2(1)(1 x ,I + 2(-4)(1 x k) + (-3)5(, x 1) + (-3)(-4)(, x k)

+ 1(5)(k xII + 10Hk x,))


= (31 + k) . .12k + 8, + 15k + 121 + 5, - 1)

= (31 + k). [Ill + 13, + 17k)

= 3(11) + 0(131 + 1(171 == SO

Interchanging the dot and cross and recomputing as a check, ,


f
(e x Al . B = [3(-3)(1 x ,) + 3(1)(1 x k) + 1(2)(k x 1)

+ 1(-3)(k x,)) . B

= [- 9k - 3, + 2, + 31) . B
= (31 - , - 9k) • (51 + , - 4kl,
"•.•..!' .".~'

= 3(5) + (-110) + (-9)(-4) = SO

The reader is encouraged to confirm this result by using the determina'nt form,
Equation (A.I I).

A vector triple product is the result of the cross product of a vector with the
result of a preceding cross product as in A x (B x cr.
The parentheses, which
denote the order of multiplication, were really unnecessary in the case of the
scalar triple product because the result of a dot product is a scalar and the cross
product of a scalar and a vector is without meaning. Here, however, the
parentheses are needed because (A x BI x e is itself a legitimate veCtbNriple
product. Moreover, in general

A x IB x Cl * (A x BI x e

We may easily illustrate this fact by an example: let I, J, and k be the usual right-
handed system of mutually perpendicular unit vectors; then

1 x (1 x ,) = 1 x k = -,

But
11 x II x , = 0 x , = 0
i""
which establishes the importance of the order of multiplication. ,i
l
Identities involving vector triple products'that will be particularly useful in
dynamics are

A x (B x Cl == (A • ClB - (A . Ble (A. 131

and
(A x BI X e =(A. ClB - A(B. Cl (A. 14) !
l
Both (A. 131 and (A.141 can perhaps be rem~mbered by noting that in both
formulas, the answer is "the vector in the middle times the dot product of the

580
A.S Scalar and Vector Triple Products 581

other two, minus the other one in parentheses times the dot product of the other
two."
To prove the identity (A.13I, we apply the component form (A.71of the cross
product to B x C:

B x C = (ByCz - BzCyl1 + (BzCx - BxCzl; + (BxCy - ByCxlk


Now letting B x C play the role of B in Equation (A.71,

A x (B x q = [Ay(BxCy - ByCxl - At(BzCx - BxCz1l1


+ [At(ByC. - BzCyl - Ax(B"Cy - ByCxll;

+ [AxlBtCx - BxCzl - Ay(ByCz - BzCyllk

= (AyCy + AzCzlB) + IAzC. + AxCxlB3


+ (AxCx + AyCy)B.k - HAyBy + AzBzlC)
+ (AzBz + AxBxlCy; +(AxB~ + AyBylC.k1
If on the right-hand side we add and subtract the vector

AxBxC) + AyByCy; + AzBzC.k


then we have, after regrouping,

A x (B x q = (AxCx + AyCy + AzCz)(B) -+- By; + B.k)


- (AxBx + AyBy + AzBz)(C) + Cy; + C.k)
But by Equation (A.6),

AxCx + AyCy + AzCz = A . C

and

AxBx + AyBy + AzBz == A. B


Therefore,
, '

A x (B x q =(A . qB - (A . BIC

which is the desired result. Identity (A.141 may be similarly established.

E x A M p L E A.2~
Determine A x IB x q for the vectors
A = 21 - 3; +k
B = 51 +; - 4k
C = 31 + k
(Continued)
SOLUTION
A x (B x q = (A - qB - (A - BIC

= [2(31 + L:- 3)0 + 1(11](51+ , - 4k) f


.~

- [2(51 + (-3)1 + 1(-4)](31 + kl


== 7(51 +

= 261
, - 4k) - 3(31 + k)
+ 7, - 31k
••j..
,
i.

The reader is encouraged to confirm this result by first computing B x C and


then "crossing" A into it.

PR OB L EMS / Appendix A ."


'f

.-~

A.1 Find A + B and A - B if A == 21 + 6, - 4kand Find:


B = - 31 - , + 10k.
(A -BIC, A- BxC,
A.2 What is the unit vector in the direction of the vectOr
(31 + , - k)? (A x B) x C, Ax (B x q
For the following sets of vectors in Problems A.6-A.9,
A.3 Find the dot product of the vectors:
find: (a) A + B; (b) Ii - C; (c) A- B; (dl B x C;
F = 21 + 3; + 6k Ie)A- (B x q; (f)A x (B x q; (g)the unit vector in the
direction of C; (hIthe direction cosines of B.
Q = -I - 6; - 3k
A.6 A == 1+ ;;B = 2,-3; <.+' 4k; .','",:,
A.4 Make up a rule about the direction of A x B that C = - 51 + 4; - 2k .
relates to a right-handed screw turning from A into B. (See
Figure PAA.) A.7 A = 2041 - 6.3k; B = 4.11 - 20.5; + 6.0k;
c=s.I1+8.7,

B A.8 A = 31; B = - 7; + k; C = I+ ; + k
A.9 A = 21 - ; + k; :B = 151 - 20, + 18k;
C = I + 7k

A.10 Find the cross product of the vectors

P = 21 + 3, + 6k and Q = -I - 6, - 3k
A.11 Find the dot and cross products of

Figure PA.4
A = 21 - ; + 3k' and B = 31 + 2J - Sk
A.12 Find the angle between k and the sum of the vectors
in Problem A. 11.
A.5 Given:
A.13 Show that the magnitude of A x B equals the
A = 61 + 2; + 9k, B = 101 + 6k,
"area" of a parallelograni having side lengths equal to IAI
C = 21 + 4} + 6k and IBI.

582
A.5 Scalar and Vector Triple Products 583

A.i4 Find the angle between the forces FJ and F2 in A.i7 The three vectors A = 21 + 3" B = S, + k, and
Figure PA.14. C = 1+ , + k define a parallelepiped with the origin 0
as one vertex~Find the length of the diagonal from 0 to
the opposite comer.

A.iS Show that any vector C perpendicular to both A and


B has the form

AxB
C = :t ICI IAx BI

A.i9 Given A = 31 + 4J and B = 4, + k, write unit


y
vectors (a)having the same direction as A; (b)perpendicular
to B and lying in the xy plane; (c) perpendicular to both
A and B.
Figure PA.14 A.20 The vector D in Figure PA. 20 has a magnitude of
60. Resolve the vector into, two nonorthogonal compo-
• A.iS In Problem A. 14, find the distance between forces F4
nents, one along AB and another along BG.
and F3 (i.e., the length of the line segment intersecting the
two and perpendicular ,to ,them both).
, A.i6 Show that A . (B X q is the volume of the paral-
,~_\ lelepiped in Figure PA.16.
~ .
IS

A'

Figure PA.16
----~----.I'
Figure PA.20

'"

* Asterisks identify the more difficult problems.


Tables Relating to Units

The tables in this appendix are useful in identifying and converting .units
between the SI and U.S. systems.

Table B.1 Units Commonly Used in Engineering Mechanics

51 (Standard International
Quantity or "Metri.c") Unit u.s. Unit

force newton (N) pound (lb)


mass kilogram (kg) slug
length meter (m) foot (ft)
time second (s) secorid(sec) .
moment of force Nom lb-ft
work or energy joule 0)( = N . m) ft.lb
pressure or stress pascal (Pa)("= N /m 2) lb/ft2
velocity m/s ft/sec
angular velocity rad/s rad/sec
acceleration m/s2 ft/sec2
angular acceleration rad/s2 rad/sec2
mass moment of inertia kg m2
0 slug-ft2
moment of inertia of area m4 ft4
E
momentum kg m/s
0 slug-ft/sec
moment of momentum kg. m2/s slug-ft 2/sec
impulse N. s lb~sec
angular impulse Nom s0
Ib-ft-sec
mass density. kg/m3 slug/ft3
specific weight N/m3 lb/ft3
power watt (W)(= J/s) ft-Ib/sec
frequency hertz (Hz)(= 1 cycle/s) . Hz (same)
i ,r

J~
584 __ •..••.. -o'm:;i:;;,;'._
r
Appendix B Tables Relating to Units 585

Table B.2 Standard Prefixes Used in the SI System of Units


j"
tera T 1012 centi c 10-2
giga G 109 milli m 10-3
mega M 106 micro Jl 10-6
kilo k 103 nano n 10-9
hecto h 102 pico P 10-12
deka da 101 femto f 10-15
deci d 10-1 atto a 10-18

Table B.3 Conversion Factorsfor SI and U.S. Units*


ReciprocalIto Get
To Convert From To Multiply By from SI to U.S. Units)

Length, area, volume


foot (ftl meter Iml 0.30480 3.2808
inch (in.) m 0.025400 39.370
statute mile (mil m 1609.3 6.2137 x 10-4
foot2 (ft2) meter2 (m21 0.092903 10.764
inch2 (in.2) m2 6.4516 x 10-4 1550.0
foot3 (ft31 meter3 (m3) 0.028317 35.315
inch3 (in.31 m3 1.6387 x 10-5 61024

Velocity
feetlsecond (ft/sec) meter/second (m/sl 0.30480 3.2808
feetlminute (ft/min) m/s 0.0050800 196.85
knot (nautical mi/hr) m/s 0.51444 1.9438
mile/hour (milhrl m/s 0.44704 -2.2369
mile/hour (mi/hrl kilometer/hour (km/h) 1.6093 0.62137""
~,.

Acceleration
feetlsecond2 (ftlsec2) meter/second2 (m/s2) 0.30480 3.2808
inch/second 2 (in./sec2) m/s2 0.025400 39.370

Mass
slug (lb-sec2/ft) kg 14.594 0.068522

Force
pound lIb) or
pound-force (lbf) newton (Nl 4.4482 0.22481

Density
slug/foot3 (slug/ft3) 515.38 0.0019403

Energy, work, or moment of force


foot-pound or pound-foot joule (J) 1.3558 0.73757
(ft-Ibl llb-ftl or newton . meter (N . m)
(Continued)

*Rounded to the five digits cited. Note, for example, that 1 ft = 0,30480 m, so that

0.30480
(Number of feet) x ( ----
m) = Number of meters
1 ft
586 Appendix B Tables Relating to Units

Table B.3 (continuedl


Reciprocal Ito Get
To Convert From To Multiply By from 51 to U.S. Units!
Power .
foot-pound/second watt (WI 1.3558 0.73756 of:

(ft-Ib/sec)
horsepower (hp)
:';~
W 745.70 0.0013410
(550 fdb/sec) .- -'~

Stress, pressure
poundlinch2 N/m2 (or Pal 6894.8 1.4504 x 10-4
IIb/in.2 Orpsi)
pound/foot2 N/m2 (or Pal 47.880 0.020886
(lb/ft21

Mass moment of inertia


slug-foot2 kg'm2 1.3558 0;73756
(slug-ft2 or Ib-ft-sec2)

Momentum (or linear momentum)


slug-foot/second kg'm/s '~ 4.4482 0.22481
(slug-ft/sec)

Impulse (or linear impulse)


pound-second N . s (orkg . m/s) 4.4482 0.22481
(lb-secl

Moment of momentum (or angular momentum)


slug-foot2/second kg'm2/s 1.3558 0.73756
(slug-ft2/secl
-l.-
Angular impulse
pound-foot-second N. m. s (or kg' m2/s1 1.3558 0.73756
(lb-ft-secl

RI
Moments and P:roducts of
Inertia of Areas

Shape Area Moment of Inertia

Parallelogram
Y (b + h cot a)12 bh bh3
I
x
== -"- 3
bh ' b2h2
Iy = ~ (b +hcot IX) 2 - --- cot IX
3 6
bH2
Ixy = -- (3b + 4h cot IXI
x 12

Rectangle bh bh3
I =--
x 3
hb3
I =-'-
y 3
~
-,

b2h2
Ixy = ,.,---
4
x bh3
I =-,-
Xc ,12
. hb3
I =--
,Yc 12
IxyC = 0

'587

!
I
!

j
Shape Area Moment of Inertia

Triangle bh3
1 =-

-) 12 ":.'
x

bh
1
y
= _(a2
12
+ ab +b21
2.
.
h
1
xy
= - _.bh24 (2a + b)

_J
x

Right Triangle bh3


1 =-
x 12
hb3
!.=-
y 4
b2h2
x 1
xy
= -_.- 8
bh3
1 =_.-
Xc 36
hb3
! "'=..-
Yc.; 36'"
b2h2
1= ---
XYc. 72

Circle nR4
lxc = lyC =4
lxyc.= a

Semicircular Area y nR4


1=1=-
x, y 8
lxy = a

-t
R

-~

588

I
Shape Area Moment of Inertia

Quarter Circle y nR4


1=1=-
x y 16
R4
Ixy =-- 8

-+-
4R
3n:

I---R -I x

Annulus nlR: - Rf)


Ixc = IyC = - 4

IxyC = 0

Thin Annulus 2nRt It« R)

Thin Annular Sector tRa (t« R) tR3


Ix ~ -. - (a ~ sin a)
2
iR3
Iy ~ -. -- (a + sin a)
2
Ixy = 0

!. x

1- :

I
r",~
~~-
589

J
~,,:.

:1
Shape Area Moment of Inertia

Circular Sector y rxR2 4


rxR
1=,-, (
1 _ sin rx),
2 x 8 rx
rxR
4
( sin rx),
1=-'-,
y 8
I +--
rx
. Ixy = 0

Segmento' a .Clrcle y R2 R4
~ (rx - sin rxl Ix =-
24
(3rx - 4 sin rx
.
+ sin rx cos rx)
2 .
. ,R4
Iy = -(rx - sin rx cos rx)
8
Ixy: =:0

Ellipse nab ,. nab.3


Ixc"'" ~,."

nba3
'Yc
I =--
4
IxyC == 0

Semlelllpse nab .nab3


1=:-,-
2 x' " 8
na3b
.1 =-'-
8
r
b
y

Ixy =,0

I- 2a-.I~

590
!
i
I Shape Moment of Inertia
Area

I Segment ofa Parabola y


1ab I = -ab3
4
x 15
4 3
Iy = ya b
b
Ixy = a
x
b

Bent Strip y 2ft (t« I) 2f3t cos2 ex


lx::::::----
3
213t sin2 ex
Iy ~----
3

,
,
i
.591-

~-
_f

Examples of
Numerical Analysis/
The Newton-Raphson
Method

1(0) There are a few places in the book where equations arise whose solutions cannot
easily be found by dementary algebra. These equations are either polynomials of
degree higher than two, or else transcendental equations. .
In this appendix, we explain in brief the fundamental idea behind the
Newton-Raphson numerical method for solving such equations and others of the
form f(O) = o. We shall do this for two equations. The .firstis
f(O) = 1 - sin 0 - 2.5 tan 0 (D.1)
o which occurs in Section 8.1.
4
We are looking forthe smallest positive rOot0 of Equation (D.1).We see that
the function has the value unity at 0 = 0 and its slope, -cos 0 - 2.5 sec20,
is - 3. 5 there. Furthermore (again by inspection), f(O) is negative at 0 = 71:/4.
Thus we expect a root between 0 and 71:/4. The Newton-Raphson algorithm,
..•found in more"detailin.any book onnumerical.analysis, works ,asfollows> If 00
is an initial estimate of a root 0, then a better approximation is
f(Ool
0, = 00 - 1'(0 ) ~
0 .~
The figure shows what is happening. The quantity f(8ollf'(Oo) causes a 1
backup, from the initial 00 approximation, toward the actual root we seek. *
Let us use the initial guess 00 = 71:/4 (see Example 8.3). Then f(7I:/41 =
-2.207106781 and 1'(71:/4) = - 5.707106786 sothat the improved estimate is
-2.207106781
0, = 0.785398164 - -5.707106786 = 0.398668624

*The reader may wish to show that the idea works as well for the other three possible
sign combinations of 1 and f' Ii.e., + +, + -, and - +).

592
--::;:_::---------------------------------------------------

Appendix D Examples of Numerical Analysis/The Newton-Raphson Method 593


~'.

Repeating the algorithm, f(Od = -0.441253558, !'lOll = -3.865154335, and


so

o = 0.398668624 - -0.441253558 = 0.284506673


i. 2 -3.865154335
".
Further applications of the Newton-Raphson algorithm yield

03 = 0.281298920
04 = 0.281297097}
Os = 0.281297097 convergence!

06 = 0.281297097

We have nine good digits after just four iterations, and the approximation
o~ 0.281 agrees with the solution in the text.
As a second example, we consider the cubic polynomial equation from
Example 8.12: .

fit) = t3 - 7t2 + 15.3t - 1.34 = 0 (D.2)


!
f.,' We begin with the estimate to = 1 in. and quickly obtain, using fit) =
I ' 3t2 - 14t + 15.3,

f(to) 7.96000000
tl = to - -- = 1 - ---- = -0.851162791
f(to) 4.30000000 -

Continuing, the results are:

tl = 1.000000000

t2 = -0.851162791

t3 = - 0.168924556

t4 = 0.063694406

ts = 0.090990090

t6 = 0.091349686

t7 = 0.091349748}

ts = 0.091349748 convergence!

t9 = 0.091349748

This time we have eight significant digits after only seven iterations. Note
that we have in fact found the only root of Equation (D.2I, as the reader may wish
to show by examining the values of f at the two roots of f = O. (Note that f is
negative at t = 0 and positive for large t, and that by Descartes' rule of signs/the
maximum number of positive real roots is three, and of negative ones is zero.)

[
~;" -
.'f'

Answers to
Odd-Numbered Prablems

Unless suggested otherwise by the problem statement or figure, i, ;, and k are unit
vectors in the respective directions ->, i, and out of the page.
.,;
1.'
CHAPTER 1
~o
2.25 50.2 i Nand 39.8~: N .
1.1 An automobile is parked on a hill. Find the forces
exerted on the tires by the road.
2.27 (a121 (
- 2i + 6;
7
-3k) .
..
.,. .,.
Ib(b) -61+,18J'+"9k
A

lb
1.3 Velocity is defined from the concepts of space and
time. The dimension of velocity is LIT. Energy is
defined from the concepts of force and space; the
2.2. Amw« giv::S::~bl,m A
dimension of energy is L - F. 1.5 1.609 km
1.7 1060 T 1.9 Yes; both sides have dimension LIT. 2.31 FPR =( ~O:.~ ~;2F=R) = 418
0

20 N
1.113.44 x 1O-8ft3/(slug.sec21[orft4/(lb.sec4)] 2.3397.5 13i -13j + 13k lb;
1.13 0.7 m 3; one significant figure

CHAPTER2
(8~ IL)
122 - i + - jIb; 42.2j lb; no
17 17
~
2.35 20i + 77.5; lb and 20i - 77.5; lb
2.1 F2, IF21 = 9.00 N 2.37 0.0683i + 0.820; - 0.569k "2.39 IFd = 12.51b
2.3 Result is apparent from Figure 2.2. 2.41 4700k Ib.ft 2.43 1830k lb.ft
2.5 .(a)87.8 N (b)0.456,0.569, -0.684 2.45 -174i + 13.4; + 6,6.8k lb-ft 2.47824 N .
2A3A 6 A 2.49 400i - 800; + 300k IMt '~..•'l';
,
2.7 - i - - j + - k
777 2.51 (al - 64i + 240; + 6k lb
2.9 From 2.6, F = 70i - 20; ::t 68.6k lb (b) - 36i - 241 + 240k lb.ft (e) - 241 lb-ft
For +,(F - ele = 42.1(-0.3i + 0.1; + 0.949kllb 2.53 (al -7.07i - 10.6; - 7.07k lb-ft (b) ~ 1O.6jIb-ft t
For -, (F- ele = - 88.1 (- 0.3i + o. I; + 0.949k) lb
2.55 715(-~, + ~k)' lb.ft
2.11 -97; - 48k lb 2.13 0.8i + 0.6; 2.15 37.5 N 13 13
2.17 (aI25.7i - 10.7; + 16klb 2.57 (al 1080i + 1200~ lb-in.
(bI55.Ii - 37.1; + 16k Ib (b)900(0.667, + 0.750k)lb.in.
2.190 2.21 ~ 15 N - m 2.23 0.1 lb.ft (c)- 81oi ~ 1350, - 2850 k lb.in.

594 -.•..
Appendix E Answers to Odd.Numbered Problems 595

(d)923(0;.5431 + 0.466, - 0.699 k) lb-in. 2.129 Fr = -21 lb, along the line y = 9 in. (doesn't
2.59 18k lb.ft cross x axis!l
2.61 (a) -0.231,0.923,0.308 (b) -91 + 36, + 12k lb 2.131 Fr = 31 + 3, lb, crossing at x = 5 in. Equation is
leI 961 + 84} - 180klb.ft (dl 84, lb-ft (el 0 y=x-5
2.133 Resultant is couple, C = 301 + 480,lb-ft
2.63 (a)F = 520(' :"'~1 +~, - ~k)N 2.135 40k N; x = -3 ID; Y = 3.46 m
13 13 13
(b) 01 + 160(31, + 480(3)k N . m 2.137 (a) 1201 - 50,lb, 190k lb-ft
lei 480, + 1_440kN . m (d) Same as (cl (b) 1201 - 50,lb, intersecting x axis at x = -3.8 ft
2.65 - 800k lb.ft 2.67 3601 + 360} lb.ft 2.139 301 lb; 25 in. above 80-lb force
2.69 -541 + 18JIb-ft -2.71641 N . m 2.141 (a) 3Pl - P, + 2Pk, Pbl - 2Pc, + 3Pak
(b) 6a + 3b + 2c = 0 (cl No, Fr #- 0
2.73 132,N' m
2.75 483,000 k lb-in. about 0, A, and any other point. 2.143 Yes; IMrpl/IFrl
2.77 109UAB lb.ft 2.145 (a) 10k lb, - 801 + 105; lb.ft
2.79 Evaluate rEDx lAD and divide by its magnitude. (b) IOklbat(-10.5, -8)ft
2.81 - 74k lb-ft 2.83 ~ 52.0k lb.ft 2.147 Fr = -21 + 12, - 9k lb;
2.85 - 792k lb-ft 2.87 - 176u, N . m MrA = 61 - 9; - 15k lb-ft; No, Fr • MrA #- 0
2.89 -3, + 22kN . m 2.149 Add 11lb at A; original's screwdriver: -II
lb,
1 ft below A
2.91 250 ~ lb at Q along with 1560k lb-in. 2.151 41 - 12; lb; 261 + 24, + 48k lb-in.;
2 Fr' MrA #- 0
couple; 1:MB = 2790k lb-in. 2.153 -6k N along x = 0.667 m, y == 1.5 m.
2.93 1,7301 lb; I,OOOJIb;-10, 700k lb-ft 2.155 -1.731- 3,lbthroughx =-2.5ft,y = 0
2.95 Second force is - 180}N, along x = - 70 cm. 2.157 3k lb through the origin
2.97 Mo(0.8941 + 0.707, + 0.154k) 2.159 Fr = 3P1 - P; + 2Pk along 28y + 14z = 9a,
2.99 0 (two); 1180 lb (two up, two downl;
1670 Ib (one up, one down) 28x - 42 z = 73a; C = Pa (75 - 25 -
-i - -j +-k
50 -)
14 14 14
2.1,01 61 + 8, - 8k lb; -11.21 + 24} + IS.6k lb.ft
2~161 - 501 - 120, N intersecting a horizontal line ..
2.103 F = 6, - 4k lb; C = -22, - 33k lb-ft
through A 4.33m to the right of A.
2.105 F = 401 - 20, - 30k lb;
2.163 ~ = 91_+ 5, lb along z == 9 ft, 5x- 9y'''= 0;'
Mro = 401 + 180} - 120k lb.ft
C = 45i + 25j lb-ft
2.107 Fr = -91 + 12j + 16klb;
2.165 Write y = (Fr/FrJx + Yo and evaluate Yo from
MrA = - 361 - 18, - 90k lb-ft
the moment condition.
2.109 Fr == -5481 + 3, - 1020k N;
2.167 61 + 8, - 8k lb, piercing xy plane at (3, 1.4) ft
MrA = 12901 - 634} - 1520k N . m;
2.169 100,lb at x = - 360 ft
Mro = - 8541 - 634} - 368k N . m,
2.171 -21.3 x 1041lbaty = 50ft,z = 300ft
which = MrO + lAO X Fr 2.173 4340 lb (tall buildingl; 2870 lb(wide building)
2.111 Fr= 0.8661 - LSD; lb; Mrc = 0
2.175 la) The infinitesimaIiforce on any element of the
2.113 (a) 241 - 24} - 32k lb-ft
base is parallel to the axis, and the force on any element
(bl 361 - 36, + 48k lb-ft
of the curved surface has line of action through the axis.
leI61 - 6, + 8k IMt (dl 0 (e)24k lb-ft
(blUse item (11on page 107:
2.115 Fr = -30k kN; Mrc == -91 - 4.5;kN . m
2.177 Fr = .150nlN; MrQ =.= 0
2.117 6001 - 504} N through 0.
2.179 0,0
2.119 -2k N through (3, l)m
2.181 Letting x be measured from left end of beam,
2.121 -401 + 550, lb through intersection of lines of
300 lb down at x= 1 ft, 700 lb down at x = 3.5 ft, and
action. 600 lb up at x = 8.33 ft.
2.123 -90.31 - 88.6, + 369k N through A.
2.183 (a) -61 - 10, - 7kN' m
2.125 19.31 - 11.6J1b through intersection of lines of
1
action. (b)2e + 4;3- 12k) N_' m
2.127 8501 - 1110, lb; y + 1.35x = 1.11; at
(cIA.5m 2.185 -280k N; (x, yl = (-78.2,96.1) m
x = 0.82 ft
596 Appendix E Answers to Odd-Numbered Proble

C H A P T E R 3 3.9 z

3.1
c;

.•A
T,

B
c,
t
Bx
3ft-..
2ft

T, 40lb By
y

x
3.3

3.11 (a) T = lOJR + I>H lb; N == 10/H',,- lb


2

(bl T -> JOlb and N --> 0, which checks since the cable is
nearly vertical.
3.13 (alAC: 2680 N; BC:2240 N(b) 96.9°, 1220 N
3.1541.7lb 3.17 392 N
3.19 FI = 40 lb; F2 = 30lb 3.21 0.3091
3.5
3.23 491 b o
N and:850 5\1 ~
3.25 53.0 N 3.27 190 lIb

where Wsin 81 Wsin 82


\'1
. ".
the angle is 3.29 .. . . and ----- .~'.,O.'.'
sin(82 - 8tl! sin(82 -:- 811
computed
3.31 57.6° 3.33 23.2° ,,'3'.35 4.5Tlbeach,
using,
3.37 Since T =J25/cos(f, the tension increases as8
increases. Also, [J = T, so when T = man's

weight = 800Ni8max = COS-IG~~) = 81.0°.


3.7 3.39 No. Letting P be the point of intersection of the 26-
and 27. 2-lb forces, we seeJhat LMp # 0 unlessF O. =
And even if F = 0, LF # O.
3.41 B cannot lie on a line through A parallel to the y
B
Bx axis; 3.43 At A, 50..- N; at B, J90 j N
3.45222 jIb; 0.919 ft

_ 3.47 (a)30 ;,(b) at A, 120:j lb; at C, 120 3~


0
lb
3 3.49 On top face, 1263 lb L tme-thirdof distance from
14 ft
~ left end; on bottom face, 12331Oj, one-third of distance

A~
t
2ft
t _! from right end; smaller distance between resultant
forces. '
3.51 WL/(2tI; The other tension is <0 if I < L/2, and
"you can't push a rope."
3.53 (a)229~Olb; 2421b; J20~lb-in.
Appendix E Answers to Odd.Numbered Problems 597

3.151 250 N tension; 0.0500 m


3.55 At A, 1901 N; at B, 303 ~o N; 3.153 COS-I (::z} cosines can't begreater than 1
atC,85.6 ~ N 3.15529.0°

3.57
- .
At A, 250 1 N; at B, 50 1 N
3.157 Solution outlined in problem.
3.159 To Pt, 101 - 24k lb; to Pz, -101 + 24k Ib
3.59 wL/4 1 at each end 3.161 30 N 3.163200 N;'200 N
3.61 At A, 800 lIb; at B, 1000 lIb 3.165(aII00 lb; 1/2 3.161 W/4
3.63 At B, 3500 rib; at D, 500 lIb 3.169 Case (a): On painter, 110 lIb from rope, 180 lib
3.65 At A, -2161 - 178; lb; at F, 1781lb from gravity, and 70 lIb from scaffold. On scaffold,
3.67 1501 + 1500; Ib and 9000 C> lb.ft 110 lIb from cable, 70 lIb from painter, and 40 lIb
,i. 3.69 7110 lb; 45701 + 7450; lb from gravity. Case (b): On painter, 180 lIb from gravity
3.71 86.2 c> N . m and 180 lIb from scaffold. On scaffold, 180 lIb from
3.73 2.80 ft 3.75 4790 lb (compressionl painter, 40 lib from gravity, and 220 lIb from rope.
3.77 89,600 lb (tension); -17,8001 + 112,000; lb 3.1712625 N
3.79 At C, 2201 N; at D, 10601 N 3.81 0.897 W 3•.173157 Nj 530 1N 3.175250 N
3.83 12.1 -+ N 3.851710 -+ N 3.177 Middle cable, 1650 lbj outer cables, 165 I lb each
3.87 Tension ismg/2 3.179 From left to right: (aIAB (b) ABj BC; EF (c) BD
3.89 3.03 ft and 48.5 tons; 54.2 ~ We ~ 100 tons (d) BD3.181 222lb
3.91 401+ 80;lb .3.93 13W/11 3.183 At bottom, 280 1 N j from inclined plane,
3.95 () ~ 7.18 (considered only ()'s between 0 and 90°1
0

3.97 It blows over; the moment imbalance about B is


3450Ib.ft. 3.99 5.321 N 3.101 24.8 cm
127 ~o N; from.A, 184 j4 N
3.103 89.6Ib3.105 5260 lb compression
3.107 AtA, 10,9001 - 14,500;N; at B, 3.185 25/sin IX lbj 25 cot IXlbU on top; 1 on bottom)j
zero 3.187 14.1 <- lb
3.189 (a) 2W(D - 2R)/D (~) Answer given in problem.
18,200 ~ N
3.191 (al 34.7° (bl 181 N tension in AB;
114 N compression in BCj 125N tension in AC
3.109881 - 108;lb 200
3.19343.7 N
3.'" OnA, ~ 112' + 1360jIb, on C. 328 ~ Ib 3.195 (a) Qr/(2.jRz + 2Rr)(b) (P + Q/2) 1
(c)Q(R + rl/(2.jRz + 2Rrl
, 3.113 115 N j 120 N in each rope 3.197 Answer given in problem. 3.199 52.5°
'- ..•~.,
3.115592 N each 3.1171450 N 3.1190.581 m 3.201 2 mg/n
3.121 56.6lb each; no. 3.123 292 N 3.20327.2°
3.125 16.61b compression 3.205 15.0 P
3.127 Cable 2,324 N; Cable 1,240 Nj at A, 1281 N 3.207 (a) 79.8 lb (b)Tl.'3'lb:to the right.
3.129 DA: 294 N; DB: 208 N; DC: 339 N 3.209 Answer given in problem. 3.211 0
3.131 0.214 mg; all forces pass through the bar, so that 3.21349.7Ib 3.215 47,900 Ib compression
moments about its centerline add to zero automatically. 3.217 2c(b + c + d)/(ad),Jesser
3.133 Answer given in problem. 3.219 At B, 64.0 kNj at C and at D, 47.2 kN
3.135 FBF = 3840 Nj FeE = 7510 ~ . - 3.221 (a)ACA, - 1601 + 280; lbj atE, 200jlb
3.1372021 + 122; + 82klb; 20klb.ft (b)at B, - 2401 - 140; lb; at C, 4001 -' 140; lb
3.139371 Nj CBy == 24.9 N . m; CB, = -31.2 N . m 3.223 (a)361 -+ N (b) 194 '--+ N
3.141 At A, -12,OOO;+6,000kN; 3.225 319 Ib tension (each Strut);
at B, - 12,0001 + 12,000; N; at C, 16,0001 - 4,000k N 3051 + 1,030; lb (each pin)
3.143 144 lb compression each ~ 3.22724,000 Ib compression
3.145421b 3.147 8 3.229 (a)At A, 39.01 - 353; N; at D,
3.149 -101 - 3; - lOOk lb; -39.01 + 549;Nj (b)at B, 1571 + 549;N;
- 10601 + 3200; + 10k lb.ft atC, -1961 - 196;N
598 Appendix E Answers to Odd-Numbered Problems

4.7 AC: 150 lb QJ; AF: 175lb <Q; AB: 90 lb Q);


3.231 At F, 4361 + 390, N; II 70 ~ N onto EF: 90 lb <Q; BF: 350 lb @; CD: 233lb @;
, ~o
DE: 120 Ib,QJ; BE: 408lb QJ; CE: 1100 lb @;
BDE at D
3.233 (a)4000 sin(812)lb (b)2000 lb directed from B BC: 120 lb @; FC = 0
4.9 CD: 167 N @; AC: 208 N QJ; others: 0
towardC '3.235-": 7501 +600, lb
4.11 AB: 667lb @; BE: 1200 lb @; BC: 2670 lb @;
3.237 - 1771 - 125, N
CD: 2670 lb @; BQ: 2670lb QJ; CQ: 3330 lb Q);
3.239 9600 lb compression; - 73501 - 4970,lb
QC: 0; QD: 0
3.241 At D, 2 !kN; at B, 4 i kN
4.13 OH: 4 k Q); AB: 3 k'@; BH: 3.89 k QJ
3.243 Force = 1,5001 + 1,500jIb and
4.15 AB: 0.415W@;AC: 0.915W@;
couple = 10,500 <:> lb-ft 3.245 Cy = 0; no.
BC: 0.215W Q)i BC: 0.414W <Q; CC: 0.704W@i
3.247 a = 1.23 ft, b == 0.77 ft; new ball
FC: 0.289W@; CD: 1.05W <Qi CF: 0.395W Q);
force = 1020 lb (and was 289 in 3.246); on trailer tires,
540 lb each (and was 5061 3.249612 N; 520 N
DF: 0.354W @; DE: 0.317W <Qi EF: 0.183W CD
4.17 AB: 67 lb QJ i AE: 83lb <Qi BC: 400 lb CDi
3.251 1410 lb compression; 382 -+ lb by AB;
BE: 50 lb CD; BF: 417lb @; CD: 400 lb CD;
-3821 - 7l0,lb by CD 3.253480 -+ N
OF: 400 lb <Qi DF: 500 Ib:<Qi EF: 67 lb <Q
3.255 1.09W 3.257 1860 N
4.19 AC: 3.33 kN CDi AF;: 18.3 kN CD;
3.259 (a)At A, - 1201 - 147, lb; at E, 427, lb
AB: 9.0 kN CD; CD: 3 kNiCD
(b) 1201 + 333,lb
4.21 CE and CF: 0; DE, BF, and FC: 19.2 k CD;
3.261 (a)At A, 251 - 25, lb; at B, - 251 + 225,lb
BF: 8 k @; BC and CD: 20.8 k @i AB: 27.7 k @i
(b)on EB at D, 350 ! lb 3.263 - 9581 + 708,lb .,
3.265 120 lb of compression on pecan; 100 lb of tension
AC: 10.7 k QJ; BC: 8.33 k CD
4.23 FI = F2 = Oi F3 = 3.54 k CD; F4 = 20 k CDi
in linkAB
Fs = 63.5 k @
4.25 BC: Oi AC: SOkN @i CC: 39.3 kN QJ
3.267 (a)On .A 384 ~o N (normall and
4.27 Start with a triangle in the middle and work
outward. '
165 3~ N (friction) (bl On B, 450 i Nlnormal and
4.29 m = ml + m2 - I, P = PI + P2 - 2,
165 <- N (friction) 3.269 3130 <- N 3.271 859 N ml == 2PI ~ 3, and m2 == 2P2 - 3 yields m = 2p - 3.
4.31 2.77 kN @ 4.33,[10.8 kN @ ....• ',
3.273 3.684N, 4.78 ~ N, 47.8 ~ N-cm 4.35 EC: Oi BC: 0.894 k:@

3.275 At A: ~8.91 + ~O.O, lb; at B: - 28.91 - 50.0, lb


4.37 AB: 1610 N <Q; CC: 943 N CD; FC: 3220 N <V
4.39 FH: 732lb <Q; IH: 333lb <Vi TK: 0
3.277 - 720i + 460j lb
4.41 IE: 0; TD: 200 lb QJ.;XT: 231 lb <Q; Cf: 115 lb @
3.279 1.36 N 3.281 245 lb Q) each
4.43 DE: 523lb CDi AD: 178lb @; AB: 266lb @; no.
3.283~ At E: 3600 K lb with - 3000 j lb-ft; at 0:
4.45 (a)Ax = 100 lb, Bx = -100 lb (b)A,B: ISO Ib <Q;
3600 K lb with the couple 6000 j lb
AH: 125lb Q); DF: 75lb CD; DC: Oi only AB
3.285 Answer given in problem.
4.47 667 lb QJ .
3.287,50
4.49 Cf: 400 lb <Q; BL: Oi FI: 424 lb Q)i HC: 0
4.51 DE and CD each: 1050 lb Q); CH: 5000 lb <V
4.53 675lb @ 4.55 1620 N <Q
CHAPTER 4
:4.57 AB: 9.28 k <Vi BC:,.7.02 k @4.59 5.59 k <Q
4.1 BC: 1100 lb @;CD: 880 lb QJ; BD: 500 lb QJ. -4.61 EF: 1.09 kN <Qi BE: 0.07 kN @i BC: 1.26 kN (jJ
AD: 880 lb Q); AB: 200 lb Q) , 4.63 DE: 2900 lb @i QE: 542lb <Qi bp: 2900 lb Q)
4.3 AB: 4.241b @; AH and HC: 3 Ib Q); BH, EF, and 4.65 346 N @ 4.67 If: 21,000 N @; Cf: 292 N @
FC: 21bQJ; BC and CD: 4lb @; CC and DF: 0; 4.69 DA: 500 lb CD; BD: 283lb @; CD: 2831b<Q
BC: 1.41lb Q); DC: 2.831b QJ; DE: 2.831b @ 4.71 469lb <V '
4.5 CD: 800 N@; BD: 800 N QJ; AE: 800 N @; 4.73 (a)at A: 33.3, lb; at B: - 751 lb. at C'
BE: 2000 N @; AB: 2260 N QJ; BC: 1130 N Q) - 251 - 33.3, + lOOkIb (b)AD: 33:3 lb @i
DE: 1130 N <Q ' CD: 108lb QJ (clBC: OiAB: 0; BD: 75 lb QJ (dID

_._~
Appendix E Answers to Odd~Numbered Problems 599

4.75 AB: OAI0P CD; BG: 0.273P ~ wx2 wL wLx wx3


4 119 V = - - - and M = -- - --,
4.77 347 N ~ each • L 4 4 3L
4.79 B1: 20S0 N ~; B2: 20S0 N CD; B3: 0
for 0 < x ::; !:.-.For!:.- ::; x ::; L, obtain Vand M
4.81 B1: 307 N ~; B2: 2100 N CD; B3: 1190 N ~ = -2 2--
.,"'4:83 Ia)'Starhvith'tetrahedron GBDF; (b)67~50 lb ~ "'by symmetry.
4.85 - 101 + 5; kN with the couple - 7 .5k kN . m 4.121 ForO ~ x ~ 3ft, V= -SOOlband'
~
.. 4.87 ~750N' m M = SOOxlb-ft; for 3 ~ x ~ 9 ft,
V = - 50x2 + 900x - 3050 Ib and
500 N
M = 50 x3 _ 450x2 + 3050x - 3150 lb-ft.
4.89 On A-A, 20 ! lb with 20 ;:J lb-in.; on B-B, 20 jib 3
with 40 C> lb-in.; on G-G, 20 ! lb with 40 ;:J lb-in. 4.123 In the six intervals, from left to right,
(V, MI = (-31, 31x);
4.91 At AA,
Ik
M ; at BB,
2k
~ 8k.ft 2000
-. -' (x - 0.1)2 - 31,---(x
[ 6
1000 ]
- 0.113 + 31x ;
8k-ft(3) :y 9"
(IOOx - 41, -50x2 + 41x + 1); (19, -19x + 19);
.

(19, -19x - SI): (- 200, 200x - 300) with units of


at GG, (N,Noml.
500
3k 4.125 (a)For 0 < x < 0,6 m, V = 3X2 and
4.93 75 ! N with 45 ;:J Nom (or V = 75 N; -500 d
M = 45N m) M = __ x3; for 0.6 < x < O.S, V = 60 an
0

9
4.95 16.71 N with S c> Nom (or V = 16.7 N; M = -60x + 24; forO.S < x < I, V = 660 and
M = -S N° ml M = - 660x + 504. Units are N for V, and Nom for
4.97250 i N with 125 c> Nom (or V = -250 N; M. (b1660 N for O.S < x ::; 1 (c) -156 Nom at the wall
M = -125 N . m) 4.99 40 i lb and 260 c> lb-ft 4.127 V = 400x lb; M = 1500x - 125001b-ft
4.101 On EB at E, 375 -4 lb with 92~ ;:J lb-ft; 4.129 Vexlremes = 13.31b for 6 < x < 12 ft, and
on FG at F, 5001 + 500; lb with 200k lb-ft _ 46.7 lb at x = 0; Mexltemes= '109 Ib-ft at x= 4.67 ft,
4.103 (a)At the ends; (b) 216 kN . m at the center and 0 at x = 0 and 12 ft
4.105 Pab!L, at x = a G
4.107900 N at center; IS00 N . m at the wall 4.131 V = constant = -..,- throughout;
L
4.109 (al I sol
+ 3000; lb arid IS000k lb-ft M = 0 at x = 0 and G at x = L
(bIN = 150Ib~forO::;x<6ft;N= 150lbQ)for
4.~;e;e{rextremes= 500 lb for 0 ::; x < 3 ft and - 500 lb
6 < x ::; 12 ft; V = 250x - 3000 lb, 0 ::; x ::; 12 ft;
for 3 < x ::; 6 ft, Mextremes= 0 at x = 0, 6 ft and
M = - 18000 + 3000x - 125x2 lb-ft, 0 ::; x ::; 12 ft
-1500 lb-ft at x = 3 ft
4.111 (a) 27000 iN with 4S000 N . m (b) N = 0; 4135

Vextremes = 1500 kNiat x = 6- m, and - 1200 kN
V = - 27000 + 9000x - 563x2;
at x = 6+ mi Mextremes= :t IS00 kN m at x = 2 and
M = -4S000 + 27000x - 4500x2 + ISSx3
0

6 m, respectively. For 2 < x < 6 m, V = 300(x - 11kN


2P L 2L PL L
4 113 V = - for - < x < -; M = - at x = - and M = -150x2 + 300x + IS00 kN m. 0

• 3 3 3 9 3 4 • 137 Vextremes = 20 kN atx = 9- m, and -16 kN for


-PL 2 O= < X < 3m., M extremes = 4S kN m at x= 3 m, and
and -- at x = - L 0

9 3 ~ IS kN . m at x = 9 m
-3P L 3PL L
4.115 V = _. - for 0 < x < -; M = -S- at x = -2
4 .2.
4 •139 Vextremes=qoL
n
at x= !:.-,and 0 at x = 0 and L;
2
4.117 V = P for 0 ::; x ::; L; 2q L2 .

PL atx
T . (L)+ I -:.
Mextremes =
.
_0_
n 2
atx = 0, andOatx = L

(~r
M = ="2 ; 4 • 141 Vextremes = 550 lb f6r 6 < X < 12 ft, and
. .

M = - P: at x =
-1000 lb at x = 12+ fti Mextremes= 1200 lb-ft at
x = 4 ft, and -2500 lb-ft at x = 12 ft
600 Appendix E Answers to Odd-Numbered Problems

4.143 Vextremes = 83.3 N for 6 ~ x < 9 m, and - 100 N 5.53 (a)Answer given in problem; (b) rolling to the right.
for 9 < x ~ 12 m; Mexlremes= 0 at x = 0, and 5.55 P = 40 lb to the right; 40 <- lb by slot on .A;
- 500 N . m at x = 9 m '60 i lb by track on C.
4.145 Vextremes = 14.4 N for 0.3 < x ~ 1.2 m, and 5.57 0.350 5.59 0.511
-5,6,N.for.O.~,J(_.< .•O.3.m.;,..Mextremes
=.22.4N . mat 5.61 (a) Both f and N increase with P; (bl 0.2;
x = 0.6+ rn, and -2.64 N . m at x = 0.6- m (c1250i + 1250,lb 5.6331.3 N 5.650.333
4.147 50.7 ft from P; 3310 Ib at P 5.67 112 N 5.69 I1T ~ I1p/[( 1 - I1pltan 8° + I1p)
4.149 y= H( 1 _ cos n:) 5.71 (a) 0.819 cm (in tension) (bl 0.588
5.75 982 N 5.77 115 N 5.79 0 5.81 (cot a)/2
5.731270 N

4.151 Answer given in problem. 5.83 0.208 5.85 563 N 5.87 3.61°, tips
4.153 214ft; TA = 175,OOOlb, TB = 144,OOOIb 5.89 lal 115 lb (b) 0.577 5.91 (a) 2.05; (bl 1.45
2n+2(X/Lt+2 - 2(X/L)] 5.93 85lb 5.95 0.192 5.97 52.9 N . m
4.155 fL ------- n .?!
[ 2+ 2 - 2 5.99 1.55lb
5.101 If tan a > 110' slips first at P = 110mg. If
4.157 I~cosh-' ,jK/(K - 1), where K = (2Tqolo)2 tan a < 110' tips first at P = mg tan a. If tan a = 110.1
4.159 2.3 m 4.161 (a) 1.3% (bI6.8% (cI38.3% slips and tips simultaneously at P = l1omg.
4.163 2.95 N/rn 5.103 mg(32110 + 3x)/(32 - 311oX) 5.105 213
5.107 24.0°
4.165 (a1120 jib; (b) 57.4 -> Ib with line of action 8.3 ft
above B 4.167 qoLj 2.06L; 0.152L 5.109 (a) :EMlineOB = 0 shows that N is in plane ADB;
4.169 5, = 3.94 ft; 52 = 4.61 ft; T, = 12401bj (b) ,j x2 + y2/(2z1
T2 = 1110 Ib; T3 = 14001b 5.111 la) 240 lb, slips only on plane and to the right as .A
4.171 Assuming a uniform beam, turns ~ (b) 480 lb, slips only on B and to the right as .A
(a)cP, = cP2 = 39.9° (b) Each = 156lb turns ~
lc) 6.42 ft above the horizontal line through A 5.113 B is needed for equilibriurn;,Vl'tJmin= 29.2lb;.
4.173 (a) 0.436 m (b) T, = 600 N; T2 = 557 N;
T3 = 572 N 4.175 60°
73.2
'/60
li-
0

lb 5.115 12 ft 5.117 H ~ 1.21 ft and


H ~ 12.1 ft5.119.204 N 5.121 80.4 -> lb
CHAPTER 5
5.123 cP = sin-'(~} I1min= TI,jW2R2 - T2
5.1 (a) 60 lb; (b140 Ib 5.3 31.0° 5.5 82.9lb
5.7 94.6 N 5.9 28mg129 5.11 5.71° 5.125 cos-'[(16,j16112 + 9 + 27)/(25,j16112 +9)]
I
5.13 166 N . m 5.15 18 ~ W.• ~ 36lb 5.127 W(,j4112 cos2 cP - 2 sin2 cP - J2
sin cP)/2
5.17 115lb; 0.577 5.129 0, = 26.2r; O2 = 59.67°; T = 0.7116mg;
5.19 (a) f = 601b and fmax= 75lb; (bl R = (60 N'eft = 0.8967mg; Nright 7' 0.5050mg 5.131 0.725W
-> + 1011lb 5.133 72.0° 5.135 (a)Answer given in problem;
5.21 1.37mg at the interface with the bar (b)I1min= 1m2 --mil cot 0/(2m, ~'m2) 5.1371610 N
5.230.857, the same because fiN doesn't change. 5.139417 N 5.141 Yes
5.25 100 lb-ft 5.27 115 N 5.143 No. F = 261 N needed.
5.29 Nrear = 140 lb; Nfront = 43.2lb; f = 49.2lb; 5.145 53.3lb ~ WA ~ 262lb 5.147 125lb
I1min= 0.351 5.149 25.2lb (tipping governs) 5.151120 lb
5.31 (M + m)g(R/I) sin cP -'---5.153 13:8Plb-in.if"Pisin'pounds. -5.155 275lb
5.33 18.4° 5.35 0.625 5.157 (aI15.8 Nom (b) 26.3 N . m
5.37 (x, y) = (0.395, 0.500)m 5.159 19.5 lb to 49.0 lb .5.161 0.468 FoR
5.39 (a) 2 WI sin cP ~(b) Answer given in problem. 5.163 8.18lb; 143lb and 4.09lb
5.41 0.0577; 25.8lb 5.43 (a120 lb; (b) 20 lbeach; 5.165 6048lb; 3.59lb 5.167 36.7°; 0.554
(c)Mr. A; 0.125 .5.45 No; 6.56 -> lb 5.16988.5lb-in. 5.171 Answer given in problem.
5.47 51.8 lb; answer is same if P pushes to the left. 5.173 1O.7lb-in. 5.1750.50511PR
5.49 0.373L; before slip occurs have 5 unknowns but 5.177 2npl1(D3 - d3)/(3 sin f3l 5.179 282lb-in.
only three independent equations. 5.51 1370 lb 5.181 same as 5.129 5.183 107° to 116°
Appendix E Answers to Odd-Numbered Problems 601

CHAPTER 6
n
7.23 bh3. n [ ------
2
+ 4n +----
7 J- 7.25 1.82 cm 4
6.1 Answer given in problem.
12(n + 3)(n + 2)2
6.3 x
= 12r sin(exl2j[/ex 6.5 2.10 6. 7 (0.549Q, 0) ft 7.27 r
(ex
3 'sin a: - -4 sin 2 -a:)
- + --
t
6.9(1.00,O.393)ft 6.112.73in. 6.134.50in. 2 2 a: 2
'''6.15T'60'iri ..''''6;'1'1T~T5'h--6.19 (1~40, =-O~160)'mm -.7.29 Answers given in problem. 7.31 120 ft4
6.21 Z = 3 ft6.23 Answer given in problem. 7.33 0.0361 S4; 0.0313 S4 = 0.867 of Ie for the triangle,
6.25 (t t il m 6.27 5 cm 6.29 (a, 2.42) in. which has some areas farther from the axis.
6.31 (1.03, 2.03) in.; no difference to 3 digits 7.35 7.15 m4
>"-.-
6.33(0.255, 0.225, -0.0200) m 7.37
6.35 Answer given in problem. 6.37 (- 0.05, 0.1171 m
6.39 lines: (2.5, 1.0) cm; triangle: (2.67, 1.0) cm
bh3n[
2
n + 4n + 7
12(n + 3)(n + 2)2
J
+ hb3n[ n
2

(3n + 1)(2n + Il2'


]-

6.41 (a, 6.24) in. 6.43 (2.39, 2.39) in. 7.39 0.894 m 7.41 83.0 f4 7.43 l4bh3
6.45 (a, -0.849) cm 6.47 (33.5, 17.0) cm 7.45 0.000235 m4 7.47 453 in4 7.49 2.53 ft4
6.49 (6.67, 2.50) in. 6.51 (6.19, 5.77) in. 7.51 8,800,000 cm4 7.53:0.806 ft4 7.550.806 ft4
6.53 (a, -4r3/[31t(2R2 - r2111 6.55 (0.943, 1.19) in. 7.57 0.0966 m4 7.591560 in4 7.61 8070 in.4
6.51 (a, 0.584R) 6.59 3.46 cm 6.61 (0.397, 0.260) m 7.63 2.49 S4 7.65 0.252 m 4 1.67 0.326 m4
6.63 (3.94,2.85) in. 6.65 (1.25, 1.25) m 6.67 0.494 7.69 5J3 s4/16 7.71 21.1 in.4, the same as table Value.•
4 7.73 6.91 in.4 7.7515.4 in.4 7.77194 in.4
6.69 -(a + b); yes
7.79 (a) 77,900 mm4 (b) 538,000 mm4
3n
2r sin ex) 2r - 7.81 Ixc = 1740 in.4; IyC = 666 in4
.-0
6.71 lim ( ---'
3ex
6.73 (a, -0.I98RI
== -
- 6.753.00 in.
3
= Y for the triangle
7.83
th3 tbh2 tb3
(a) Ix = 12+ -2-; Iy = 6 (b11.81

6.77 (10.4, -3.81,35) mm 6.79 I 6.811.73R 7.85 62.5 in.4 7.87 Answers given in problem.

6.83 .(21 21
7'7'2
!') 6.85
5'5'
(_!. !. _!.)
2
7.89 Iyy = 26.6 in.4, right on; ky =,1.55 in.-, off by less
than 1% 7.91 377 A 7.93 -R4/8
6.87 Answer given in problem. 7.95 ~ in4; 3/ in.4; -38 in4
7.97 Answers given in problem. 7.99 - 1.05
6.89 (0.840,0, 0.560) in.
6.91 25.4° 6.93,(0,56.6, 0) mm
7.101 19.0 in4 7.103 -0.000172 m4
7.105 -1.33 ft4 7.107 -1.36 ft4
J J
6.95 roc = rp dV 1 P dV = (if p = constant)
7.109 -0.0466m4 7.111-1700in4
J J
r dV 1 dV = (if t is small and constantl
7.113 I 1,300,000 cm4
JrtdAIJtdA = JrdAIJdA 6.97619in.2
6.99 1580 cm 2 6.101 307 ft 2 6.10392.6 m2 7.115 12.4in.\ 79.9in.4; -12.4in.4
2
6.105 17.0 in. 6.101 x = a, y = 4R/(3nl 7.1170.267 in ; 1.16 in.4; -0.533 in.4
4

6.109 5.44 S2 6.1111.20m3 6.1132.67 m2 7,1191.41 in.4,64.9°clockwisefromxe;0.0174in.4,


2
6.115 2n Rrt
25.1 ° counterclockwise from Xc
6.117 Ax = 250,0'00 mm3; V = 1.57(1061 mm3
7.121 (iiih4/24, h4/72 (respectively normal and parallel
6.119 347 in.3 6.121 nr(nR + 2rl
to the hypotenuse) (bl If Ix = I)' and Ix)' = 0, the circle is
a point. (cl The triangle contributes half the inertia of
6.123 nr(n; + r) 6.125 n2Rr2/2 the square in all directions. (d)Answer given in
nr2 problem. 7.123 7b4/6; b4/6
6.127 nr2(3nR + 4r)/6 6.129 -(3nR - 4rl 7.125 Imax~=' 52:0 in.4 at 27.5° c;from Xci
6
Imin = 5.60 in.4 at 62.5° ~ from Xc.
6.131. (1.l3, LSI) ft 6.133 5.05 ft
7.127 (al Ix = 14.96 ft4, Iy = 10.51 ft4;
CHAPTER 7 (b)1xc. = 3 091 ft4, I Yc = 3.883 ft4; the results check.
7.1 la) bh3/3; (b)yand Ycare the same line!
7.3 4.27m4 '1;5 25.1 cm4 ~ 7.7 tr3(a: --'- sina:1/2 CHAPTER 8
7.9 bh3nl[3(n + 311 7.11 Answer given in problem.
1.13 bh(4h2 + b21/12 7.15 bh(12h2 + b21/48 8.1 mg tan 8; the limits are as stated. 8.3 5250 lb
7.17 extr3 7.19 n(R6 - Ri)/2 7.21 bh3/36 2

I
I
..I
602 Appendix E Answers to Odd-Numbered Problems

8.543.7N 8.7AtD,2lkN;atB,4jkN
8.981 = 26.27°; 82 = 59.6r; T = 0.7116mg;
Nlcf! = O.8967mg; Nrigh! = 0.~5(omg W r)
8.11.P ~ ~1.5.•~~:aIF,.8:13 2 1- Ii ="4
8.15 21.3 j N 8.17 P =Ia + 2b + 2e) cot 8 W
2a
2
8.19 (a)5wJ9sin {J + cos2{J
4 sin {J
Ib)5w 25J sin {J + cos2 {J
2

12 sin {J
5 _ 13 ~
(c) - - W(cot fJli - - Wi; no 8.21 2860 jIb
4 4
8.23 (a) 779 q lb-ft (b) 300 -+ lb,

300lb
300lb
5

450lb
8..
Yo:
300lb 8.6
600lb
D A

(c)Answergiven in problem. 8.43


8.25 Answer given in problem.
V
8.27 9140 lb
(I b)
8.29 P = WI2
8.31 P = W (independent of a and b!j
8.33 Answers given in problem. (Consider thin
horizontal and vertical cylinders of fluid, respectively.) x (ft)
8.35 Answer given in problem. -18"
8.37 132,000 lb, normal to the area at depth 75.004 ft.
(Note from the fifth decimal digit that, as depth M
(lb.ft)
increases, distance between centroid and center of
pressure decreases!j
8.39 22.1 lb normal to the area at depth 2.130 ft
8.41 (xp, Yp, zpl = (6.98, 0, 0) ft
x (ft)
10ft

" = specific weight, Ib/ftJ


---:::/~~ __~~'l!I!I~---- __ II!!"'_---_ ••~------------------------------

Appendix E Answers to Odd-Numbered Problems 603

rrydh2 APPENDIX A
8.45 -4-- -> at center of door 8.4772y •...
.A.1 -1 + 5, + 6k; sl + 7, - 14k A.3 -38
8.49 105y -~ ' at the center of pressure of A.5 2281 + 456, + 684k; 120; 4041 - 112, - 60k;
5121 :- 456, - 240k
Example 8.7 """A.7 (a)6.51 - 20.5, - 0.3k (b) -1 - 29.2, + 6k
8.51 Answer given in problem. lc) - 28.0 (dl - 52.21 + 30.6, + 140k (e) -1010
8.53(-15.61+ 26.6jl x 1061b, 7.71 ft to the left ofD (f) 1931 - 7.14, + 73.4k (g)iO.S061+ 0.863,
(h) (0.189, -0.943,0,2761 --
8.55 3. 74yR2L 48~ through center of quarter circle A.9 la1171- 21, + 19k (b1l41 - 20, + 11k (c168
(dl ~ 1401 - 87, + 20k Ie) -173 (f1671 - 180, - 314k
213HZ + 3HR + R21 (g)0.1411 + 0.990klh) (0.487, -0.649,0.584)
8.57 -312H + Rl below the water line A.l1 - 11; A x B = -1 + 19, + 7k
8.59.j3B A.13 Area of shaded parallelogram = Area of dashed
8.61 (a) -515,00Di - 98,1OojN on the line rectangle = (Base)(Heightl~ IAIlIBIsin 0) and

Y = 0.190x - 1.81 with


YL_ at A(~l at A,
. IA x BI = IAIIBIs.in 0

x
515,0001 - 834,00ojN; at B, 932,0001 N

8.63642;000 40.~ N, along the line


y = -0.843x + 5.33 8.65.Answer given in problem. ..15 2.91 m A.17 9.70
8.670.293 8.69 0.091349748 A;19Ial!1+ H (b)1 (cl 1431.'-/3' + gk

I".
Index

Acceleration of gravity,S .Bending moment, 291, 300


Action, line of, 13 dia.gram, 319
Action-reaction principle, 121, relationship to .shear force; 318
\
180 sign convention, 301
Angle of friction, 360 Bodies,'.interacting,t180>.:"~,,
Archimedes' principle, 557 Body, HJ~
Area deformable, 118
centroid of, 422 free, 119
first moment of, 422 rigid, 118
moments of inertia, 468 Body fcirce, 95
polar moment of inertia, 473 Buoyancy, 557
products of inertia, 494
second moments of, 468 . 'Cables! 330
Associative property for addition of catenary, 336
vectors, 571 parabolic, 332
Axes, principal, 509 Center
Axial force, 291 of:buoyancy, 558
of gravity, 109
Ball and socket, 125 of mass, 109
Beam, 300 of.,pressure, 542
axial force, 300 Centroids, 421
bending moment, 300 of areas, 422
cantilever, 305 by composite parts, 436
shear force, 300 oOines, 424
simply supported, 302 of 'Volumes, 421
twisting moment,. 300 Coefficient of friction, 358
Belt Commutative property'
friction on flat, 398 addition of vectors,S 70
friction on V-, 402 scalar (dot) product, 573

604
r--/::::----------------------------------------

Index 605

Components gravitational, 4
scalar, 14,572 internal, 238
vector, 14,572 moment of, 27
Composite parts parallel system, 8 1
area moments of inertia, 483 resultant, 66
centroids,436. shear, 291
Compression, 143, 242 surface, 95
Concentrated force, 13 units of, 5
Concurrent forces, 75, 132 work of, 519
Constraint, 123 Frame of reference, 3
Coplanar forces, 76, 132 Frames, 182
Coulomb friction, 357 nonrigid, 196
Couple, 47 rigid, 194'
moment of, 48 Free-body diagram, 119
work of, 521 Friction, 125, 357
Cross (vector) product, 28, 575 angle of, 360-
belt,398
Density, 108 coefficient, 358
Dimensions, 6 Coulomb, 357
Direction cosines,S 72 disk, 409
Disk friction, 409 dry, 357
Displacement, 5 17 kinetic, 359
virtual, 521 screws, 403
Distributed forces, 95 static, 358
Distributive property
for scalar product, 573 Gravitation
for vector product, .576 Newton's law of, 4
Dot (scalarl product, 20, 573 universal constant,S
Dry friction, 357 Gravity
~. acceleration of,S
f .- center of, 109
Equilibrium, 54
equations, 54, 126 Gyratiop, radius of, 480
stability of, 137 Hinge, 122
!'- Equipollence, 57 Hydrostatic pressure, 540
Euler's laws, 4 on curved surface, 548
External forces, 54, 117 on plane surface, 541
Face Impending motion, 359
left-hand, 301 Indeterminacy, static, 131
right-hand, 301 Inertia
First moments, 422 area moments of, 468
Foot,S area products of, 494
Fluid pressure, 103, 540 parallel-axis theorems, 476, 499
Force, 3, 12 principal axes, 509
axial, 291 Inertia!'frame of reference, 4, 55
body, 95 'Interacting bodies, 180
buoyant, 557 InternaUorces, 238
components, 14
concurrent system, 75, 132 Joints, method of, 244
coplanar system, 76, 132
dimension, 6 Kilogram,S
distributed, 95 Kilopound, 243
equipollent systems,S 7 Kinem:hics of rigid bodies, 517
external, 117 Kinetic friction, 359
friction, 357 Kip, 243
/

606 Index

Lead of a screw, 406 on curved surfaces, 548


Left-hand face, 301 on plane surfaces, 105,431,
Length, units of, 5 541
Line of action, 13 Pressure, resultant, 102, 431
Loads, 117 Principal axes of inertia, 509
Product
Machines, 182 of inertia of area, 494
Magnitude of a vector, 572 scalar, 20, 573
Mass, 3 scalar triple, 578
center of, 109 vector, 28, 575
units of, 5 vector triple, 580
Mechanics, 2 Pulley, 134
Metacenter, 558
Meter, 5
Radiusof gyration, 480
Mohr's circle, 503
Reactions, 117
Moment
Resultant
bending, 291, 300
of concurrent force system, 75
of a couple, 48
of coplanar force system, 76
of a force about a line, 37
of distributed forces, 95
of a force about a point, 27
of parallel force system, 81
twisting, 300
of pressure on a flat surface, 105,
Moments of inertia of areas, 468
431 'I
by composite parts, 483 '1 .
screwdriver, 84 .'!. :

maximum and minimum, 509 Revolution . .!


Mohr's circle, 503
surface of, 456
parallel-axis theorem; 476
volume of, 457 "
polar, 473 -\
Right-hand face, 30 l' !.
principal, 509 f;
Right-hand rule, 28, 575
radius of gyration, 480
Right-hand system of unit vectors,
for rotated axes, 503
28,577
Rigid body, 118
Newton, unit of force, 5
displacement;" 5 1T .•
Newtonian frame of reference, 4
Rigid
Newton-Raphson method, 592
frame, 194
Newton's law(s)
truss, 260
of gravitation, 4
Rolling, 367
of motion, 3
Normal stress, 106
Scalar, 571
Pappus, theorems of, 456 components of a vector, 14,572
Parallel-axis theorem product, 20, 573
for area moments ofinertia, 476 Screwdriver form of resultant, 84
for area products of inertia, 499 Screws~friction in, 403
Parallel force system, 8 1 Second moment of area, 468
Parallelogram law, 12, 570 Second, unit of time, 5
Particle, 3 • Sections, method of, 263
Pascal's law, 103, 540 Self-locking ..
Pitch of a screw, 406 pipe wrench, 372 -
Plane truss, 238 screws, 406
simple, 260 Shear force, 291, 300
Point of application of a force, 12 diagram, 315
Polar moment of inertia, 473 relationship to bending
Position vector, 28 :moment,318
Pound, 5 relationship to distributed load,
Pressure, hydrostatic, 540 314
center of, 542 sign convention, 301
/

Index 607

Sl units, 5 Twisting moment, 300


Sign conventions, shear and Two-force member, 134
moment, 301
Simple truss, 260, 281 Unit vector, 14,571
Slug, 5 Units, is
Space truss, 281 Unstable equilibrium, 137
simple, 281
Specific weight, 540 V-belt,402
Spring, 179, 528 Varignon's theorem, 30
Stability of equilibrium, 137 Vector, 12,570
Static indetenbinacy, 131 addition, 12,570
Statically components, 14,572
determinate truss, 259 scalar (dot) product, 20, 573
l\:.' indeterminate frame, 194 triple scalar product,S 78
i indeterminate truss, 259 triple vector product, 580
Statics, 2 unit, 14,571
Submerged surfaces, pressure on, vector (cross)product, 28, 575
541,548 Virtual displacement, 521
Surface forces, 95 Virtual work, 522
principle of, 522
Tension, 143, 242 of spring forces, 528
, Three-force body, 127, 180 for system of bodies, 529
Time, 3
i, Transfer (parallel-axis)theorem Wedge, 395
- t
..
f=
for moments of inertia of
areas, 476
Weight, 109
specific, 540
for products of inertia of Work
areas, 499 of acouple, 521
,,'.' Triple products of vectors,S 78, 580 of a force, 519
t
Truss, 238 virtual, 522
i compound, 261 Wrench (screwdriver) form of
•i plane, 238
rigidity, 260
resultant, 84- I
i
I
\ simple, 260, 281 Zero-force (truss) members, 255 --1
space, 281
stability, 260
static determinacy, 259
~.
, J

, .
~ T
"t-,. J

i '
... \
1

I
.1

,
~~>~ .1

Вам также может понравиться